Вы находитесь на странице: 1из 240

A.

RATIONAL NUMBERS: THE SET Q


1. Understanding Rational Numbers
Definition
The ratio of an integer to a non-zero integer is called a rational number. The set of rational
a
numbers is denoted by Q. Q = { | a, b  Z, b  0}
b

2. The Set of Positive Rational Numbers


If a rational number represents a point on the number line on the right side of zero, then it
is called a positive rational number.
a
In short, is a positive rational number if a and b are both positive integers or both nega-
b
tive integers.
2 –2 2
For example, and are positive rational numbers, and denoted by .
7 –7 7

Definition

The set of positive rational numbers is denoted by Q+.


a a
Q+ = { |  0 and a, b  , b  0}
b b

3. The Set of Negative Rational Numbers


If a rational number represents a point on the number line on the left side of zero, then it is
called a negative rational number.
a
In short, is a negative rational number if a is a positive integer and b is a negative integer,
b
or if a is a negative integer and b is a positive integer.
–5 5
For example, and are negative rational numbers. We can write negative rational
4 –4
5 –5 5
numbers in three ways: –   .
4 4 –4

Definition
The set of negative rational numbers is denoted by Q–.
a a
Q– = { |  0 and a, b  , b  0}
b b

10 Algebra 8
A. THE SET OF REAL NUMBERS
1. Understanding Real Numbers
In algebra we use many different sets of numbers. For example, we use the natural numbers
to express quantities of whole objects that we can count, such as the number of students in
a class, or the number of books on a shelf.
The set of natural numbers is denoted by N.
N = {1, 2, 3, 4, 5, ...}

-6 -5 -4 -3 -2 -1 0 1 2 3 4 5 6
Natural numbers

The set of whole numbers is the set of natural numbers together with zero. It is denoted by
W.
W = {0, 1, 2, 3, 4, 5, ...}

-6 -5 -4 -3 -2 -1 0 1 2 3 4 5 6
Whole numbers

The set of integers is the set of natural numbers, together with zero and the negatives of the
natural numbers. It is denoted by Z.
Z = {..., –5, –4, –3, –2, –1, 0, 1, 2, 3, 4, 5, ...}

-6 -5 -4 -3 -2 -1 0 1 2 3 4 5 6
Integers

We use integers to express temperatures below zero, distances above and below sea level, and
increases and decreases in stock prices, etc. For example, we can write ten degrees Celsius
below zero as –10°C.
To express ratios between numbers, and parts of wholes, we use rational numbers.
8 2 3 0 17
For example, , , – , , and are rational numbers.
3 5 7 7 1
The set of rational numbers is the set of numbers that can be written as the quotient of two
integers. It is denoted by Q.
a
Q={ | a, b   and b  0}
b
7 1 2 5 13
2 2 3 4 2
-7 -6 -5 -4 -3 -2 -1 0 1 2 3 4 5 6 7
Some rational numbers
Radicals 11
We can write every rational number as a repeating or terminating decimal. Conversely, we
can write any repeating or terminating decimal as a rational number.
3 321
For example,  0.6, and  0.324 = 0.324242424...
5 990
––
0.6 is a terminating decimal, and 0.324 is a repeating decimal.
There are some decimals which do not repeat or terminate.
For example, the decimals 0.1012001230001234000 ...
3.141592653 ... = 
2.71828 ... =e
R = R+  {0}  R–
R+ is the set of positive 1.4142135 ... = ñ2
real numbers
R– is the set of negative do not terminate and do not repeat. Therefore, we cannot write these decimals as rational
real numbers numbers. We say that they are irrational.

Definition
A number whose decimal form does not repeat or terminate is called an irrational number.
The set of irrational numbers is denoted by Q or I.

Definition
The union of the set of rational numbers and the set of irrational numbers forms the set of
all decimals. This union is called the set of real numbers.
The set of real numbers is denoted by R.
R = Q  Q

Real Numbers For every real number there is a point on the number line. In
other words, there is a one-to-one correspondence between
the real numbers and the points on the number line.
-2.35 -0.5 0.6 e p
-5 -4 -3 -2 -1 0 1 2 3 4 5
Some rational numbers

The real numbers fill up the number line.


We can summarize the relationship between the different sets
of numbers that we have described in a diagram. As we know,
the set of natural numbers is a subset of the set of whole
numbers, the set of whole numbers is a subset of the set of
integers, the set of integers is a subset of the set of rational
numbers, and the set of rational numbers is a subset of the
NWZQR
Q  R set of real numbers. This relationship is shown by the dia-
gram on the left.

12 Algebra 8
1. Understanding Square Roots
Remember that we can write a  a as a2. We call a2 the square of a, and multiplying a number
by itself is called squaring the number. The inverse operation of squaring a number is called
finding the square root of the number.

Objectives
After studying this section you will be able to:
1. Understand the concepts of square root and radical number.
2. Use the properties of square roots to simplify expressions.
3. Find the product of square roots.
4. Rationalize the denominator of a fraction containing square roots.

Definition

If a2 = b then a is the square root of b (a  0, b  0).


We use the symbol ñ to denote the square root of a number. ñb is read as ‘the square root of
b’. So if a2 = b then a = ñb (a  b, b  0).

Here are the square roots of all the perfect squares from 1 to 100.
12 = 1  ñ1 = 1 62 = 36  ò36 = 6
2
2 =4  ñ4 = 2 2
7 = 49  ò49 = 7
32 = 9  ñ9 = 3 82 = 64  ò64 = 8
2
4 = 16  ò16 = 4 2
9 = 81  ò81 = 9
2
5 = 25  ò25 = 5 2
10 = 100  ó100 = 10
2
The equation x = 9 can be stated as the question, ‘What number multiplied itself is 9?’
There are two such numbers, 3 and –3.

Rule
If x  R then
 x if x  0.
x2 | x | 
 – x if x  0.

In other words, if x is a non-negative real number, then x2  x, and

if x is a negative real number, then x2  – x.

Radicals 13
For example, 32  3, ( 32  9  3), and

(–3)2  –(–3)  3 ( (–3) 2  9  3).

We can conclude that the square root of any real number will always be greater than or equal
to zero. ò–9 is undefined. Negative numbers have no square root because the square of any
real number cannot be negative.
ò–9  3, since 32 is 9, not (–9).
ò–9  –3, since (–3)2 is 9, not (–9).

Note
x = ñ9 and x2 = 9 have different meanings in the set of all real numbers.

 ñ9 = 32 = |3| = 3

 If x2 = 9 then x = 3 or x = –3.

EXAMPLE 1 Evaluate each square root.


a. ò81 b. ñ1 c. ñ0 d. ò64 e. ñ9 f. ó0.64

g. –ó100 h. –ó0.09 i. ò–4 j. (–4)2 k. –4 2

Solution a. ò81 = 9 b. ñ1 = 1 c. ñ0 = 0
4 2
d. ò64 = 8 e.  f. ó0.64 = 0.8
9 3

g. –ó100 = –10 h. –ó0.09 = –0.3 i. ò–4 is undefined

j. (–4)2  16  4 k. –42  –16 is undefined

EXAMPLE 2 Evaluate each square root.


a. ó100 b. ó121 c. ó144 d. ó169 e. ó225 f. ó361
g. ó400 h. ó625 i. 1225 j. 10000

Solution a. ó100 = 10 b. ó121 = 11 c. ó144 = 12


d. ó169 = 13 e. ó225 = 15 f. ó361 = 19
g. ó400 = 20 h. ó625 = 25 i. 1225  35
j. 10000  100

14 Algebra 8
2. Properties of Square Roots
Property
For any real number a and b, where a  0, and b  0,
ñañb = óab.

For example, 25  16  25  16  5  4  20,

3  27  3  27  81  9,

36 a2  36 a2  6 a ( a  0), and
5  5  5  5  25  5.

Note b0 óa . 6 =
If a  0 then a0
 ña . ñ6

ña  ña = óa  a = a2  a.

Mathematics is a universal language.

EXAMPLE 3 Simplify each of the following.


a. ñ2 ñ8 b. ñ7 ñ7 c. ò50ñ2 d. ò25ñ1 e. ó576 f. ò10ò90

Solution a. ñ2ñ8 = ó28 = ò16 = 4 b. 7  7  7  7  49  7

c. ò50ñ2 = ó502 = ó100 = 10 d. 25  1  25  1  25  5

e. 576  36  16  36  16  6  4  24

f. 10  90  10  90  900  30

Property
For any real numbers a and b, where a  0, and b > 0,
a a
 .
b b

24 24
For example,   4  2, and
If a > 0 then 6 6
a a
= = 1 = 1. 1 1 1
a a   .
49 49 7

Radicals 15
EXAMPLE 4 Simplify the expressions.
25 50 16 1 1 625
a. b. c. d. e. – f.
9 2 49 64 100 144

24a 3 a5  b6 xy
g. h. i.
2
6a ab x3  y3

25 25 5 50 50
Solution a. = = b.   25  5
9 9 3 2 2
16 16 4 1 1 1
c.   d.  
49 49 7 64 64 8
1 1 1 625 625 25
e. – – – f.  
100 100 10 144 144 12

24a3 24 a3
g.   4  a2  4  a2  2a
6a 6a

a5  b6 a5  b6
h.   a4  b4  ( a2  b2 )2  a2 b2
ab2 ab2

xy xy 1 1 1
i.    
3
x y 3 x3  y3 x2  y2 2
x  y 2 xy

Property

For any real number a and n  Z, ( a )n  an (a  0).

Proof
( a )n  a  a  a  ...  a a  a  a  ... a  an

n factors of ña n factors of a

For example, ( a )2  a2  a,

( 5)3  53  125, and

( 2 )8  2 8  256  16.

16 Algebra 8
EXAMPLE 5 Evaluate
(ñ2)4 + (ñ5)4 – (ñ5)2 – (ñ2)6.

Solution ( 2 )2  ( 5)4 – ( 5)2 – ( 2 ) 6  2 4  5 4 – 5 2 – 2 6

 (22 )2  (52 )2 – 52 – (23 )2


 22  52 – 5 – 2 3  4  25 – 5 – 8  16

3. Working with Pure and Mixed Radicals


Definition

A radical expression is an expression of the form n


a.

index radical sign


ña
n

radicand

Square roots have index 2. However, we usually write square roots in their shorter form, ña:
2
a a

Definition
A mixed radical is a radical of the form
x  n a (x  Q, x  {–1, 0, 1})

For example, 3ñ2, 6ñ7, and 9ó115 are mixed radicals.


ò55, ò99, and ò27 are not mixed radicals. We say that they are pure radicals.
We can convert between mixed and pure radical numbers to simplify radical expressions.

Property
For any real numbers a and b, where a  0 and b  0,

a2  b  a b and a b  a 2  b.

For example, 8  4  2  2 2  2  2 2  2  2 2,

27  9  3  32  3  32  3  3 3,

32  16  2  4 2  2  4 2  2  4 2, and

50  25  2  5 2  2  5 2  2  5 2.

Radicals 17
EXAMPLE 6 Simplify the expressions.
a. ñ8 + 2ò32 – ò18 + ò72 – ò98
b. 2ò48 + 3ò27 – ó108 + ó243

Solution a. 8 = 22  2 = 2 2 


2 32 = 2 4 2  2 = 8 2
 8  2 32  18  72  98
18 = 3  2 = 3 2
2
 
  2 2  8 2  3 2  6 2 7 2
2
72 = 6  2 = 6 2 
  2  (2  8  3  6  7)  6 2

98 = 7  2 = 7 2
2

b. 2 48  3 27 – 108  243  2 4 2  3  3 3 2  3 – 6 2  3  9 2 3

 8 3  9 3 – 6 3 9 3
 (8  9 – 6  9) 3  20 3

EXAMPLE 7 Write the numbers as pure radicals.


a. 2ñ2 b. 3ñ5 c. 5ñ3 d. 10ò10 e. xñy

Solution a. 2 2  2 2  2  2 2  2  4 2  8

b. 3 5  32  5  9  5  45

c. 5 3  52  3  25  3  75

d. 10 10  10 2  10  100  10  1000

e. x y  x2 y

Property
For any non-zero real numbers a, b, c, and x,
añx + bñx – cñx = (a + b – c)ñx .

Note
ña + ñb  óa+b
For example,
ñ9 + ò16 = 3 + 4 = 7, but ó9 + 16 = ò25 = 5.

18 Algebra 8
EXAMPLE 8 Perform the operations.

a. ñ3 + ñ3 b. 2ñ5 + ñ5 c. 3ñ6 + 4ñ6 d. 10ñ5 – 3ñ5


e. ò50 + ò98 + ó162 f. 5ñx – ò9x + ó64x

Solution a. ñ3 + ñ3 = (1 + 1)ñ3 = 2ñ3


b. 2ñ5 + ñ5 = (2 + 1)ñ5 = 3ñ5
c. 3ñ6 + 4ñ6 = (3 + 4)ñ6 = 7ñ6
d. 10ñ5 – 3ñ5 = (10 – 3)ñ5 = 7ñ5
e. ò50 + ò98 + ó162 = ó252 + ó492 + ó812 = 5ñ2 + 7ñ2 + 9ñ2
= (5 + 7 + 9)ñ2 = 21ñ2
f. 5ñx – ò9x + ó64x = 5ñx – 3ñx + 8ñx = (5 – 3 + 8)ñx = 10ñx

EXAMPLE 9 Compare the following numbers.

a. ñ7 ... 3 b. 3ñ5 ... 2ò10 c. 2ñ7 ... 3ñ3 d. –2ñ3 ... –3ñ2

Solution a. 7...3 b. 3 5...2 10 c. 2 7...3 3 d. –2 3... – 3 2

7... 9 32  5... 2 2  10 22  7... 32  3 – 22  3... – 32  2


7 9 45  40 28  27 – 12  – 18
7 3 3 5  2 10 2 7 3 3 –2 3  –3 2

Property
Let a, b, m, and n be four real numbers, satisfying a = m + n and b = m  n. Then,

1. m  n  a2 b 2. m – n  a–2 b ( m  n)

Proof
1. In order to verify these expressions, suppose that t = òm + ñn.
t2 = (ò m + ñn)2 = (ò m + ñn)  (ò m + ñn)
= (ò m  ñn) + (òm  ñn) + (ñn  òm ) + (ñn  ñn)
(by the distributive property)
= m + (ò m  ñn) + (ñn  ò m ) + n
= m + n + 2ómn (by the commutative property)

a b

 t = a + 2ñb  t = a  2 b
2

2. We can prove the second part in the same way. Try it yourself.

Radicals 19
EXAMPLE 10 Simplify the expressions. Use the property to help you.

a. 32 2 b. 52 6 c. 6  32 d. 6–4 2

e. 5  21 f. 2 3

Solution a. 3  2 2  2  1  2 1 b. 52 6  3  2

2+1 21 3+2 32

c. 6  32  6  2 8  4  2  2  2

4+2 42

d. 6 – 4 2  6 – 2  2 2  6 – 2 2 2  2  6 – 2 8  4 – 2 2 – 2

4+2 42

e. We need a 2 in front of ò21 before we can use the property. Therefore, let us multi-
2
ply the expression by .
2
7+3 73

2 2  (5  21) 10  2 21 7 3 7 3
5  21   5  21     
2 2 2 2 2 2

3+1 31

2 42 3 3 1 3 1
f.  2 3    
2 2 2 2 2

Check Yourself 1
1. Simplify the expressions.
a. ñ2  ñ2 b. ñ8  ò32 c. ò3x  ó12x d. ñ2  ò18
32 12 a b 1
e. f. g. h. –
2 3 3
a b 3 49
2. Evaluate the following.

a. (ñ3)2 + (ñ4)4 – (ñ5)2 – (ñ2)4 b. (ña)4 + (ñb)2 – (ñc)6


3. Simplify the expressions.
a. ò18 b. ò50 c. ò48 d. ò20 e. 5ñ3 – 2ñ3 + ñ3
f. 2ñ2 + 3ñ2 – 4ñ2 g. ò50 – ò18 – ò32 h. ó12x + ó27x – ó48x

20 Algebra 8
4. Write each number as a pure radical.
a. 5ñ3 b. 3ñ5 c. 4ñ2 d. 2ñ5 e. añb
5. Perform the operations.
10 15 27 75 12
a. 6 2 b. 5ñ2 – ñ8 c. ò27 – ò48 d.  –
3 2 4 4 4
6. Compare the numbers.
1 1
a. 3ñ5 and 2ò10 b. and c. –2ñ5 and –3ñ3
2 3
7. Write each expression in its simplest form.

a. 3–2 2 b. 62 8 c. 7 – 2 10 d. 3  8 e. 9–4 5 f. 7 – 48

g. ( 6 – 2 )  ( 8  2 12 ) h. ( 7  1)  ( 8 – 28 ) i. 3– 5  3 5

Answers
1 1
1. a. 2 b. 16 c. 6x d. 6 e. 4 f. 2 g. h. – 2. a. 10 b. a2 + b – c3 3. a. 3ñ2 b. 5ñ2
ab 7
c. 4ñ3 d. 2ñ5 e. 4ñ3 f. ñ2 g. –2ñ2 h. ò3 x 4. a. ò75 b. ò45 c. ò32 d. ò20 e. a2  b
1 1
5. a. 3ò30 b. 3ñ2 c. –ñ3 d. 3ñ3 6. a. 3ñ5 > 2ò10 b.  c. –2ñ5 > –3ñ3 7. a. ñ2 – 1
2 3
b. 2 + ñ2 c. ñ5 – ñ2 d. ñ2 + 1 e. ñ5 – 2 f. 2 – ñ3 g. 4 h. 6 i. 2

EXAMPLE 11 Simplify the following.


1 9
a. 4  21  13  9 b. 6 6 72 16 c. 1 1
16 16

Solution Start from the radical on the ‘inside’ of the expression and move outwards.
a. Start with ñ9, on the inside, and work outwards.

4  21  13  9  4  21  13  3  4  21  16

 4  21  4  4  25  4  5  9  3

b. 1 4 1
6 6 72 16  6 6 72  6 6 72
16 16 4

1
 6 6 72   6 6 36
2

 6 6  6  6 36  6  6  36  6

9 25 5 9 3
c. 1 1  1  1  
16 16 4 4 2
Radicals 21
EXAMPLE 12 a. Evaluate 2 2 2 2 ... b. a a a a ...  7. Find a.

c. x  x  x  ...  5. Find x.

Solution a. Let x  2 2 2 2 ... .

x2  ( 2 2 2 2 ... )2 (take the square of both sides)

x2  2 2 2 2 2 ... (remove a square root)



x

x 2x
2
( 2 2 2 ...  x)
x  x 2x
 (simplify)
x x

 x  2. Therefore, 2 2 2 2 ...  2.

b. a a a a ...  7 c. ( x  x  x  ... )2  5 2

( a a a a ... )2  7 2 x  x  x  x  ...  25
 
5

a a a a ...  49 x  5  25
 
7
x  20
a  7  49
a7

4. Multiplying Square Roots


To multiply expressions containing square roots, we used the product property of square
roots: ña  ñb = óa  b. We can also use the distributive property of multiplication over
addition and subtraction to simplify the products of expressions that contain radicals.
For example,
2ñ8  3ñ2 = 2  3  ñ8  ñ2 Multiply the rational part by the rational
= 6ò16 part and the radical part by the radical
part.
= 64
= 24

ñ2  (ñ3 + 2ñ2) = ñ2  ñ3 + ñ2  2  ñ2
= ñ6 + 2  ñ2  ñ2
= ñ6 + 2  2
= ñ6 + 4
22 Algebra 8
EXAMPLE 13 Perform the operations.

a. ñ2(ñ5 + ñ3) b. ñ3(3ñ3 + 2ñ2) c. 2ñ5(ñ3 + ñ2 + 2ñ5 – ñ7)

Solution a. ñ2(ñ5 + ñ3) = ñ2  ñ5 + ñ2  ñ3 = ó2  5 + ó2  3 = ò10 + ñ6


b. ñ3(3ñ3 + 2ñ2)= ñ3  3ñ3 + ñ3  2ñ2 = 3  ó3  3 + 2  ó3  2
= 3  3 + 2  ñ6 = 9 + 2ñ6
c. 2ñ5(ñ3 + ñ2 + 2ñ5 – ñ7)= 2ñ5  ñ3 + 2ñ5  ñ2 + 2ñ5  2ñ5 – 2ñ5  ñ7
= 2ò15 + 2ò10 + 4ò25 – 2ò35
= 2ò15 + 2ò10 + 20 – 2ò35

EXAMPLE 14 Multiply and simplify.

a. (ñ2 + ñ3)  (ñ2 + ñ3) b. (5 + ñ5)  (5 + ñ5)

Solution a. (ñ2 + ñ3)  (ñ2 + ñ3) = ñ2  ñ2 + ñ2  ñ3 + ñ3  ñ2 + ñ3  ñ3


= ñ4 + ñ6 + ñ6 + ñ9 = 2 + 2ñ6 + 3 = 5 + 2ñ6

b. (5 + ñ5)  (5 + ñ5)= 52 + 2  5  ñ5 + (ñ5)2


= 25 + 10ñ5 + 5 = 30 + 10ñ5

EXAMPLE 15 Multiply and simplify.

a. (ñ2 + 1)  (ñ2 – 1) b. (ñ5 + ñ3)  (ñ5 – ñ3) c. (1 – 2ñ2)  (1 + 2ñ2)


d. (ña + 1)  (ña – 1) e. (ña + ñb)  (ña – ñb)

Solution a. (ñ2 + 1)  (ñ2 – 1) = ñ2  ñ2 – ñ2  1+1  ñ2 – 1  1 = (ñ2)2  12 = 2 – 1 = 1


b. (ñ5 + ñ3)  (ñ5 – ñ3) = (ñ5)2 – (ñ3)2 = 5 – 3 = 2
c. (1 – 2ñ2)  (1 + 2ñ2) = 12 – (2ñ2)2 = 1 – 4  2 = 1 – 8 = –7
d. (ña + 1)  (ña – 1) = (ña)2 – 12 = a – 1 (a  0)
e. (ña + ñb)  (ña – ñb) = (ña)2 – (ñb)2 = a – b (a, b  0)

EXAMPLE 16 Multiply and simplify.

a. 3 5  3– 5 b. 2 2  2– 2 c. a b  a– b

Solution a. 3  5  3 – 5  (3  5)  (3 – 5)  3 2 – ( 5) 2  9 – 5  4  2

b. 2  2  2 – 2  (2  2 )  (2 – 2 )  2 2 – ( 2 ) 2  4 – 2  2

c. a b  a– b  a2 – b

Radicals 23
EXAMPLE 17 Multiply and simplify.

a. (ñ3 + ñ2)  (ñ5 – 1) b. (ñ5 + ñ3)  (ñ7 + ñ2)


c. (2ñ3 + 1)  (ñ5 + 1) d. (3ñ2 – 2)  (ñ5 – ñ3)

Solution a. (ñ3 + ñ2)  (ñ5 – 1)= (ñ3  ñ5) – (ñ3  1) + (ñ2  ñ5) – (ñ2  1)
= ò15 – ñ3 + ò10 – ñ2
b. (ñ5 + ñ3)  (ñ7 + ñ2) = (ñ5  ñ7) + (ñ5  ñ2) + (ñ3  ñ7) + (ñ3  ñ2)
= ò35 + ò10 + ò21 + ñ6
c. (2ñ3 + 1)  (ñ5 + 1) = (2ñ3  ñ5) + (2ñ3  1) + (1  ñ5) + 1
= 2ò15 + 2ñ3 + ñ5 + 1
d. (3ñ2 – 2)  (ñ5 – ñ3)= (3ñ2  ñ5) – (3ñ2  ñ3) – (2ñ5 + 2ñ3)
= 3ò10 – 3ñ6 – 2ñ5 + 2ñ3

5. Rationalizing Denominators
1 3 10 19
Look at the numbers . They are all fractions, and each fraction
, , , and
5 2 12 13
has an irrational number as the denominator. In math, it is easier to work with fractions that

have a rational number as the denominator.


Definition
Changing the denominator of a fraction from an irrational number to a rational number is
called rationalizing the denominator of the fraction. Rationalizing the denominator does not
change the value of the original fraction.
To rationalize the denominator, we multiply the numerator and denominator of the fraction
a
by a suitable factor. For example, if the fraction is in the form , we multiply both the
b
numerator and the denominator by ñb.

a a b ab ab a ab
So,     . Note that and have the same value: they are
b b b bb b b b
equivalent fractions.
Look at some more examples:
3 3 2 3 2 3  2 6 6
      ,
2 2 2 2 2 2  2 4 2
3 3 3 3 3 3 3
     3, and
3 3 3 3 3 3
3 5 3 5 2 3 52 3  10 3 10
     .
2 2 2 2 2 2 2 2 22 4

24 Algebra 8
Definition
An expression with exactly two terms is called a binomial expression. Two binomial expressions
whose first terms are equal and last terms are opposite are called conjugates, i.e. a + b and
a – b are conjugates.

If a  0 and b  0, then the binomials xña + yñb and xña – yñb are conjugates. We can use
conjugates to rationalize denominators that contain radical expressions.
1
For example, let us rationalize . ñ3 – ñ2 is the conjugate of ñ3 + ñ2.
3 2
Therefore, we multiply the numerator and the denominator by ñ3 – ñ2 to rationalize the
denominator.
1 3– 2 1  ( 3 – 2) 3– 2 3– 2 3– 2
      3– 2
3 2 3– 2 ( 3  2 ) ( 3 – 2 ) 2
( 3) – ( 2 ) 2
3–2 1

Remark
(a + b)(a – b) = a2 – b2
(ña + ñb)(ña – ñb) = a – b where a  0 and b  0.

EXAMPLE 18 Rationalize the denominators.


5 3– 2 6 2 3 2 –2
a. b. c. d.
3–2 2 2 2 –1 1– 3 52 5

5 5 32 2 5  (3  2 2 ) 5 3  5 2 2
Solution a.    
3–2 2 3–2 2 3 2 2 (3 – 2 2 )(3  2 2 ) 3 2 – (2 2 ) 2
3 5  2 10 3 5  2 10
   3 5  2 10
9–8 1

3– 2 ( 3 – 2 ) (2 2  1) 3  2 2  3  1 – 2  2 2 – 2 1
b.   
2 2 –1 (2 2 – 1) (2 2  1) (2 2 ) 2 – 12
2 6  3–2 2 2 – 2 2 6  3–4– 2
 
8 –1 7

6 2 ( 6  2 ) (1  3) 6  6  3  2 1  2  3
c.   
1– 3 (1 – 3) (1  3) 12 – ( 3) 2
6  18  2  6 6  3 2  2  6 2 6 4 2
   – 6 –2 2
1– 3 –2 –2

3 2 –2 (3 2 – 2) (5 – 2 5) 3 2  5 – 3 2  2 5 – 2 5 – 2 2 5
d.   
52 5 (5  2 5) (5 – 2 5) 5 2 – (2 5) 2
15 2 – 6 10 – 10 – 4 5 15 2 – 6 10 – 10 – 4 5
 
25 – 20 5
Radicals 25
EXAMPLE 19 Rationalize the denominators to find the sum.

3 2

3 2 2 3–2 2

3 2  3 3–2 2   2 3 2 2 
Solution      
3 2 2 3 – 2 2  3 2 2 3–2 2   3–2 2 3 2 2 
( 3  ( 3 – 2 2 ))  ( 2  ( 3  2 2 ))

( 3  2 2)  ( 3 – 2 2)
( 3  3) – ( 3  2 2 )  ( 2  3) ( 2  2 2 )

( 3)2 – (2 2 )2
3–2 6  6 4 7– 6 6 –7
  
3–8 –5 5
Check Yourself 2
1. Rationalize the denominators and simplify.
3 5 1 3 3 5 3 5
a. b. c. 2 d. –5  e. f.
7 3 2 5 3 2 6

1 a  b 2 x  y a 3  b4
g. h. i. j.
2 10 a3 x3  y3 a  b2
2. Rationalize the denominators and simplify.
1 2 2 3 5 1 3– 2
a. b. c. d. e. f.
2 –1 3– 2 5 1 6 –2 5 –1 2 3– 2

2 5–2 2 5– 7 a a b 3
g. h. i. j. k.
2 10 – 6 7– 5 a b a– b 3 3–2 7
3. Rationalize the denominators and simplify.
1 1 2 2 1 1
a.  b.  c. 
5 2 5– 2 2– 2 2 2 3 3 3– 3
2 2 3 3 2 3
d. –  e. – 
2 1 2 –1 2 2 3 1 1– 3 3 3
Answers
21 15 30 10 b 2
1. a. b. c. ñ2 d. –ò15 e. ò15 f. g. h. i. j. ab 2. a. ñ2 + 1
7 3 4 20 a xy

5 –1 3 6 6 3 5 4– 6 35  3
b. ñ6 – 2 c. d. e. f. g. 5ñ2 – ò10 + 3 ñ5 – 3 h.
2 2 2 10 2
a – ab a  b  2 ab 2 5 6 –16  3 2
i. j. k. –9ñ3 – 6ñ7 3. a. b. ñ2 c. d. e. 17 3 – 3
a–b a–b 3 6 4 6
26 Algebra 8
EXERCISES 1 .1
1. Evaluate the square roots. 5. Perform the operations.

a. ò36 b. ó100 c. –ó121 a. 3ñ3 + 2ñ3 b. 6ñ5 + ñ5

d. 16x2 e. 25  y2 f. 121  a4 c. –5ñx + 5ñx d. ñ6 – 3ñ6

e. 3ò18 + 2ò72 f. ò80 – ó125 + ò45

g. ò75 + ó108 – ò48 + ò27

h. 9x3  16 x3 – 4 x 25 x
2. Simplify the expressions.
a. ñ3  ñ3 b. ñ5  ñ5 i. 2.25 – 2.89  1.44
c. ñ3  ò12 d. ñ3  ò27

e. ò2x  ò8x f. j. 0.9 – 2.7  1.7  0.4


6 xy  24xy

g. 3a  5a h. 3 2 x  4  18 xy2

3. Simplify the expressions.

50 72 108 6. Write each expression in its simplest form.


a. b. c.
2 8 27

72 x3 32 x3 y2 3xy3 a. 32 2 b. 5–2 6


d. e. f.
2 2x 24 x 12 x y3

c. 8  2 12 d. 11  96

e. 8–2 7 f. 5  24

g. 7–4 3 h. 3 8  3– 8
4. Write each number as a mixed radical.
i. 4  15 – 4 – 15 f. a  2  8a – a
a. ñ8 b. ò72 c. ó243

d. 1000 e. ó125 f. x3 y2 k. 2  2 4  12

Radicals 27
7. Simplify the expressions. 10. Rationalize the denominators.
a. 32  21 – 23  4 3 3 2 1
a. b. c. –
3 6 11
7 2
b.  –  6 9 1 –2
3 9 d. 1  2 e. f.
4 3 1– 2 3 2
1 3 1 4 10
c. 8  16   9  16 g. h. i.
96 3 3 2
3 –1 2 7 –5

3 2 2 10  2 21
d. 13  6  6  9 j. k. l.
2 3 3– 5 7 3

8. Find x in each equation.

a. 2 2 2 2 ...  x b. 3 3 3 3 ...  x
11. Perform the operations.
c. 3x  3x  3x  ...  9 1 1
a. 
3 2

3 2
b. 
9. Find the products. 3 1 3 –1

a. ñ5  (ñ2 + ñ3) 5 2
c. –
b. ñ7  (1 + ñ7) 2 3 – 11 2 3  11

c. –ñ2  (ñ3 – ñ8 + 1) d. 4 2 2 3

3 2 2 3–2 2
d. ñ2  (ñ8 + ò32)

e. ñ6  (2ñ3 + 3ñ2)

f. (3 + ñ5)  (3 – ñ5)

g. (2ñ2 – 3)  (2ñ2 + 3) 12. Perform the operations.


h. (2ñ3+2)  (2ñ3 – 2)
5 2 – 5–2
a.
i. (ò12 + ñ8)  (ñ3 – ñ2) 5 –1
j. (–ò12 + 2ñ2)  (ñ2 + ñ3)
b. 1 2
k. 2 1  2 –1
2  11  72
l. 2 33  2 3–3
1 1 1 1
c.    ... 
m. 5  2 3  2  3  16 – 9 3 2 1 3 2 4 3 100  99

28 Algebra 8
Objectives
After studying this section you will be able to:
1. Understand the concepts of nth root and rational exponent.
2. Write numbers in radical or rational exponent form.
3. Understand the properties of expressions with rational exponents.
4. Use the properties of rational exponents to solve problems.

A. RATIONAL EXPONENTS
1. nth Roots
In section 8.2 we studied exponential equations.
For example, 2n  2n = 2 is an exponential equation. Let us solve it.
2n  2n = 22n = 2 (an  am = an+m)
22n = 21
1 1
1 1
2n = 1, n = . If we substitute for n in the original equation we will get 2 2  2 2  2,
2 2
1

but we know that ñ2  ñ2 = 2. Therefore, 2 2  2  2 2.


Remark
Let x  R – {–1, 0, 1}.
If xm = xn then m = n.
Definition
For any natural number n and a, b  R.
1
If an = b then a = b n  b. a is called the nth root of b. It is denoted by ñb.
n n

In the expression ña, a is called the radicand and n is called the index.
n

Remember that we do not usually write the index for square roots:
ña = ña.
2

Look at some examples of different roots:


52 = 25 and 5 = ò25 ‘the square root of 25 is 5’,
2 = 8 and 2 = ñ8
3 3
‘the cube root of 8 is 2’,
3 = 27 and 3 = ò27
3 3
‘the cube root of 27 is 3’, and
2 = 16 and 2 = ò16
4 4
‘the fourth root of 16 is 2’.
Radicals 29
2. Rational Exponents
Definition
If m and n are positive integers (n > 1), and b is a non-negative real number,
m
n
bm  b n .
m
is called a rational exponent.
n
2 1 2

For example, the numbers 8 3 , 4 2 , and 2 2 have rational exponents.

EXAMPLE 20 Write the expressions in radical form.


1 1 2 3 1 1 1

a. 3 2 b. 32 5 c. 5 3 d. x 4 e. ( x4 ) 4 f. ( a 3 )2

1 1 2
Solution a. 3 2  2 3  3 b. 32 5  5 32 c. 5 3  3 52  3 25
3 1 1 1
4 3
d. x 4  4 x3 e. ( x4 )4  x4 f. ( a 3 )2  a

EXAMPLE 21 Write the expressions with radical exponents.


3
3 7 5
a. 5
35 b. 64 c. x14 d. a2 e. ab2
5 3 14
5 3 7
Solution a. 35  3 5  31  3 b. 3
64  43  4 3  41  4 c. x14  x 7  x2
2 1 1 2
3 5
d. a2  a 3 e. ab2  ( ab 2 )5  a 5  b 5

Rule
If b is any real number and n is a positive integer (n > 1):
1. If b > 0 then ñb is a positive real number.
n

2. If b = 0 then ñb is zero.
n

3. If b < 0 and n is odd, then ñb is a negative real number.


n

4. If b < 0 and n is even, then ñb is not a real number.


n

EXAMPLE 22 Simplify the following.


4 3 4 5
a. 3
8 b. 81 c. –27 d. –16 e. 0
3 4
3 3 4 4
Solution a. 8  23  2 3  2 (8 > 0) b. 81  34  3 4  3 (81 > 0)
3
3
c. –27  3 (–3)3  (–3)  –3 3
(–27 < 0 and 3 is odd)
4
d. –16 is not a real number (–16 < 0 and 4 is even)
5
e. 0 =0

30 Algebra 8
Check Yourself 3
1. Write the expressions in radical form.
1 1 2 3

a. 2 2 b. a 3 c. a 3 d. x 5
1 1 1 1 1

e. ( x2  y3 )6 f. ( x2 )3 g. ( a2  b2 ) 2 h. (( x  y)  x2 ) 3

2. Write the expressions with rational exponents.


4 5 3 6 4
a. 3
2 b. 4 c. a2 d. x  y2 e. x4 f. a8 g. 3
x2  y4 h.
12
x6  y4
3. Simplify the expressions.

a. ò16
3 3 4 6 9
b. 27 c. –64 d. 625 e. 64 f. –512 g. x2  y4 h. 4 16  a4  b8

Answers
1
1. a. ñ2 b. 3 a c. 3 a2 d. 5 x3 e. 6 x2  y3 f. 6 x g. a2  b2 h. 3 ( x  y)x2 2. a. 2 3
1 2 1 2 1 1
b. 2 2 c. a 5 d. ( xy2 )3 e. x 3 f. a2 g. ( x2  y4 )3 h. ( x3 y2 )6 3. a. 4 b. 3 c. –4 d. 5 e. 2
f. –2 g. xy2 h. 2ab2

B. PROPERTIES OF RADICALS
Property
For all real numbers a and b, where a > 0 and b > 0, and for any integer m and n, where
m > 1 and n > 1:

n n n m
1. an  a if n is odd. 5. a am
n
an | a | if n is even. 6.
m
a b
n mn
an 
mn
bm  m n
an  bm
n n
2. a  b n a b m
a
mn
an an
7. n
  mn
n
a a b mn
bm bm
3. n
 n

b b m n mn
8. a  a
n n
4. a  b  an  b and n
an b  a  n
b

Look at the examples of each property.


3 5
1. a. 3
27  33  3 b. 32  5 (2)5  –2
4 4 4
c. 16  2 4 | 2| 2 d. (3)4 | 3| 3

Radicals 31
a. ñ5  ñ4 = ó54 = ò20
3 3 3 3
2. b. 2  4  324 8 23  2
5
c. x5 y5 xy
3
8 8 81 81 3
3. a.   4 2 b. 3
 3
 27  3
2 2 3 3
4
64 x2 64 x2 4 4 4 4
c. 4
4  16 x  4 2 4  x  2 4  x  2 x
4x 4x
3 3
4. a. 2  3  3 2 3  3  3 8  3  24
4 4
b. x  x  4
x4  x  x5
3 3
c. x3  y6  z2  3 ( x  y2 )3  z2  x  y2 z2
6 6 32 3 5 3 2 6
5. a. 4  22  22  2 b. 3 52  25
3 23 2 3 6
6. 5 3 53  32  6
5 3  32  6
125  9  1125
3 34 3 32
2 24 2 4 12 16 a a2 a2
7. a. 4
  12  b.  6
3 34
33 33 27 b 32
b3 b3

3 3 2 4 3 4  3 2 24
8. a. 5  5 5
6
b. 12  12  12

4 4 2 8
c. x8  x8  x8  x

Check Yourself 4
1. Simplify the expressions.
4 5 3
a. 4
16 b. (–2)4 c. (–3)5 d. 7
–128 e. 27x3

2. Perform the operations.


5 5
a. ñ3  ò12
3 3 3
b. 3 9 c. x 2  x3 d. x  3 x2 y2

3. Simplify the expressions.


3 4
2 625 x5 3
a. b. 3
c. 4
d. 3
8 5 x 3
4. Simplify the expressions.
3 3 4
a. 40 b. 81 c. 4
32 d. x5  y6

5. Perform the operations.


3 3 4 3 6 4 3
a. 2 4 b. x  x3 c. a2  a3 d. 32  16

32 Algebra 8
6. Simplify the expressions.
3 3 4 6
4 6 a3 x4
a. 4
b. c. 5
d. 8
3 2 a2 x3
7. Write each expression in its simplest form.

3 3 3 3 4 3 3
a. 3 b. 5 4
x c. x27 d. x16 e. 3 3

Answers
1
1. a. 2 b. 2 c. –3 d. –2 e. 3x 2. a. 6 b. 3 c. x d. x  3 y2 3. a. b. 5 c. x d. 3 9
2
12
4. a. 2 3 5 b. 3 3 3 c. 2 4 2 d. xy 4 xy2 5. a. 2 6 2 b. x 12 x c. a 6 a d. 4  27

256 9
6. a. 12 b. 6 c. 20 a7 d. 24 x7 7. a. 6 3 b. 20 x c. x d. x2 e. 9 81
27 2

C. RADICAL EQUATIONS
Definition
An equation that has a variable in a radicand is called a radical equation.

For example, the equations ñx = 25, x  1  3, and 2 x – 1  3 x  5 are radical equations.


Let us look at some examples of how to solve radical equations.

EXAMPLE 23 Solve x  1  3.

?
Solution ( x  1)2  32 (take the square of both sides) Check: 8 13
x 1 9 ?
9 3
x8
33
Therefore, 8 is a solution.

EXAMPLE 24 Solve 3x  1  5.

?
Solution ( 3x  1)2  52 Check: 3  8  15
3x  1  25 ?
25  5
3x  24
55
x8
Therefore, 8 is a solution.

Radicals 33
EXAMPLE 25 Solve 2 x  3  3  8.

?
Solution 2x  3  3  8 Check: 2  11  3  3  8
2x  3  8 – 3 ?
25  3  8
2x  3  5 ?

( 2 x  3)2  5 2 5  3 8
2 x  3  25 88
2 x  22 Therefore, 11 is a solution.
x  11

EXAMPLE 26 Solve x2  12  x  6.

?
Solution ( x2  12 )2  ( x  6)2 Check: (–2)2  12  – 2  6
?
x2  12  x2  12 x  36 4  12  4
12 x  –24 ?
16  4
x  –2
44
Therefore, –2 is a solution.

EXAMPLE 27 Solve
3
3x  2  5.

3 ?
Solution ( 3x  2 )3  5 3 Check: 3
3  41  2  5
3x  2  125 3
?
123  2  5
3x  123 ?
3
x  41 125  5
5 5
Therefore, 41 is a solution.

EXAMPLE 28 Solve 4x  1 – 5x – 1  0.

?
Solution 4x  1 – 5x – 1  0 Check: 4  2 – 1 – 5  2 – 1 0
( 4 x  1)2  ( 5 x – 1) 2 ?
9 – 9 0
4x  1  5 x – 1 ?
3 – 3 0
x2
0 0
Therefore, 2 is a solution.

34 Algebra 8
EXAMPLE 29 Solve 3x  1  3x  6  5.

Solution 3x  1  5 – 3 x  6
( 3x  1)2  (5 – 3 x  6 ) 2 (take the square of both sides)

3x  1  25 – 2  5  3 x  6  3 x  6 (simplify)
10  3x  6  30
( 3 x  6 )2  3 2
3x  6  9
3x  3
x1
?
Check: 3  1  1 – 3  1  6 5
?
4  9 5
?
2  35
5 5

Therefore, 1 is a solution.

Check Yourself 5
1. Solve each equation and check your answer.

a. ñx = 5 b. ñx + 1 = 3 c. óx+1 = 6

d. ñx = –3 e. 2x – 1  7 f. 3 x 1 –1  8

5x  1
g. 3x – 5  4  6 h. –30 i. 2x  6 – x
2

j. 4x – 3 – 2 x – 2  0 k. x3  x–2 5 l. x2  1  2 – x

3 3 3 3
m. x 1  3 n. x 2 o. 3x  2  5x

4 4
p. x  2x – 1

Answer
17 1
1. a. 25 b. 4 c. 35 d.  e. 25 f. 8 g. 3 h. i. 2 j. k. 6 l. 3 m. 26 n. 2ñ2 o. 1 p. 1
5 2 4

Radicals 35
EXERCISES 1 .2
1. Write the expressions in exponential form and 5. Solve the equations.
simplify if possible.
a. 2x  1  3 b. 3x – 14  8
a. ò21 2 7 3 2 3
b. 7 c. 5 d. a x
c. ò7x = x
3
d. 4x  6
5 1
e.
3 5
2 f. ( 3)6 g. 3
3 3 3
e. 5x – 4  6 f. 7x  6  5

2. Write the expressions in radical form. g. 5x – 1  3  7 h. 2x – 3 – 2  4


ab 3 1
1 2
3 –3
a. a 2 b. b 3 c. ( ) 2 d. x c
e. ( a )6
2
i. x2  5  x  1 j. x2  9  5
2
3
k. 7x – 6  4 l. 2x – 5  x – 4
3. Simplify the expressions.
6 2
m. x  x  16 – 2 n. 
4 5
x 3
3
a. 3 b. a2 c. 1016
o. ñ6x – x= 5
3 5
d. 5 5 e. 243 f. (–5)4 p. x  1 2 x  x 1– 2 x  8

3 3 3
g. 3
–27 h. 64x6 i. a3  a2 33x – 6 1
q. 
9 2 x1 9
3 4 4 162
j. 3
2 4 k. 3  27 l.
2
4 3 1
3 4 3 r. 8x – 2  6
2 21 3 4x – 3
m. 3
n. o. 3
250 6
7 5
1 1
4
s. –  2
3 4 3 3
p. 3 9 3 q. 2 2 4
2 5
2– x 2 x

4. Perform the operations.


3 6. Simplify the expressions.
3 2 3 6
a. b. 2 3 6
6
108 a. 23  24 b. 53  52

3 3 7 3 10 c. 43  42  4 d. 26  2–4  22
c. 11 – 3 4 – 27 d. 2  2
9 27 e. 3x  3–2x  33x+1 f. 2x  3x  5x

e.
4
17  6 8 f. ( 2 – 6 )( 2  3 ) g. (x – y)2  (y – x)3  (x – y)–4
h. 298 + 298 + 298 + 298
36 Algebra 8
7. Simplify the expressions. 10. Solve the equations for x.

25 53 ax –1 a. 4x+1 = 23x – 4 b. 3x – 4 = 81
a. b. c.
23 54 ax – 2 1 3– x
c. 9x –2  ( ) d. 8x+1 = 42x – 3
a x 1
(–2) –3 5
5 2 4 27
d. e. f. e. (2x – 3)3 = (x + 1)3 f. (2x – 4)6 = x6
b x 1 (–2)–4 104
g. 5  23x – 1 – 23x+1 = 256
12 x  12 x  12 x
h.  81
4x  4x  4x

i. 22 x
3  4x – 3  4 x –1 –  128
4

8. Perform the operations. 3  93 x –1


j.  ((–3)2 ) –4
81  (243) x
a. (–2)5  (–2)3 b. –32  (–3)3  (–3)
(–2 3 )–3  (–2 2 )6
c. (–4)2  (–22)3  (–2–2)4 d.
1 a
(– )3 11. 3a = 25 and 3b = 5. Find .
2 b
e. (–a)7  (–a4)  (–a)–2
12. 3x = 4. Find 92x + 3x+1.
f. (–a–3)–2  (–a2)–3  (–a–1)–1

13. 2x = 3y = a. Write (12)xy in terms of a, x, and y.

14. 2a – 3 and 4b + 7 are integers with 32a – 3 = 54b + 7.


Find a + b.

9. Simplify the expressions.


2

a. 2x – 1 + 2x – 1 + 2x – 1 + 2x – 1 15. (x + 1)x – 16
= 1. Find the sum of the possible
values of x.
b. ax + 2  ax – 3ax
c. 3x+1 + 3x – 1 + 3x+2 – 3x – 2
4
10 x  10 x  10 x  10 x 16. 44 = 16x. Find x.
d.
5x  5x  5x  5x
(2 –7  3–5 )2 313  311 – 39 17. |x + y – 3| + |x – y – 1| = 0. Find x.
e. f.
36 –7 312  310
g. (–1)101  (–1)125  (–1)100  (–1)99  (–1)49
18. Simplify |x – 4| + 2|3 – x| if 3 < x < 4.
3 –4 0 2003
((–39)  (–2) )  (–1)
h.
(–3)125  6 –127  2126

Radicals 37
19. Find the sum of the possible values of x that
22 . x  72  72  72  ... and
satifsfy the equation |(|2x + 4|) – 2| = 4.
y  42 – 42 – 42 – ... . Find x + y.
20. Find the product of the possible values of x that
satisfy the equation |2x + 3| = |5 – 3x|.
23 . 2a  2a  2a  ...  12 – 12 – 12 – ...  3.
21. Simplify the expressions. Find a.

3 4
a. (–3)2 – –8  16
4 3
24 .Simplify 27  11 – 2  30  30  30 ... .
72  48
b.
54
4 4 4
c. 2.89  6.25 – 1.96 25 . x2 – x2 – x2 –...  2. Find x.

2 2
d.  26 .Simplify
5 –1 5 1
1 1 1 1
3 4   ...   .
18  3  29 – 16 2 4 4 6 252  254 254  256
e.
3

f. 12 2  16 2 27. How many different natural number solutions


does the equation ñx + ñy = ó300 have?
5 5
g. 
7– 2 2 7
28. ( 2  1)7  57122  57121. Find (ñ2 – 1)7.
h. 4  12  4 – 12 1
(Hint: ( )  2 1 )
2 –1
( 6  20 )  ( 5 – 1)
i.
2
1 9 16 15
xy 29. 1  9   24 – 16  . Find x  .
j. x y x – y  2 2
if x < y < 0 x x x 8
x–y

k. 2.7  5.4  7.1  2  0.2 30. Evaluate (50  51  52  53)  1.

4 4
l. 15 – 6  15  6 – 3
31. 3 3
x  2 – x – 2  4. Find x2.
m. 4 7  4– 7 (Hint: (a – b)3 = a3 – 3ab(a – b) – b3)

38 Algebra 8
CHAPTER REVIEW TEST 1A
1. Simplify (|(–(–2)3) – 2|) – [3  |–|(–3)2 – 3||]. 6. |x – 2| = |x + 3|. Find the value of x which sat-
isfies the equation.
A) –8 B) –12 C) 8 D) 28
1 1 3
A) – B) – C) 1 D)
4 2 2

2. Calculate |1 – ñ2| + |1 + ñ2|. 7. a is a positive real number. Which one of the


A) 0 B) ñ2 C) 2 D) 2ñ2 following numbers is negative?

A) a–1 B) a–2 C) –a–3 D) –(–a)3

3. How many elements are there in the solution set 1


–1

of |x – 2| + 1 = 0? 8. Simplify (– )3  .
 2 
A) 0 B) 1 C) 2 D) 3 1 1
A) – B) C) –8 D) 8
8 8

4. –1 < x < 3. Calculate |x + 1| + |x – 3|. 9. Simplify 4x+1  8x – 1  16x.

A) 2x B) 2x – 2 C) 2 D) 4 A) 23x B) 24x C) 29x – 1 D) 28x+1

4m  4m
10. Simplify .
5. ||x + 1| – 5| = 3. Find the sum of the possible 2m  2m  2m  2m
values of x.
2 m 1

A) –3 B) –2 C) 2 D) 4 A) 22m B) 2m +1 C) 2m – 1 D) 2 m  2

Chapter Review Test 1A 39


3x 1  3x  2  3x  3 16. Evaluate (3ñ2 – ò12)  (ò18 + 2ñ3).
11.  27. Find x.
39
A) 36 B) 180 C) ñ6 D) 6
A) 1 B) 3 C) 13 D) 27

1 17. Evaluate 4 – 12.


12. 2 a – b  2 –  0. Find a.
2 2 a  b 1
A) –ñ3 B) ñ3 C) ñ3 – 1 D) ñ3 + 1
A) 0 B) –1 C) –3 D) 3

13. a = 2x, b = 3x, and c = 5x. Express 90x in terms 18. Evaluate 2  2 4  12 .
of a, b, and c.
A) 1 B) ñ3 C) ñ3 – 1 D) ñ3 + 1
2 2 2 2 2 4 2
A) (abc) B) a bc C) ab c D) a b c

27 m3  9 m2
19. Evaluate .
14. Evaluate 3ñ8 – 2ñ2 + ò32. 12 m3  4 m2
A) ñ6 B) 8ñ2 C) ò38 D) 26ñ2 3 9
A) B) ñ2 C) D) 3m+1
2 4

15. Evaluate 2.25 – 2.89  1.44. 5 5


20. Evaluate  .
7– 2 7 2
1 2
A) –1 B) C) 1 D)
2 3 A) 7 B) 2ñ7 C) 5ñ7 D) 5

40 Algebra 8
CHAPTER REVIEW TEST 1B
(–1)55  (–172 ) – (–1) 64 569 – 568
1. Evaluate . 6. Evaluate .
(–121 ) – (–1)82 – (–1)15 568  567

3 1 5 5 10
A) – B) –3 C) 1 D) –1 A) B) C) D)
2 6 6 3 3

2 –2  4–1
2. Evaluate . 7. 3x = 5, 2y = 3, and 5z = 0.125. Find x  y  z.
2 –2 – 3–1
A) 3 B) 10 C) –3 D) 30
1 –1
A) 3 B) C) 2 D) –6
3

8. 3x+1 + 2  3x – 1 = 33. Find x.


3
x(2 ) A) 1 B) 2 C) 3 D) 4
3.  16. Find |x|.
( x2 )3

A) 2 B) 4 C) 8 D) 16

3 2
9. a = (52)3, b = 5(2 ), and c = 5(3 ). Which statement
4. What is half of 220? is true?

1 A) a > b > c B) a > c > b


A) 210 B) 221 C) 219 D)
210 C) b > c > a D) c > b > a

3x  3x  3x
5. Find the simplest form of . 10. k = 1254  642. How many digits are there in the
9x  9x  9x
number k?
1
A) 3x B) C) 32x D) 3x+1
3x A) 10 B) 11 C) 12 D) 13

Chapter Review Test 1B 41


32 x  243x 1 1
11.  9x  3. Find x. 16. Evaluate  .
81x1 2 2 2 –2

2 A) 2ñ2 B) –2 C) –ñ2 D) 4
A) B) 2 C) 9 D) 10
9

11  2 30 – 8  2 15
17. Evaluate .
9 3–2 2
12. Evaluate 1 1 .
16
A) 3  15 B) ñ6 – ñ3
41 41 3 2
A) B) C) D)
16 4 2 3 C) ñ3 D) 3 2

12  48 – 27 3 5 – 3– 5
13. Simplify . 18. Evaluate .
75 – 2 3 2

A) 2ñ3 B) –1 C) 1 D) 3ñ3 A) ñ5 B) 2ñ5 C) 2 D) 1

4
14. Evaluate 3
13  13 – 64 . 19. Evaluate
3 3
2 4 2 4 ... .

A) 1 B) 2 C) 3 D) 4
A) ñ8 D) ñ8
6 3
B) 2 C) 8

4 3
15. x x2 x  xn . Find n. 5
8 x 1
20. 3
7
 2. Find x.
42 x  3
11 4 11 1
A) B) C) D)
24 7 7 6 A) 9 B) 105 C) 114 D) 120

42 Algebra 8
Objectives
After studying this section you will be able to:
1. Define statistics as a branch of mathematics and state the activities it involves.
2. Describe some different methods of collecting data.
3. Present and interpret data by using graphs.
4. Describe and find four measures of central tendency: mean, median, mode, and range.

A. BASIC CONCEPTS
1. What is Statistics?
Statistics is the science of collecting, organizing, summarizing and analyzing data, and draw-
ing conclusions from this data. In every field, from the humanities to the physical sciences,
research information and the ways in which it is collected and measured can be inaccurate.
Statistics is the discipline that evaluates the reliability of numerical information, called data.
We use statistics to describe what is happening, and to make projections concerning what will
happen in the future. Statistics show the results of our experience.
Many different people such as economists, engineers, geographers, biologists, physicists,
meteorologists and managers use statistics in their work.

Definition statistics
Statistics is a branch of mathematics which deals with the collection, analysis, interpretation,
and representation of masses of numerical data.
The word statistics comes from the Latin word statisticus, meaning ‘of the state’.

The steps of statistical analysis involve collecting information, evaluating it, and drawing con-
clusions.
For example, the information might be about:

 what teenagers prefer to eat for breakfast;

 the population of a city over a certain period;

 the quality of drinking water in different countries of the world;

 the number of items produced in a factory.

44 Algebra 8
The study of statistics can be divided into two main areas: descriptive statistics and inferential
statistics.
Descriptive statistics involves collecting, organizing, summarizing, and presenting data.
Inferential statistics involves drawing conclusions or predicting results based on the data
collected.

2. Collecting Data
We can collect data in many different ways.

a. Questionnaires
A questionnaire is a list of questions about a given topic. It is usually printed on a piece of
paper so that the answers can be recorded.
For example, suppose you want to find out about the television viewing habits of teachers.
You could prepare a list of questions such as:

 Do you watch television every day?


 Do you watch television: in the morning?
in the evening?

 What is your favourite television program?


 etc.
Some questions will have a yes or no answer. Other questions might ask a person to choose
an answer from a list, or to give a free answer.

When you are writing a questionnaire, keep the following points in mind:

1. A questionnaire should not be too long.


2. It should contain all the questions needed to cover the subject you are studying.
3. The questions should be easy to understand.
4. Most questions should only require a ‘Yes/No’ answer, a tick in a box or a circle round a
choice.

In the example of a study about teachers’ television viewing habits, we only need to ask the
questions to teachers. Teachers form the population for our study. A more precise population
could be all the teachers in your country, or all the teachers in your school.

Statistics and Graphics 45


b. Sampling
A sample is a group of subjects selected from a population.
population
Suppose the population for our study about television is all
the teachers in a particular city. Obviously it will be very
Xdifficult to interview every teacher in the city individually. sample
Instead we could choose a smaller group of teachers to
interview, for example, five teachers from each school.
A sample is a subset of a population.
These teachers will be the sample for our study. We could
say that the habits of the teachers in this sample are
probably the same as the habits of all the teachers in the city.

The process of choosing a sample from a population is called sampling.


The process of choosing a sample from a population is called sampling.
When we sample a population, we need to make sure that the sample is an accurate one. For
example, if we are choosing five teachers from each school to represent all the teachers in a
city, we will need to make sure that the sample includes teachers of different ages in different
parts of the city. When we have chosen an accurate sample for our study, we can collect the
data we need and apply statistical methods to make statements about the whole population.

c. Surveys
One of the most common method of collecting data is the use of surveys. Surveys can be car-
ried out using a variety of methods. Three of the most common methods are the telephone
survey, the mailed questionnaire, and the personal interview.

3. Summarizing Data
In order to describe a situation, draw conclusions, or make predictions about events, a
researcher must organize the data in a meaningful way. One convenient way of organizing the
data is by using a frequency distribution table.
A frequency distribution table consists of two rows or columns. One row or column shows the
data values (x) and the other shows the frequency of each value (f). The frequency of a value
is the number of times it occurs in the data set.
For example, imagine that 25 students took a math test and received the following marks.
8 7 9 3 5
10 8 10 6 8
7 7 6 5 9
4 5 9 6 4
9 3 8 8 6

46 Algebra 8
The following table shows the frequency distribution of these marks. It is a frequency distri-
bution table.

mark (x) 1 2 3 4 5 6 7 8 9 10

frequency (f) 0 0 2 2 3 4 3 5 4 2

Note
The sample size is the number of elements in a sample. It is denoted by n.

We can see from the table that the frequency of 7 is 3 and the frequency of 8 is 5.
The sum of the frequencies is equal to the total number of marks (25).
The number of students took test is called the sample size (n). In this example the sample
size is 25.
The sum of the frequencies and the sample size are the same.

EXAMPLE 1 Twenty-five students were given a blood test to determine their blood type. The data set was
as follows:
A B AB B AB
A O O AB A
B O O O B
AB A O B O
O B AB B O
Construct a frequency distribution table of the data and find the percentage of each blood
type.

Solution There are four blood types: A, B, O, and AB. These types will be used as the classes for the
distribution. The frequency distribution table is:

class frequency percent


We can use the following formula to find the
percentage of values in each class:
A 4 16 % f
%   100% where
B 7 28 % n
f = frequency, and
O 9 36 %
n = total number of values (25).
AB 5 20 %
For example, in the class for type A blood, the
Total 25 Total 100 percentage is
4
 100%  16%.
25

Statistics and Graphics 47


B. PRESENTING AND INTERPRETING DATA
When we have collected, recorded and summarized our data, we have to present it in a form
A graph is a diagram
that people can easily understand.
that relates two or more
Graphs are an easy way of displaying data. There are three kinds of graph: a line graph, a bar
different types of
information. graph, and a circle graph (also called a pie chart).

1. Bar Graph
The most common type of graph is the bar graph (also called a histogram). A bar graph uses
rectangular bars to represent data. The length of each bar in the graph shows the frequency
or size of a cooresponding data value.

EXAMPLE 2 The following table shows the marks that a student Subject Mark
received at the end of the year in different school subjects.
Maths 9
Draw a vertical bar graph for the data in table.
Physics 7

Chemistry 7

Biology 8

Computer 10

History 5

Music 6

Solution We begin by drawing a vertical scale 10


to show the marks and a horizontal 9
8
scale to show the subjects. 7
Then we can draw bars to show the 6
5
Marks

marks for each subject. 4


3
2
1
0
History
Mathematics

Chemistry

Music
Computer
Physics

Biology

Lessons

48 Algebra 8
2. Line Graph
We can make a line graph (also called a broken-line graph) by drawing line segments to join
the tops of the bars in a bar graph.
For example, look at the line graph of the data from Example 5.2.
10 10
9 9
8 8
7 7
6 6
5 5
Marks

Marks
4 4
3 3
2 2
1 1
0 0

History

History
Mathematics

Mathematics
Chemistry

Music

Chemistry

Music
Computer

Computer
Physics

Physics
Biology

Biology
Lessons Lessons

To draw the line graph, we mark the middle point of the top of each bar and join up the points
with straight lines.

EXAMPLE 3 The following table shows the number of cars produced


Car Production
by a Turkish car company between 1992 and 2000.
Draw a bar graph and a line graph of the data in this Year Production
table. 1992 110 659
1993 133 006
1994 99 326
1995 74 862
1996 65 007
1997 91 326
1998 88 506
1999 125 026
2000 140 159

Statistics and Graphics 49


160
Solution First we need to choose the axes.
140
Let us put the years along the

Number of cars (10 000)


120
horizontal axis and the production 100
along the vertical axis of the graph. 80
It will be difficult to show large 60
40
numbers such as 133 006 on the
20
vertical axis. Instead, we can 0
choose a different unit for the

2000
1999
1992

1993

1994

1995

1996

1997

1998
vertical axis, for example: one unit
on the axis means 10 000 cars. We Year

write this information when we 160


140
label the axis.

Number of cars (10 000)


120
100
80
60
40
20
0

2000
1999
1992

1993

1994

1995

1996

1997

1998
Year

EXAMPLE 4 The gross domestic product (GDP) of a country is


8
Share of education expenditures as a percentage of GDP in
selected countries *
the total value of new goods and services that the 7%
7 6.4%
country produces in a given year. 6 6% 6%
5.3%
5.2%
5
The graph below shows the amount of money 4
3.4%
that seven different countries spend on education 3
2
in 2003, as a percentage of each country’s gross
1
domestic product. Look at the graph and answer 0

the questions.

Australia
Turkey

USA

Norway

Germany
United
Kingdom

Canada
a. Which country spent the largest percentage of
its GDP on education? * Source: Education at a Glance 2003 - OECD indicators

b. Which country spent the smallest percentage of


its GDP on education?
c. Find the percentage difference between the
countries which spent the largest and smallest
percentage of their GDP on education.
d. Which countries spent the same percentage of
their GDP on education?

50 Algebra 8
Solution a. The USA spent the largest percentage (7% of its GDP).
b. Turkey spent the smallest percentage (3.4% of its GDP).
c. 7 – 3.4 = 3.6%
d. Norway and Australia spent the same percentage: both countries gave 6% of their GDP.

Practice Problems
1. The bar graph below compares different causes of death in the United States for the year
1999. Look at the graph and answer the questions.

Comparative causes of annual deaths in the United States (1999)*

Cause

* Source: World Health Organization

a. What was the most common cause of death?


b. What was the least common cause of death?
c. What is the ratio of the number of deaths caused by smoking to the number of deaths
caused by alcohol?
d. How many deaths are shown in the graph?
e. On avarage, how many people died per day from each canse in 1999? (Hint: There
were 365 days in 1999.)
Answers
16
1. a. Smoking b. Drug Induced c. d. 632000 e. 1732
3

Statistics and Graphics 51


EXERCISES 2 .1
1. The set of quiz scores in a class is as follows. 4. The following table shows the amount of sea fish
8 5 6 10 4 7 2 7 6 3 1 7 caught in Turkey in 2003.
Fish Quantity (1000 tons)
5 9 2 6 5 4 6 6 8 4 10 8
Anchovy 416
Construct a frequency distribution table for this Horse Mackerel 295
data. Scad 16
Gray mullet 12
Blue fish 11
Pilchard 11
Whiting 12
Hake 8
Other 32
Source: Turkey’s Statistical Yearbook 2004
2. A student’s expenses can be categorized as shown
Present this information in a circle graph.
in the table.
Expenses Percent of total income.
Food 30%
Rent 27%
Entertainment 13%
Clothing 10%
Books 15%
Other 5%

Present this information in a bar graph.


5. The following bar graph shows the hazelnut
production in Turkey from 1999 to 2003. Use the
graph to answer the questions.
Hazelnut production in Turkey (tons)
700 000
600 000
500 000
3. The following table shows the favorite sport 400 000
300 000
chosen by each of forty students in a class.
200 000
Sport Number of class members 100 000
1999

2000

2001

2002

Football 8 2003
Basketball 5
Volleyball 7 a. Estimate the total production for all five years.
Swiming 12
b. Which year had the highest production?
Wrestling 3
Karate 2 c. Find the combined production for 2002 and
Judo 4 2003.
Present this information in a circle graph. d. Draw a broken line graph of the data.

52 Algebra 8
Definition
An equation that can be written in the form
ax2 + bx + c = 0, a0
is called a quadratic equation.

In the equation, a, b, and c are real number coefficients and x is a variable. A quadratic equa-
I’m sick of being tion written in the form ax2 + bx + c = 0 is said to be in standard form. Sometimes, a quad-
an unknown
ratic equation is also called a second degree equation.
For example,
1
x2 + 3x – 5 = 0, 2x2 – x – 1 = 0 and ñ2x2 –
x+3=0
2
are all quadratic equations. By the definition of a quadratic equation, a cannot be zero.
However b or c or both may be zero. For instance,
3x2 + 5x = 0, 2x2 = 0 and x2 – 9 = 0
are also quadratic equations.
We can see that quadratic equations are formed by second-degree polynomials. Polynomials
of a different degree do not form quadratic equations.
Let us look at the coefficients a, b, and c of some quadratic equations.
Equation a b c

3x2 + 5x – 9 = 0 3 5 –9

1 – x + 3x2 = 0 3 –1 1

ñ2x2 + 5x = 0 ñ2 5 0
1 1
– x2 + =0 –1 0
2 2
1 – x2 = 0 –1 0 1

(ñ3 + 1)x2 = 0 ñ3 + 1 0 0

x2 x 1 1 1 1
– + =0 –
2 3 4 2 3 4

EXAMPLE 1 Determine whether the following equations are quadratic or not.


1 2
a. x2 + 1 = 0 b. x – 2 x +5 = 0 c. 2x2 – 3x = 5
2
d. x2 – 2x–1 + 3 = 0 e. (x – 1)(x + 2) = 0 f. (x – 2)x2 = 0

54 Algebra 8
Solution a, b, c, and e are quadratic equations. Equation d is not quadratic, since the power of x is –1,
which does not meet the requirements for a quadratic. Equation f is a third degree equation,
so it is not quadratic.
To the best of our
knowledge, the origin of
the term ‘quadratic’ is
Latin. It is derived from To solve a quadratic equation we must find the values of the unknown x which make the
quadratus which is the
past participle of quadrare
left-hand and right-hand sides equal. Such values are called the solutions or roots of the
which means ‘to make quadratic equation. A number of techniques are available to help us obtain a solution to any
square’. From this it is
clear that part of the quadratic equation.
word is connected to the
Latin word for ‘four’: it
refers to squaring, and
a square is a regular
four-sided figure.

A. SOLVING EQUATIONS OF THE FORM ax2 = 0


We have seen that the solutions (or roots) of a quadratic equation are the values of x that make
the two sides of the equation equal. Let us find the roots of the simple quadratic ax2 = 0.
ax2 = 0 (a  0)
x2 = 0
xx=0
x = 0 or x = 0
x1 = x2 = 0
If A  B = 0, then We can see that this equation has two equal roots. When the roots of a quadratic equation
A = 0 or B = 0. are the same, we say that the equation has a double root.

EXAMPLE 2 3
Solve the equation  x2  0.
2

3
Solution  x2  0
2
x2  0
x1  x2  0

Quadratic Equations 55
B. SOLVING EQUATIONS OF THE FORM ax2 + bx = 0
Let us look at the solution of this more complex quadratic.
ax2 + bx = 0
x(ax + b) = 0
x = 0 or ax + b = 0, so
b
x1 = 0 and x2 = – .
a
This kind of quadratic equation has two roots and one of them is always zero.

EXAMPLE 3 Solve the equations.


a. x2 + x = 0 b. –4x2 + 5x = 0 c. ñ3x2 – 2x = 0

Solution a. x2 + x = 0 b. –4x2 + 5x = 0 c. ñ3x2 – 2x = 0


x(x + 1) = 0 x(–4x + 5) = 0 x(ñ3x – 2) = 0
x = 0 or x + 1 = 0 x = 0 or –4x + 5 = 0 x = 0 or ñ3x – 2 = 0
5 2 2 3
x1 = 0 or x2 = –1 x1 = 0 or x2 = x1 = 0 or x2 = =
4 3 3

C. SOLVING EQUATIONS OF THE FORM ax2 + c = 0


Look at the calculation.
ax2 + c = 0, c0
2
ax  c
c
x2  
a
c
x 
a
c
Here the sign of is important.
a
c
If > 0, the equation has no real solution, because we cannot find the square root of a
a
negative number.
c
If < 0, the equation has two real solutions. These roots are symmetric, i.e. they are the
a
same numeral with opposite signs.

Note
All positive real numbers have two square roots. One root is the positive square root and the
other root is the negative square root, i.e. if a2 = b and a is a positive real number, then a = ñb.

56 Algebra 8
EXAMPLE 4 Solve the equations.

a. 3x2 – 27 = 0 b. 2x2 + 6 = 0 c. 7 – 4x2 = 2

Solution a. 3x2 – 27 = 0 b. 2x2 + 6 = 0 c. 7 – 4x2 = 2


3x2 = 27 2x2 = –6 4x2 = 5
x2 = 9 x2 = –3 5
x2 =
x = 3 no real solution 4
(x2 cannot be negative) 5 5
x= =
4 2
Check Yourself 1
Solve the equations.
1. –3x2 = 0 2. 5x2 – 20x = 0 3. 7x2 + 35 = 0 4. 2x2 – 8 = 0
Answers
1. 0 2. 0, 4 3. no real solution 4. 2

FIND THE MISTAKE!

Let a and b be two arbitrary numbers such that a  b. Then


(a – b)2 = a2 – 2ab + b2 = b2 – 2ab + a2
(a – b)2 = (b – a)2
a–b=b–a
2a = 2b
a = b.
Can you find the mistake in this working?

D. SOLVING EQUATIONS OF THE FORM ax2 + bx + c = 0


There are three basic methods for solving a quadratic equation of the form ax2 + bx + c = 0:
1. factoring,
2. completing the square, and
3. using the quadratic formula.

1. Factoring
If we can write ax2 + bx + c = 0 as the product of two linear factors, then we can easily solve
the equation.
To solve a quadratic equation by factoring, follow the steps.
1. Write the equation in standard form, ax2 + bx + c = 0, a  0.
2. Factor the left side of the equation.

Quadratic Equations 57
3. Apply the zero product property, that is, set each factor equal to zero.
4. Solve each equation to obtain the roots.

EXAMPLE 5 Solve by factoring.

a. x2 + 3x + 2 = 0 b. 6x2 – 19x – 7 = 0 c. 2x2 = x + 3

Solution a. x2 + 3x + 2 = 0 b. 6x2 – 19x – 7 = 0


(x + 2)(x + 1) = 0 (2x – 7)(3x + 1) = 0
x + 2 = 0 or x + 1 = 0 2x – 7 = 0 or 3x + 1 = 0
x1 = –2, x2 = –1 7 1
x1 = , x2 = –
2 3
c. 2x2 = x + 3
2x2 – x – 3 = 0
(x + 1)(2x – 3) = 0
x + 1 = 0 or 2x – 3 = 0
3
x1 = –1, x2 =
2

Note
When you are solving an equation, do not divide both sides by an expression containing
the variable for which you are solving. You may be dividing by zero. For example, to solve
x2 – 2x = 0, do not divide both sides by x, because x may be zero and you will also lose one
of the solutions.

Check Yourself 2
Solve the following equations.
1. 3x2 = 5x + 2 2. (5x – 1)(x + 2) = x + 2 3. 4x(x + 1) = 3
Answers
1 2 3 1
1.  , 2 2. 2 , 3.  ,
3 5 2 2

2. Completing the Square


The idea behind this method is to adjust the left side of the equation ax2 + bx + c = 0 so that it
becomes a perfect square, that is, the square of a first-degree polynomial. Expressions in the
form x2 + 2xy + y2 and x2 – 2xy + y2 are perfect square polynomials.
Numbers whose square For example, x2 + 6x + 9 and x2 – 4x + 4 are perfect squares, because
roots are integers or
quotients of integers x2 + 6x + 9 = (x + 3)2 and x2 – 4x + 4 = (x – 2)2.
are perfect squares.
To write a quadratic equation as a perfect square, follow the steps,

58 Algebra 8
1. Make sure a = 1 in the quadratic. If it isn’t 1, x+2
divide each term by a. x 1 1
2. Rewrite the equation so that the constant term is
alone on one side of the equation.
3. Take half of the coefficient of the x term and square x
x+2
it.
4. Add this number to both sides of the equation.
1
5. Factor the left-hand side into a perfect square. 1
2 2
6. Solve for x by using the square root property.

Let us look at some examples of completing the square. x2 + 4x


4
(x + 2)2 = x2 + 4x + 4

Start
_____________ Add
_____________ Result
___________________________
x2 + 4x 4 x2 + 4x + 4 = (x + 2)2
x2 + 12x 36 x2 + 12x + 36 = (x + 6)2
x2 – 6x 9 x2 – 6x + 9 = (x – 3)2
1 1 1
x2 + x x2 + x + = (x + )2
4 4 2

Note
The expression x2 + 2bx is equivalent to (x + b)2 – b2.

EXAMPLE 6 Solve by completing the square.

a. x2 + 6x – 7 = 0 b. 2x2 – 4x + 1 = 0

Solution a. x2 + 6x – 7 = 0 b. 2 x2 – 4x+ 1= 0
x2 + 6x = 7 1
x2 – 2 x+ = 0
2
x2 + 6x + 9 = 7 + 9
1
x2 – 2 x = –
(x + 3)2 = 16 2
x + 3 = ò16 1
x2 – 2 x+ 1= – + 1
2
x1 = –7, x2 = 1
1
(x – 1)2 =
2
1
x – 1= 
2
2 2
x1 =1 – , x2 =1+
2 2

Quadratic Equations 59
Check Yourself 3
Solve the equations.
1. x2 + 8x – 3 = 0 2. 2x2 – 5x – 3 = 0 3. 2x2 – 2 = 4x
Answers
1
1. –4  ò19 2. 3,  3. 1  ñ2
2

3. The Quadratic Formula


The final method will work on any quadratic equation. Therefore, we can use it when the other
easier methods fail or are not easy to apply. Look at the derivation of the quadratic formula.
ax2 + bx+ c = 0 (a  0, a, b, c  )

b c
x2 + x+ = 0 (divide both sides by a)
a a
b c
x2 + x = – (try to get a perfect square)
a a
b b c b b 2
x2 + x+( )2 = – +( )2 (add ( ) to both sides)
a 2a a 2a 2a
b 2 4ac+ b2
(x+ ) =
2a 4a2

b b 2  4ac b 2  4ac
x+ = = 
2a 4a2 2a

–b b2  4ac b  b 2  4ac
x=  = ,
2a 4a2 2a

b+ b 2  4ac b – b 2  4ac


x1  , x2 
2a 2a

QUADRATIC FORMULA

– b  b 2 – 4 ac
If ax2 + bx + c = 0, a  0, then x =
2a

EXAMPLE 7 Solve x2 + 2x – 8 = 0 using the quadratic formula.

60 Algebra 8
Solution First we identify the coefficients a, b, and c. For this equation a = 1, b = 2, and c = –8.
Let us substitute the values of a, b, and c into the quadratic formula.
2
Make sure that you write b  b 2  4ac 2  2  4  1  (–8) 2  36
a quadratic equation in x= = =
standard form before 2a 2 1 2
you identify the values
2  6 4 2  6 –8
a, b, and c. x1 = = = 2, x2 = = = –4
2 2 2 2

EXAMPLE 8 Solve 3x2 + 2x – 4 = 0.

Solution For this equation a = 3, b = 2, and c = –4.


2
 b  b2  4ac 2  2  4  3  (–4) 2  52
x= = =
2a 23 6
2 + 52 2 + 2 13 1+ 13
x1 = = =
6 6 3
2  52 2  2 13 1  13
x2 = = =
6 6 3

EXAMPLE 9 Find the real solutions of the equation 9 +


3 2

x x2
= 0 , x  0.

3 2
Solution In its present form, the equation 9 +  = 0 , is not a quadratic equation. However,
x x2
we can make it quadratic by multiplying each side by x2, since x  0. The result is
9x2 + 3x – 2 = 0, x  0. Now, a = 9, b = 3, and c = –2.
b2  4ac = 32  4  9  (–2)  81

3  81 12 2
x1 = = =
29 18 3
3 + 81 6 1
x2 = = =
18 18 3

Check Yourself 4
Solve the equations.
5
1. 4x2 + 3x – 1 = 0 2. x2  = 3 x 3. 2x2 – 4x = 5
2
Answers
1 3  19 2  14
1. 1, 2. 3.
4 2 2

Quadratic Equations 61
4. Discriminant of a Quadratic Equation
Definition discriminant of a quadratic equation
The quantity b2 – 4ac is called the discriminant of a quadratic equation.
The discriminant tells us whether the equation has real solutions, and also tells us how many
roots of an equation exist. The discriminant is denoted by  (delta).

b  
x= ,  = b2  4ac
2a

For a quadratic equation ax2 + bx + c = 0, the value of  determines the number of real roots.
1. If  > 0, there are two distinct real roots.
2. If  = 0, there is one real root (a double root).
3. If  < 0, there is no real root.

Use the discriminant to check the number of roots before you solve a quadratic equation.

EXAMPLE 10 Solve x2 + 6x + 7 = 0.

Solution First, find .


 = b2 – 4ac = 36 – 4  7  1 = 8
So  is positive.

6 + 8 6  8
x1 = = 3 + 2, x2 = = 3  2
2 2

We can see that  > 0 and there are two real roots.

EXAMPLE 11 Solve x2 – 4x + 4 = 0.

Solution First, check .


 = b2 – 4ac = 16 – 4  4  1 = 0
4 0 4+ 0
So  = 0 and x1 = , x2 = .
2 2
Hence x1 = x2 = 2.
We can see that = 0 and there is only one real root (a double root).

EXAMPLE 12 Solve x2 – 2x + 5 = 0.

Solution  = b2 – 4ac = 4 – 4  1  5 = –16


 is negative, so there is no real root.

62 Algebra 8
EXAMPLE 13 For which values of k does the equation 3x2 – 4x + k = 0 have no real solution?

Solution If there is no real solution, the discriminant must be negative. Therefore,


 = (–4)2  4  3  k = 16 12 k < 0 .
16  12 k < 0, 16 < 12 k,

16 4
< k, k> .
12 3
4
So the equation has no real solution for k > .
3

EXAMPLE 14 For what values of m does the equation x2 + 3mx – 5m – 1 = 0 have a double root?

Solution A quadratic equation has a double root if its discriminant is 0. Therefore,


 = (3m)2 – 4  1  (–5m – 1) = 0.
9m2 + 20m + 4 = 0
(9m + 2)(m + 2) = 0
9m + 2 = 0 or m + 2 = 0.
2
So m1 = and m2 = 2 .
9

EXAMPLE 15 The equation mx2 + (2m + 1)x + m – 1 = 0 has two real roots. Find m.

Solution If the quadratic equation has two real roots, then its discriminant is positive.
 = b2  4ac = (2m+ 1)2  4m(m 1)

= 4m2 + 4m+ 1  4m 2 + 4m
= 8 m+ 1

 = 8 m+ 1 > 0

8 m > 1
–1
m>
8
–1
Therefore, the equation has two real roots if m > .
8

Quadratic Equations 63
EXAMPLE 16 Prove that (a2 + b2)x2 + 2(a + b)x + 2 = 0 has no real root if a and b are unequal.

Solution  = 4(a + b)2 – 4(a2 + b2)  2

= 4(a2 + 2ab + b2) – 8(a2 + b2)

= 4a2 + 8ab + 4b2 – 8a2 – 8b2

= –4a2 + 8ab – 4b2

= –4(a – b)2

But (a – b)2 is always non-negative, and it is also non-zero, since a  b. So  < 0.

Thus the equation (a2 + b2)x2 + 2(a + b)x + 2 = 0 has no real roots if a and b are unequal.

EXAMPLE 17 From each corner of a square piece of sheet metal, a man removes a square of side 3 cm. He
turns up the edges to form an open box. If the box holds 48 cm3, what are the dimensions of
the piece of sheet metal?

Solution Let x be the length of a side of the square.

3 x–6 3

3 3

x–6 x–6

3 3

3 x–6 3

Since the volume of the box is 48 cm3, we have


3(x – 6)2 = 48
(x – 6)2 = 16
x – 6 = 4
x1 = 10, x2 = 2.
We discard the solution x = 2 since length cannot be negative. So the sheet of metal is
10 cm by 10 cm.

64 Algebra 8
EXAMPLE 18 A ladder 5 m long is leaning against a house.
The distance from the bottom of the ladder to
the house is 1 m less than the distance from
the top of the ladder to the ground. How far is
the bottom of the ladder from the house?

Solution Let us use x to represent the distance from the top of the ladder to the ground. The ladder
then forms the right triangle shown in the diagram. By using the Pythagorean Theorem, we
get the equation
x2 + (x – 1)2 = 52
x2 + x2 – 2x + 1 = 25
5
2x2 – 2x – 24 = 0 x

x2 – x – 12 = 0
(x + 3)(x – 4) = 0
x–1
x = –3 or x = 4.
Since the length cannot be negative, x = 4. So the distance from the bottom of the ladder to
the house is 3 m.

EXAMPLE 19 A motorboat heads upstream a distance of 48 km on a river whose current is running at 3 km per
hour (km/h). Then the motorboat returns. The trip upstream and back takes 12 hours. Assuming
that the motorboat maintained a constant speed relative to the water, what was its speed?

Solution We use x to represent the constant speed of the motorboat relative to the water. Then
the true speed going upstream is x – 3 km/h, and the true speed going downstream is
x + 3 km/h. Since Distance = Velocity  Time, we can write Time = Distance / Velocity.
48
Therefore, the boat takes hours to 48 km
x–3
48
travel upstream and hours to travel
x+ 3
downstream.
x – 3 km/h
Since the total time is 12 hours,
48 48
+ = 12; x  3.
x – 3 x+3 x + 3 km/h

Multiply both sides by (x – 3)(x + 3):

Quadratic Equations 65
48(x – 3) + 48(x + 3) = 12(x – 3)(x + 3)
x2 – 8x – 9 = 0
(x – 9)(x + 1) = 0
x = 9 or x = –1.
Since the speed cannot be negative, the speed of the boat is 9 km/h.

Check Yourself 5
1. Evaluate the discriminant of the equation 2x2 – x – 3 = 0, and describe the roots.
2. The sum of two numbers is 10 and sum of their squares is 68. Find the numbers.
3. For which value(s) of m does the equation mx2 + mx – 1 = 0 have a double root?
Answers
1.  = 25, two real roots 2. 2, 8 3. –4

M AT H F U N
A mathematician, a physicist, and an engineer were traveling through Scotland when they
saw a black sheep through the window of the train.
‘Aha,’ said the engineer, ‘I see that Scottish sheep are black.’
‘Hmm,’ said the physicist, ‘You mean that some Scottish sheep are black. We
haven’t seen all the sheep yet.’
‘No,’ said the mathematician, ‘All we know is that there is at least one sheep
in Scotland, and that at least one side of that one sheep
is black! We haven’t seen the other
side of the sheep yet.’

66 Algebra 8
T HE G OLDEN R ATIO
The Golden Ratio appears again and again in art, architecture, music and nature. Its origins go back to the days
of the ancient Greeks, who thought that a rectangle with sides in the Golden Ratio, called a golden rectangle,
exhibited the most aesthetically pleasing proportion. The use of the Golden Ratio has been of interest to artists
and architects since before the building of the Parthenon in Greece in the fifth century B.C. The rectangle drawn
around the Parthenon with its upper triangular structure intact, as shown in Figure 1, is a golden rectangle.

1 x–1

1 P 1 Q

x
Figure 1 Figure 2

Figure 2 shows the dimensions of a golden rectangle. The ratio of length to width
in the rectangle is x to 1. A square P with sides of 1 unit has been marked, leaving
a smaller rectangle Q. For the smaller rectangle Q, the ratio of length to width is 1
to x – 1. In order for the larger rectangle to be a golden rectangle, the two ratios
need to be equal, creating a proportion:
x 1
=
1 x–1
When we apply cross multiplication to this proportion, we obtain a quadratic
equation:
x 1
  x(x –1) = 1 ; x2 – x = 1 ; x2 – x – 1 = 0.
1 x 1
Since we cannot factor this equation, we apply the quadratic formula.
1 5 1 5
The solutions are x1  and x2  .
2 2
Because x represents the length of a rectangle, the negative solution is discarded
1 5
and the positive solution is  1.618034 . This is the Golden Ratio.
2
The Golden Ratio occurs in nature as well as in art. For example, in sunflowers,
the ratio of the number of clockwise spirals to the number of counterclockwise
spirals approximates the Golden Ratio.
EXERCISES 3 .1
A. Solving Equations of the Form ax2 = 0 D.Solving Equations of the Form
1. Solve the equations. ax2 + bx + c = 0
9. Solve by factoring.
a. ñ2x2 = 0 b. (ñ3 – 2)x2 = 0
a. x2 – x = 0 b. –3x2 + x = 0
1 2
c.  x2 = 0 d. 0.07x = 0
5 c. x2 – 49 = 0 d. x2 – 25 = 0
e. x2 – x – 2 = 0 f. x2 + 3x + 2 = 0
B. Solving Equations of the Form g. x2 – 2x + 1 = 0 h. 6x2 + x – 15 = 0
ax2 + bx = 0 i. 10x2 – 19x + 6 = 0 j. 12x2 – 5x – 2 = 0
2. Solve the equations. 10. Solve by factoring.
2 2
a. 2x + 5x = 0 b. –7x + 3x = 0
a. x2 + (a + 1)x + a = 0
2 b. (x + 1)2 – 2(x + 1)(x – 3) + (x – 3)2 = 0
c. 3x2 – 8x = 0 d. x  6 x2 = 0
3
c. 2ax2 + (5b – 2a)x – 5b = 0

3. Solve (x + 2)2 = 2(x + 2). 11. Solve by completing the square.


a. x2 – 4x – 1 = 0 b. x2 + 4x = 3
4. Solve (2x – 1)(x + 3) = –3. c. x2 – 6x – 13 = 0 d. 3x2 – 2x + 4 = 0
e. 4x2 + 8x + 15 = 0 f. 2x2 + 7x + 11 = 0
5. Solve (x + 1) – 3(x + 1) = 0.
2
1 2 1 1
g. x  3x + = 0 h. 3x2 + x =
2 2 2
C. Solving Equations of the Form
12. Solve by using the quadratic formula.
ax2 + c = 0
a. x2 – 4x + 2 = 0 b. 2x2 + x – 1 = 0
6. Solve the equations.
c. 4x2 + 12x + 9 = 0 d. 3x2 – 5x + 1 = 0
a. x2 – 16 = 0 b. 7x2 + 3 = 0
e. 5x2 + 4x + 7 = 0 f. 3x2 – 7 = 2 – 2x
c. 9x2 – 25 = 0 d. 0.6x2 – 15 = 0
g. x2 + x + 1 = 0
2 x2
e. –5x2 + 4 = 0 f.  =0 h. 25x2 + 40x + 16 = 0
25 8
i. (2x – 3)2 = 11x – 19

7. Solve the equations. 3 2 1 1


j. x  x  =0
4 4 2
a. (x – 2)2 – 9 = 0 b. (x + 3)2 + 7 = 0 2
2 2 k. 4x  1 = x(10 x  9)
c. (x + 1) = 5 d. (1 – x) – 1 = 8 3

l. x2  1
=11( x +1)
8. Solve (4x + 1)(x – 1) = (x – 1)(x + 1) – 3(x – 3). 2

68 Algebra 8
13. Solve by using the quadratic formula. 17. For which values of k does the equation
1 2 x2 – (4k + 2)x + 7k + 2 = 0 form a perfect square?
a. 4   =0
x x2
1 1 2
b. + =
x 1 x 1 3
2

2 2
c. (x  3)  (x  2) = 1  x 18. The longest side of a right triangle is 6 cm less
16 4 2 than twice the length of the medium side. The
1 1 5 shortest side is 6 cm. Find the length of the two
d. + =
x 1 x  4 4 other sides.

14. Use the discriminant to determine whether each


19. A wire that is 32 cm long is cut into two pieces,
quadratic equation has two real solutions, a 
and each piece is bent to form a square. The total
double root, or no real solution, without solving
area enclosed by the two squares is 34 cm2. Find
the equation.
the length of each piece of wire.
a. 2x2 + 3x + 1 = 0 b. x2 + 5x – 6 = 0
c. 4x2 + 12x + 9 = 0 d. 25x2 – 20x + 4 = 0
e. x2 + 4x + 7 = 0 f. 2x2 – x + 2 = 0

Mixed Problems
20. a, b, c, and d are four consecutive even natural

numbers. The sum of a and c is one fifth of the
15. Consider the equation ax2 + 3x + 10 = 0. For
product of b and d. Find a, b, c, and d.
which values of a does the equation have

a. two distinct real roots?


b. one double root?
c. no solution?
21. Two squares have sides (x + 6) cm and (2x + 1)

cm respectively. The sum of their areas is
697 cm2. Find the areas of the squares.

16. For which values of m does the equation


2x2 + 2x + m + 4 = 0 have

a. two real solutions? 22. A year ago, a father was eight times as old as his
b. one solution? 
son. Now his age is the square of his son’s age.
c. no real solutions? How old are they now?

Quadratic Equations 69
We have seen that the roots of an equation depend on its coefficients. Therefore, there exist
certain relations between the coefficients and the roots of an equation.
In this section we will consider the relations between the roots and the coefficients a, b and
c of a quadratic equation.
We know that the roots of the quadratic equation ax2 + bx + c = 0, a  0 are

b   b + 
x1 = and x2 = .
2a 2a
François Viète
(or Vieta)
1540-1603, French Let us use these formulas above to find the sum of the roots of a quadratic equation.
mathematician. Vieta
was a founder of b   b+ 
modern algebra, who x1 + x2 = +
introduced the use of 2a 2a
letters as algebraic
symbols and correlated b   + (  b+  )
=
algebra with geometry 2a
and trigonometry.
Vieta presented 2b b
= =
methods for solving 2a a
equations of second,
third and fourth b
degree. He knew the Therefore, x1 + x2 = - .
connection between
a
the positive roots of
equations and the
coefficients of the
different powers of the We can use the same expressions for x1 and x2 to find the product of the roots of a quadratic
unknown quantity. equation.
The word ‘coefficient’
is actually due to Vieta.  b     b+   (  b   )  (  b+  )
When Vieta applied x1  x2 =      =
numerical methods to  2a   2a  4a2
solve equations, he
used methods which b2  (b2  4ac ) 4ac c
were similar to those = = 2=
used by earlier Arabic 4a2 4a a
mathematicians.
c
Therefore, x1  x2 = .
a

These relations were discovered by François Vieta, a French mathematician, and so they are
together called Vieta’s theorem.

70 Algebra 8
Theorem Vieta’s theorem
Let x1 and x2 be the roots of the quadratic equation of the form ax2 + bx + c = 0, a  0. Then

b c
x1 + x2 =  , x1  x2 = .
a a
By using Vieta’s theorem we can now find the sum and product of the roots of a quadratic
equation without calculating the roots.
Note
By using Vieta’s theorem we can also see the following.
1 1 x + x2 – b / a b
1. + = 1 = = (sum of the reciprocals of the roots)
x1 x2 x1x2 c/a c
2
 b  c b2  2ac
2. x12 + x22 = (x1 + x2 )2  2x1x2 =    2  = (sum of the squares of the roots)
 a  a a2
3abc  b 3
3. x13 + x23 = (x1 + x2 )3  3x1x2 (x1 + x2 ) = (sum of the cubes of the roots)
a3

EXAMPLE 20 Derive a formula for the difference of the roots of a quadratic equation in standard form.

Solution Let us consider the two differences. First,

b   b+  b   + b   2   
x1  x2 =  = = = .
2a 2a 2a 2a a


Similarly, x2  x1 = .
a

So we can say that x1  x2 = .
a

EXAMPLE 21 Find the sum and product of the roots of the given equations, without solving the equations.

a. 2x2 + 6x + 5 = 0 b. x2 – 3x – 5 = 0

Solution a. a = 2, b = 6 and c = 5 b. a = 1, b = –3 and c = –5

b 6 b 3
x1 + x2 =  = = 3 x1 + x2 =  = = 3
a 2 a 1
c 5 c –5
x1  x2 = = x1  x2 = = = –5
a 2 a 1
Note
The quadratic equation 2x2 + 6x + 5 = 0 has no real root. However, by Vieta’s theorem, the
5
sum of the roots is –3 and the product of the roots is , which are real numbers. Can you
2
say why?

Quadratic Equations 71
Check Yourself 6
1. Find the sum and the product of the roots of the following equations, using Vieta’s theorem.
7
a. 3x2 + 5x – 1 = 0 b. x2 – 4x + = 0 c. –x2 + 7x – 1 = 0
2
2. For the previous equations, find
a. 1 + 1 . b. x12 + x22. c. x13 + x23.
x1 x2
Answers
5 1 7
1. a.  , – b. 4, c. 7, 1 2. a. 5 b. 9 c. 322
3 3 2

EXAMPLE 22 x1 and x2 are the non-negative roots of the equation 3x2 + 2mx + 1 = 0. Given x1 = 3x2, find
x1, x2, and m.

Solution x1 = 3x2
c 1 1 1 1 1
x1  x2 = = ; 3x2  x2 = ; x2 2 = ; x2 =  since x2 > 0; x2 =
a 3 3 9 3 3
1
x1 = 3x2 then x1 = 3  =1
3
2m 1 2m 4 2m
x1 + x2 =  ; 1+ =  ; =
3 3 3 3 3
m = 2

EXAMPLE 23 x1 and x2 are the roots of the equation x2 – 3x + 1 = 0. Find the value of
x1
x2
+
x2
x1
.

(–3) 1
Solution Since x1 + x2 = – = 3 > 0 and x1  x2 = =1> 0, both x1 and x2 are positive.
1 1
x1 x
+ 2 =k
x2 x1
x12 2 x1x2 x2 2
k2 = + +
x2 x1x2 x1

x13 + x2 3
k2 = + 2 x1x2
x1x2
9 + 27
2
k = 1 + 2 1 = 20
1
k1 = 2 5 , k2 = 2 5

k= 2 5 (why?)

72 Algebra 8
EXAMPLE 24 x2 + x – 6m = 0 and x2 – 2mx + 3 = 0 have a common root. Find m.

Solution Let x1 and x2 be the roots of x2 + x – 6m = 0 and


x1 and x3 be the roots of x2 – 2mx + 3 = 0, then

 x1 + x2 = – 1 x1x2 = – 6m
(1)  and (2)  .
 x1 + x3 = 2m x1x3 = 3
From (1) we get x2 – x3 = –1 – 2m.
x2
From (2) we get = –2m ; x2 = –2mx3.
x3
From (1) and (2),
2 mx3  x3 = 1  2m

x3 (2 m+ 1) = 1+ 2m

1+ 2 m
x3 = = 1.
1+ 2 m
If we substitute 1 for x in the second equation, we have
4 – 2m = 0, so m = 2.

Note
We can apply Vieta's theorem to a cubic polynomial equation.
Let x1, x2 and x3 be the roots of the equation ax3 + bx2 + cx + d = 0. Then,
b c d
1. x1 + x2 + x3 =  . 2. x1x2 + x1x3 + x2 x3 = . 3. x1x2 x3 =  .
a a a

Check Yourself 7
1. One of the roots of the equation 2x2 – mx + 8 = 0 is 3 more than the other root. Find m.
2. x1 and x2 are the roots of the equation x2 + x – m + 1 = 0. x1 – x2 = 5 is given. Find m,
x1, and x2.
Answers
1. 10 2. 7, 2, –3

Mathematics is one component of any plan for liberal education.


Mother of all the sciences, it is a builder of the imagination, a weaver
of patterns of sheer thought, an intuitive dreamer, a poet. The study
of mathematics cannot be replaced by any other activity.

Quadratic Equations 73
EXERCISES 3 .2
1. Find the sum and the product of the roots of each 6. One of the roots of the equation
equation, without solving it. 3
6x2 + 13x + n2 + 2n – 2 = 0 is – . Find n and
a. 10x2 – 11x – 12 = 0 2
the other root of the equation.
7 2 9 5
b. x + x – =0
8 7 3
c. (x – 2)(3x – 4) = 13
d. x + 7 = (2x – 1)(3x – 2) 7. Let x1 and x2 be the roots of the equation
2 2
e. (4x + 3) = (3x + 1) 4x2 + 5x = 0. Find x12x2 + x1x22.
f. x2 = ñ2(3x – ñ2x)

2. The sum of the roots of the equation 8. Consider the equation


(5k + 2)x2 + 7kx – 8k = 0 is 3. Find the product (m + 2n)x2 – (m + 2n)x + m – n = 0.
of the roots. If the arithmetic and the geometric means of the
roots of the equation are equal, find the relation
between m and n.

3. The product of the roots of the equation


(4m2 – 1)x2 + (2m + 1)x + 2m – 1 = 0 is 5.
Find m. 9. Let x1 and x2 be the roots of the equation
x2 – 8x + 5k = 0. x1 = 2x2 – 1 is given. Find k.

4. Find the sum of the squares of the roots of the


following quadratic equations.

a. 2x2 + 5x + 1 = 0 10. Let x1 and x2 be the roots of the equation



b. –x2 + 7x + 2 = 0 x2 + (m + 1)x + m + 2 = 0.
c. 3(x + 2)(x – 1) = 4(x – 2) – 1 2x1 + 3x2 = –13 is given. Find m.
d. 4x(3 – 4x) = x – 1

5. One of the roots of the equation 11. Let x1 and x2 be the roots of the equation
4 6 6
20x2 + (2m2 + 2m + 1)x + 20 = 0 is – . Find (k + 1)x2 – kx + k – 4 = 0. + = 30 is given.
5 x1 x2
m and the other root of the equation. Find k.

74 Algebra 8
12. Consider the equation x2 – (3k + 1)x + 8 = 0. 18. Let p and q be the roots of the equation
2 2 
x + x = 20 is given. Find k.
1 2 2x2 – 5x + p2 + q2 = 0. Find the discriminant of
the equation.

13. Consider the equation x2 + (m + 1)x – m = 0. 1


2 2
19. Find x12 + , if x1 is the root of the equation
x + x = 13 is given. Find m.
1 2 
2
x1
2
 1  1
 x +   6  x +  +9 = 0.
 x  x

14. Let x1 and x2 be the roots of the equation


kx2 + (k – 1)x – 2 + k = 0.
20. Consider the equation x3 + cx + 1 = 0.
2 2 3 
+ = is given. Find k.
x1 + 3 x2 + 3 2 1 1
x1 = + is given. Find the roots of the
x2 x3
equation and c.

15. Consider the equation x2 + (k – 2)x + k – 6 = 0.


If the equation has two negative roots, find all the 21. The roots of the equation x2 – 5x + p = 0 are also

possible values of k. the roots of the equation x3 + qx + 30 = 0. Find
p + q.

16. Find two positive consecutive numbers such that 22. The sum of the two roots of the equation
the sum of their squares is 85. 2x3 – x2 – 7x – 3 = 0 is 1. Find the roots of the
equation.

23. From each corner of a square piece of sheet


metal, a man removes a square of side 2 cm. He
17. Find the product of the roots of the equation turns up the edges to form an open box which

1 1 1 1 holds 24 cm3. What are the dimensions of the
+ =  , a + b  0.
a b a+b+ x x piece of sheet metal?

Quadratic Equations 75
Let x1 and x2 be the roots of the equation ax2 + bx + c = 0.
b c
We can write the equation ax2 + bx + c = 0 as x2 + x + = 0.
a a

 b
 x1 + x2 =  a , and

We know that 
 c
 x1  x2 = a .

So we can write the equation again as x2 – (x1 + x2)x + (x1  x1) = 0.


In other words, if we know the sum and product of the roots of a quadratic equation then we
can write the equation as x2 – Sx + P = 0 where S = x1 + x2 and P = x1  x2. This means
that we can derive (find) a quadratic equation if we know its roots.

Note
If the roots of a quadratic equation are x1 and x2, then (x – x1)  (x – x2) = 0.

EXAMPLE 25 Find a quadratic equation whose roots are –1 and 5.

Solution x1 = –1 and x2 = 5
S = x1 + x2 = –1 + 5 = 4
P = x1  x2 = (–1)  5 = –5
Hence, the equation is
x2 – Sx + P = 0
x2 – 4x – 5 = 0.

1
EXAMPLE 26 Find a quadratic equation whose only root is
3
.

1
Solution x1 = x2 =
3
1 1 2
S = x1 + x2 = + =
3 3 3
1
P = x1  x2 = .
9
2 1
Hence, the equation is x2  x+ =0, or 9 x2 6 x+1 =0.
3 9

76 Algebra 8
EXAMPLE 27 Find the equation whose roots are 2 + ñ3 and 2 – ñ3.

Solution x1 = 2 + ñ3, x2 = 2 – ñ3
S = x1 + x2 = 4
P = x1  x2 = (2 + ñ3)  (2 – ñ3) = 1
Hence, the equation is x2 – 4x + 1 = 0.

EXAMPLE 28 Find the equation whose roots are 1 more than the roots of x2 – 3x – 4 = 0.

Solution Let the roots of x2 – 3x – 4 = 0 be x1 and x2. Let the roots of the equation we are looking for
be x3 and x4.
x1 + x2 = 3, x1  x2 = –4
x3 = x1 + 1, x4 = x2 + 1
S = x3 + x4 = (x1 + 1) + (x2 + 1) = x1 + x2 + 2 = 3 + 2 = 5
P = x3  x4 = (x1 + 1)  (x2 + 1) = x1  x2 + x1 + x2 + 1 = (–4) + (3) + 1 = 0
Hence the equation is x2 – 5x = 0.

EXAMPLE 29 The equation x2 + (k + 4)x + k – 4 = 0 has symmetric roots. Find them.

Solution x1 = t, x2 = –t (symmetric roots)


b c
x1 + x2 =  x1  x2 =
a a
( k+ 4) k 4
t +(  t) = t  ( t) =
1 1
0 = k  4 t2 = 8
k = 4 t2 = 8
t = 2ñ2
Therefore, x1 = 2ñ2, x2 = –2ñ2.

Check Yourself 8
1. Find a quadratic equation whose roots are –2 and 3.
2. The roots of the equation x2 – x – 2 = 0 are x1 and x2. Find the equation whose roots are
x3 and x4, where x3 = 2x1 + 1 and x4 = 2x2 + 1.
Answers
1. x2 – x – 6 = 0 2. x2 – 4x – 5 = 0

Mathematics has beauty but not everyone sees it.

Quadratic Equations 77
EXERCISES 3 .3
1. Find the equation with the given roots. 6. The roots of the equation x2 – kx – 3k + 1 = 0
1 3 are two more than the roots of the equation
a. –1, 1 b. , x2 – (k – 4)x – 4k = 0. Find k.
2 2
c. 0, 4 d. 2 + ñ2, 2 – ñ2
7. Find three consecutive integers a, b, c, such that
1 1
e. ñ3 – ñ2, ñ3 + ñ2 f. , a2 + b2 + c2 = 110.
p2 q 2

8. Find the number that is 3 more than twice its


2. Find the quadratic equation whose sum of the square root.
roots is –2 and product of the roots is 5.
9. A swimming pool can be filled by two pipes
together in six hours. If the larger pipe alone
3. The roots of the equation x2 – 4x – 3 = 0 are
takes five hours less than the smaller pipe alone
x1 and x2. Find the equation whose roots are x3
to fill the pool, find the time in which each pipe
and x4, such that x3 = 2x1 and x4 = 2x2.
alone would fill the pool.

4. Find the equation whose roots are 2 less than the 10. Alex can do a job in one hour less than Jane. If
roots of the equation 2x2 – 6x + 58 = 0. 6
Alex and Jane work together the job takes
5
hours. How long would it take each person working
5. The roots of each given equation are x1 and x2. alone?
Write a new equation with roots x3 and x4.

a. x2 – 2x – 3 = 0 b. x2 + 5x + 4 = 0 11. A man completed a job for $156. It took him


x3 = x1 – 1 x3 = 2x1 + 1 seven hours longer than he expected and so he
earned $14 an hour less than he anticipated.
x4 = x2 – 1 x4 = 2x2 + 1
How long did the man expect the job to take?
c. 6x2 – 9x – 6 = 0 d. x2  2 x+ 1 = 0
x3 = x1 – x2 x12 12. The sum of the numerator and denominator of a
x3 =
x2 certain positive fraction is 11. If 1 is added to both
x4 = x2 – x1
2
x2 the numerator and the denominator, the fraction
x4 =
x1 3
is increased by . Find the fraction.
56
e. x2 – 2mx + 3m – 2 = 0 f. mx2 – 2mx – 1 = 0
x3 = x1 – 2x2 x3 = 2x1 – x2 13. The area of an isosceles right triangle is 81 m2.
x4 = x2 – 2x1 x4 = 2x2 – x1 Find the perimeter of the triangle.

78 Algebra 8
14. The distance between two cities A and B is 19. Two painters working together can paint the front
 of a house in 16 hours. One of the painters alone
140 km. A car driving from A to B left at the same
time as a car driving from B to A. The cars met can finish this job in 24 hours less time than the
after one hour, then the first car reached city other painter alone. How much time does each
B 35 minutes later than the second car reached worker need to do this job alone?
city A. Find the speed of each car.
20. A motorcyclist traveled at a constant speed for 60
km. If he had gone 10 km/h faster, he would have
15. An aeroplane traveled a distance of 400 km at an
shortened his traveling time by one hour. Find the
average speed of x km/h. Write down an expression
speed of the motorcyclist.
for the time taken. On the return journey, the
speed increased by 40 km/h. Write down an
expression for the time for the return journey. If 21. The area of a triangle is 36 m2. The length of the
the return journey took 30 minutes less than the base is twice the height. Find the length of the
outward journey, write down an equation in x and base and the height of the triangle.
solve it.
22. A jeweler wishes to mix an alloy of 25% silver with
another alloy of 40% silver. How much of each
16. A car drove from one city to another and returned should he use to produce 60 kg of an alloy which

by a different route. The outward journey was 48 is 30% silver?
km and the return journey was 8 km shorter. The
speed of the car on the return journey increased
23. One of two pipes can fill a pool 24 hours faster
by 4 km/h. The return journey took one hour less 
than the other one. The slower pipe filled the pool
time. Find the speed of the car on the outward
for eight hours, then the other pipe was opened.
journey.
The pipes filled together for twenty hours and
2
filled of the pool. Find the time that each pipe
3
17. A group of women plan to share equally in the requires to fill the pool alone.
$14,000 cost of a boat. At the last minute three of
the women decide not to pay. This raises the
24. A worker can clean a pool in four hours less time
share for each of the remaining women by $1500.
How many women were in the original group? than it takes another worker. If the men work
8
together the job takes hours. How long would
3
it take each man working alone?
18. A car drove from a city A to a city B. The distance
between the two cities is 350 km. After 200 km,
the speed of the car decreased by 20 km/h. The 25. The length of a rectangle is 2 m more than its
total trip took 5 hours. Find the speed of the car width and the area is 48 m2. Find the length and
in the first part of the journey. width of the rectangle.

Quadratic Equations 79
A. WRITING EQUATIONS IN QUADRATIC FORM
Definition standard form of an equation
An equation is in standard form if the only term on the right-hand side of the equation is zero.

For example, the equations 6x2 + 2x – 3 = 0 and x4 – 5 = 0 are both in standard form. The
equation 6x2 + 2x = 3 and x4 = 5 are not in standard form.
Certain equations that are not quadratic can be expressed in quadratic form using
substitutions. These equations can be recognized because when they are written in standard
form, the exponent of the variable in one term is half the exponent of variable in the other term.
For example, we can write standard form equations such as
x4 + 17x2 + 72 = 0
2x8 + 4x4 = 0
x – ñx – 12 = 0
as quadratic equations, because the exponent of the first variable is twice the exponent of the
second variable.

Look at the steps to write an equation as a quadratic.


1. Let t be a variable term with the half exponent.
2. Substitute t in all the terms with the variable.
3. Solve for t.
4. Back substitute for the original variable.

EXAMPLE 30 Solve x4 – 13x2 + 36 = 0.

Solution 1 The equation x4 – 13x2 + 36 = 0 is not a quadratic equation but we can write it as
(x2)2 – 13x2 + 36 = 0. For this reason, it is a quadratic in x2. Let x2 = t.
First we solve for t, then solve the resulting equations for x.
(x2)2 – 13x2 + 36 = 0
x2 = t, so t2 – 13t + 36 = 0. By factoring,
(t – 4)(t – 9) = 0
t = 4 or t = 9.
Since t = x2
x2 = 4 x2 = 9
or .
x = 2 x = 3

80 Algebra 8
Solution 2 Alternatively, we can directly factorize the equation.
x4 – 13x2 + 36 = 0
(x2 – 4)(x2 – 9) = 0
x2 = 4 x2 = 9
or
x = 2 x = 3
In both solutions, the roots of the given equation are –3, –2, 2, 3.

EXAMPLE 31 Solve (5x – 1)2 + 4(5x – 1) – 5 = 0.

Solution For the equation (5x – 1)2 + 4(5x – 1) – 5 = 0, we let t = 5x – 1 so that t2 = (5x – 1)2. Then
the original equation becomes t2 + 4t – 5 = 0. First solve for t:
(t + 5)(t – 1) = 0, so
t = –5 or t = 1.
Now, solve for x.
5x – 1 = –5 5x – 1 = 1
or
5x = –4 5x = 2
–4 2
x1 = x2 =
5 5
–4 2
Hence, the roots of the equation are and .
5 5

EXAMPLE 32 Solve (a2 – a)2 – 2(a2 – a) = 0.

Solution Let t = a2 – a.
The equation becomes
t2 – 2t = 0
t(t – 2) = 0
t = 0 or t = 2.
Now, solve for a.
1. a2 – a = 0
a(a – 1) = 0
a = 0 or a = 1
2. a2 – a = 2
a2 – a – 2 = 0
(a – 2)(a + 1) = 0
a = 2 or a = –1
Hence, the roots of the equation are –1, 0, 1, 2.
Quadratic Equations 81
4
EXAMPLE 33  1 2
Solve  x   + 5 = 2 x2 + 2 .
 x x

Solution First x2  0, so x  0.
1
Let x  = t.
x
How can we write the right-hand side in terms of t?
Let’s take the square of t:
2
 1 1
t2 =  x   = x2 + 2  2, so
 x x
1
x2 + 2 = t 2 + 2. Now, let us multiply both sides of the equation by 2:
x
2
2 x2 + 2 = 2t2 + 4.
x
Then we have the equation
t4 +5 = 2t2 + 4
t4  2t 2 +1= 0
(t2  1)2  0
t2  1= 0
t =1 or t = 1.
1
Since t = x  ,
x
1 1
x  =1 or x  = 1.
x x
Let us multiply the equations by x:
x2  x  1= 0 or x2 + x 1= 0.
By using the quadratic formula,
1 5 1+ 5 1+ 5 1  5
x1 = , x2 = , x3 = , x4 = .
2 2 2 2

Check Yourself 9
Solve the equations.

1. (x + 15)2 – 3(x + 15) – 18 = 0.


2. (x + 5)4 – 6(x + 5)2 – 7 = 0.
3. x – 9ñx + 14 = 0.
Answers
1. –18, –9 2. –5 ñ7 3. 4, 49

82 Algebra 8
B. EQUATIONS INVOLVING PRODUCTS AND QUOTIENTS
If the product of two or more numbers is zero, then at least one of the factors must be zero.
If the quotient of a division is zero, then the dividend must be zero.

Note
1. P  Q = 0 if and only if P = 0 or Q = 0, where P = P(x) and Q = Q(x).
P
2. = 0 if and only if P = 0 and Q  0.
Q

EXAMPLE 34 Solve (x2 – 1)(x2 – 2x – 8) = 0.

Solution Try to factorize each part if possible:

(x – 1)(x + 1)(x – 4)(x + 2) = 0.

If the product is zero then at least one of the factors is zero.

x–1=0; x=1

x+1=0; x = –1

x–4=0; x=4

x+2=0; x = –2

Thus, the roots of the equation are –2, –1, 1, 4.

EXAMPLE 35 Solve (x2 – 4x + 10)(x2 – 5x + 2) = 0.

Solution We cannot factorize the parts.


Let us try to solve x2 – 4x + 10 = 0 and x2 – 5x + 2 = 0 as two different quadratic
equations.
x2 – 4x + 10 = 0;  = 16 – 4  1  10 = –24 < 0.
Therefore, this equation has no real root.
x2 – 5x + 2 = 0 ;  = 25 – 4  1  2 = 17
This time, by the quadratic formula, the roots are
5  17 5+ 17
x1 = , x2 = .
2 2

Quadratic Equations 83
EXAMPLE 36 Solve x3 – x2 – 4x + 4 = 0.

Solution First try to factorize the expression.


x3 – x2 – 4x + 4 = x2(x – 1) – 4(x – 1) = (x – 1)(x2 – 4) = (x – 1)(x – 2)(x + 2).
Now, the question becomes:
solve (x – 1)(x – 2)(x + 2) = 0.
So the solution is
x–1=0; x1 = 1
x–2=0; x2 = 2
x+2=0; x3 = –2.

x2  5x  6
EXAMPLE 37 Solve
x+ 2
= 0.

Solution The denominator of a fraction cannot be equal to zero, so x + 2  0, x  –2.


We need to solve x2 – 5x – 6 = 0.
x2 – 5x – 6 = (x – 6)(x + 1)
(x – 6)(x + 1) = 0
x1 = –1 or x2 = 6
Since these roots are not equal to –2, x1 = –1 and x2 = 6 are both solutions to the equation.

x2 + 7 x  8
EXAMPLE 38 Solve
x2  3x+ 2
= 0.

Solution First solve x2 – 3x + 2 0: x  2, x 1.


x2 + 7x – 8 = (x – 1) (x + 8) = 0
So x1 = 1 and x2 = –8 are the roots of the numerator.
We know that x  1, so the only solution is –8.

Note
It is very important to check the roots of the numerator to see whether they make the
denominator zero or not. We can do this either by substituting the roots of the numerator in
the denominator, or by finding the roots of the denominator directly and checking whether
they are common or not.

84 Algebra 8
EXAMPLE 39 Solve
x  7 x+ 3

x  4 x+ 4
= 1.

Solution x – 4  0, and x + 4  0, so x  4, and x  –4 (since the denominators cannot be zero).


x  7 x+ 3
Now we have   1 = 0.
x  4 x+ 4
Let us make the denominators common:
(x  7)( x+ 4)  ( x+ 3)( x  4)  ( x2 16)
= 0.
(x+ 4)(x  1)

 x2  2 x
=0
(x  4)(x+ 4)

–x2 – 2x = 0 ; –x(x + 2) = 0 ; x1 = 0 and x2 = – 2.

x+ 1 x2  5
EXAMPLE 40 Solve
4
 =
x+ 1 x  1 x2  1
.

4 x+ 1 x2  5
Solution In its present form, the equation  = is not a quadratic equation. However,
x+ 1 x  1 x2  1
we can make it quadratic by multiplying each side by x2 – 1 since (x2 – 1  0 ; x  1). The result is
4(x – 1) – (x + 1)2 = x2 – 5
4x – 4 – x2 – 2x – 1 = x2 – 5
–2x2 + 2x = 0
–2x(x – 1) = 0
x1 = 0 or x2 = 1, but x  1, so x = 0 is the only possible solution.

EXAMPLE 41  x 
Solve   
 x+ 2 
5x
x+ 2
+ 6 = 0.

Solution x + 2  0 ; x  –2
x
Let t = , then the equation becomes
x+ 2
t2 – 5t + 6 = 0
(t – 3)(t – 2) = 0
t1 = 2 or t2 = 3.

Quadratic Equations 85
Now, solve for x:
x x
=2 or =3
x+ 2 x+ 2
x = 2x+ 4 x = 3 x+ 6
x2 = 4 x1 = 3.

So the roots are –4 and –3.

Check Yourself 10
9
1. Solve + 2 x = 4.
4x+ 3
x+ 1 x  3 12
2. Solve  = .
x  2 x+ 2 x2  4

Answers
1 3
1.  ; 2. no real solution
4 2

C. EQUATIONS INVOLVING RADICALS


When the variable in an equation occurs in a square root, cube root, and so on, that is, when
it occurs in a radical, the equation is called a radical equation. For example, the equations
3
x+ 1 = 2 and x – 1 – 5 = 0 are radical equations. Sometimes a suitable operation will
change a radical equation to an equation that is linear or quadratic.

To solve a radical equation, follow the procedure.

1. Isolate the radicals

Isolating a radical means putting the radical on one side of the equation and everything else
on the other side, using inverse operations. If there are two radicals in the equation, isolate
one of the radicals.

2. Get rid of the radical sign

Raise both sides of the equation to a power equal to the index of the isolated radical.

3. If there is still a radical sign left, repeat steps 1 and 2.

4. Solve the remaining equation

86 Algebra 8
5. Check for extraneous solutions
When you solve a radical equation, extra solutions may come up when you raise both sides
to an even power. These extra solutions are called extraneous solutions. In radical equations
you check for extraneous solutions by putting the values you found into the original
equation. If the left side of the equation does not equal the right side then you have an
extraneous solution.

Note
1. If a value is an extraneous solution, it is not a solution to the original problem.
2. It is very important to check your results in the original equation. In many equations, one
of the results may not satisfy the original equation. However, sometimes it is possible that
all results that you have found will be acceptable.

EXAMPLE 42 Solve the radical equation 2 x+ 5 = 7.

Solution Here the radicand is already alone; we do not need to isolate it. So take the square of both sides:
2x + 5 = 49 ; 2x = 44 ; x = 22.
Now let us check to see if x = 22 is an extraneous solution:

2 x+ 5 = 7

2  22 + 5 = 7

49 = 7

7 = 7.
Since the last statement is true, x = 22 is not an extraneous solution. Therefore, there is one
solution to this radical equation, x = 22.

EXAMPLE 43 Solve 2 x  5 + x = 4.

Solution First we isolate the radical:


2 x  5 = 4  x.
Now take the squares of both sides to eliminate the square root:
2x – 5 = 16 – 8x + x2.
The new equation is x2 – 10x + 21 = 0
(x – 3)  (x – 7) = 0
x = 3 or x = 7.
Let us check the results in the original equation:

Quadratic Equations 87
x=7; 2 .7  5 + 7 = 4 x=3; 2  3  5 +3 = 4

9 +7 = 4 1+ 3 = 4
3+7 = 4 4=4
10 = 4 This is true, so x = 3 is a solution.
This is false, so 7 is an extraneous solution.
Hence, the only solution to the equation is x = 3.

EXAMPLE 44 Solve 10 x+ 56  2 x +8 = 4 .

Solution In this question there are two radical expressions. We can isolate only one expression, so it
is better to isolate the more complex one. So we have
10 x+ 56 = 2 x+ 8 + 4.
Take the squares of both sides:
10 x+ 56 = (2 x+ 8) + 2  4  2 x+ 8 +16 .
This is a new equation involving radical expressions. Follow the same steps again to isolate
the second radical.

8 x+ 32 = 8  2 x+ 8

x+ 4 = 2 x+ 8

x2 + 8 x+ 16 = 2 x+ 8

x2 + 6 x+ 8 = 0
(x+ 4)  (x+ 2) = 0
x = –4 , x = –2
Now, check these results in the original equation.

x = –4 ; 2  (–4) + 8  10 (–4) +56 = 4


0  16 = 4
4 = 4
This is true!
x = –2 ; 2  (–2) + 8  10 (–2) + 56 = 4
4  36 = 4
2  6 = 4
4 = 4
This is true!
Hence, both –4 and –2 are solutions to the equation.

88 Algebra 8
EXAMPLE 45 Solve the equation 5 + 3 x+ 3 = 3.

Solution 5 + 3 x+ 3 = 3
3
x+ 3 = 2 (by taking the cube of both sides)

x+ 3 = 8
x = –11. We do not need to check for extraneous solutions because this is an odd power.
Therefore, –11 is the only solution to the equation.

EXAMPLE 46 Solve the equation 4x+ 1 + x+ 2 = 10 x+ 5 .

Solution ( 4 x+1 + x+ 2 )2 = ( 10 x+ 5) 2

4x+ 1+ 2 (4x+ 1)(x+ 2) + x+ 2 = 10 x+ 5

2 (4 x+1)( x+ 2) = 5 x+ 2

(2 (4 x+1)(x+ 2))2  (5 x+ 2)2

4(4 x+1)(x+ 2)  25 x2 + 20 x+ 4

16 x2 + 36 x+ 8 = 25 x2 + 20 x+ 4

9x2  16 x  4 = 0

(9 x+ 2)( x  2) = 0
2
x=  or x = 2
9

Now, check these results in the orijinal equation.

2 8 2 20 1 16 5 5 5
x=  ;  +1 +  + 2 =  + 5 = + = ; =
9 9 9 9 9 9 3 3 3

x=2 ; 8 +1 + 2 + 2 = 20 + 5 ; 5 =5

2
Both statements are true, so both x =  and x =2 are solutions.
9

Quadratic Equations 89
Check Yourself 11
Solve the equations.
1. 5x+ 3 = 4 2. 2x  4 = x  2 3. x+ 2  4 x+ 8 = –3

4. 3x2  2 x+15 + 3x2  2 x+ 8 = 7


Answers
13 1
1. 2. 2, 4 3. 7 4.  , 1
5 3

D. EQUATIONS INVOLVING AN ABSOLUTE VALUE


On the real number line, the absolute value of x is the distance from the origin to the point
x. For example, there are two points whose distance from the origin is three units, –3 and 3.
So the equation |x| = 3 has two solutions, 3 and –3. Let us first remember the mathematical
definition of absolute value.

Definition absolute value of a function


For all real numbers x,
 f x  , for f x   0

f ( x) = 
 – f x  , for f x  < 0.

We can use this information to begin solving equations involving one or more absolute values.

EXAMPLE 47 Solve the equation |x – 2| = 5.

Solution Case 1
__________________ Case 2
_________________
x–20; x2 x–2<0; x<2
The absolute value of a x–2=5 –(x – 2) = 5
number is never negative. x=7 x = –3
|a|  0
So the solutions are –3 and 7.

EXAMPLE 48 Solve |2x – 3| = x + 1.

Solution 1 Case 1
__________________ Case 2
____________________
3 3
2x  3  0 ; x  2x  3 < 0 ; x <
2 2
2 x  3 = x+ 1 (2 x  3) = x+ 1
x= 4 2 2 3
x= since < ,
3 3 3 2
since 4 > , x = 4 is a solution.
2 2
x= is a solution.
3
2
So the solutions are 4 and .
3

90 Algebra 8
Solution 2 Let us take the square of both sides:
4x2 – 12x + 9 = x2 + 2x + 1
3x2 – 14x + 8 = 0
|f(x)|2=f 2(x)= f(x). f(x)
(x – 4)  (3x – 2) = 0.
2
So the solutions are 4 and .
3

EXAMPLE 49 Solve x2 – 2|x + 2| + 1 = 0.

Solution Case 1
_______________________ Case 2
________________________
x + 2  0 ; x  –2 x + 2 < 0 ; x < –2
x2 – 2(x + 2) + 1 = 0 x2 – 2(–(x + 2)) + 1 = 0
x2 – 2x – 3 = 0 x2 + 2x + 5 = 0
(x – 3)(x + 1) = 0  = –16
x = 3 or x = –1 Since  < 0, there are no real roots.
Both 3 and –1 are greater than –2, so the solutions to the equation are –1 and 3.

EXAMPLE 50 Solve the equation |x2 – 5x + 3| = 3.

Solution Case 1
_______________________ Case 2
________________________
x2 – 5x + 3  0 x2 – 5x + 3 < 0
x2 – 5x + 3 = 3 –(x2 – 5x + 3) = 3
x2 – 5x = 0 x2 – 5x + 6 = 0
x(x – 5) = 0 (x – 3)(x – 2) = 0
x = 0 or x = 5 x = 3 or x = 2
These values satisfy the inequality, so they These values also satisfy the inequality, so
are solutions. they are also solutions.
Hence, the solutions to the equation are 0, 2, 3, 5.

EXAMPLE 51 Solve the equation x  3x+ 1 = x  4x+ 4 .


2 2

Solution Remember that x2 – 4x + 4 = (x – 2)2. Hence, the expression becomes

x2  3x+ 1 = (x  2)2 = x  2
For any real number x,
x2 = x .
x2  3x+ 1 = x  2 .

Quadratic Equations 91
Case 1
_______________________ Case 2
________________________
x–20; x2 x–2<0; x<2
2
x – 3x + 1 = x – 2 –(x2 – 3x + 1) = x – 2
x2 – 4x + 3 = 0 x2 – 2x – 1 = 0
(x – 3)(x – 1) = 0 x1 = 1 – ñ2, x2 = 1 + ñ2
x = 3 or x = 1 Since (1 + ñ2) is greater than 2,
Since 1  2, x = (1 – ñ2) is the only solution.
x = 3 is the only solution.
Hence, the solutions to the equation are (1 – ñ2) and 3.

EXAMPLE 52 Solve the equation |2x – 3| = |x – 7|.

Solution 1 Let’s take the square of both sides. Then,


4x2 – 12x + 9 = x2 – 14x + 49
3x2 + 2x – 40 = 0
(3x – 10)(x + 4) = 0
10
x1 = –4 or x2 = .
3
10
Therefore, both –4 and are solutions to the equation.
3

Solution 2 2x – 3 = x – 7 or 2x – 3 = –(x – 7)
10
x = –4 x=
3
If |f(x)| = |g(x)|
then f(x) = g(x). 10
So both –4 and are solutions to the equation.
3

Check Yourself 12
Solve the equations.
1. |2 – x| = 2x + 1 2. x|x + 1| – 2 = 2x
Answers
1
1. 2. –2, –1, 2
3

Algebraic symbols are used when you do not know what you are talking
about.

92 Algebra 8
The following problem is posed and solved in the Chiu Chang Suan-Shu,
a Chinese mathematical treatise which is over two thousand years old.

A tree of height 20 m has a circumference of 3 m. An arrowroot vine


winds seven times around the tree before it reaches the top. What is the
length of the vine?

Can you solve this puzzle?

EXERCISES 3 .4
A. Writing Equations in Quadratic 1 2 6
d. + 2 = 2
Form 2
x  3x+ 3 x  3x+ 4 x  3x+ 5
1. Solve the equations. e. (x + 1)(x + 2)(x + 3)(x + 4) = 120
a. x4 – 13x2 + 36 = 0 B. Equations Involving Products and
4 2
b. 3x – 8x – 3 = 0 Quotients
c. (x2 – 9)2 – 4(x2 – 9) + 3 = 0 3. Solve the equations.
4 2
d. (x + 5) – 6(x + 5) – 7 = 0 a. (16x3 – x)(x2 – 6x + 5) = 0
2
 3  4x  b. (x2 + 8x)(x2 + 8x – 6) = 280
e.    2(3  4 x) + 25 = 0
 5 
c. x3 + 4x2 – 24 = 0
f. x+ 2  4 x+ 2 = 6 d. x3 – 5x2 + 9x – 45 = 0
3
g. x3  7 x 2  8 = 0 2x+ 3 3x  2
e. =
2 4x  1 3x+ 2
1 7 1
h.  x+    x+  = 2
 x 2  x f. (x2 – 5x + 6)2 – (2x2 – 5x + 1)2 = 0

i. x2 + 2 8x g. (6x2 – 5x – 4)2 + (10x2 – 29x + 21)2 = 0


+ 2 =6
x x +2

2. Solve the equations. 3x2  9x 12


h.  2 =3
2 x  3x
 1   1
a. 3  x2 + 2   7  x+  = 0
 x   x 3x+ 4 6(x  2)
i.  =1
2 2 2 x2 3x+ 4
b. (x – 2x – 5) – 2(x – 2x – 3) – 4 = 0
3 4 x2 x 2 8
c. = 3  x  x2 j. + + 4  + = 0
1+ x+ x2 x2
9  3 x 3

Quadratic Equations 93
C. Equations Involving Radicals D.Equations Involving an Absolute
Value
4. Solve the equations.
6. Solve the equations.
a. x  1  1= 0 a. |x| = x + 2

b. ñx + x = 5 b. |2x – 5| = x – 2
c. x  |x – 1| = 2
c. 3
x  1 = 2
d. |x2 – 4x| = 5
d. x 1 = x 1 e. |x2 – 2x + 3| = 6
f. x2 + |x – 1| + 1 = 0
e. x 3+ x 8 =5
g. x2 + |2x – 1| + 3 = 4x + 2
f. 3  x +1 = x  2 h. x2  2 x +1 = 2 x  3
3
g. x2  x +6  2 = 0 i. x2 + x2  6 x +9 = 0

h. x +16  x = 2 7. Solve the equations.


i. x2  6 x  x2  6x  3 = 5 a. |x| + x2 = 0

1
j. 4x – 1+ 2 x + 3 =1 b. ( x +1)( x – 1) = 
2
c. (x + 1)2 – 2|x + 1| + 1 = 0
5. Solve the equations.
 d. |x – x2 – 1| = |2x – 3 – x2|
a. x +1  9  x = 2 x  12 x2  4 x + 3
e. =1
x2 + x  5
b. 2 x +5 + 5 x +6 = 12 x + 25
f. |x + |3 – 2x|| = 3 – x
c. x  x +1+ x +9  x + 4 = 0
Mixed Problems
d. 2
2 x + 6 x +1 = x +1
8. Solve the equations.

e. 1  x2  3x = x +1
a. x2  2x +1 =1  x

f. x2 +1+ x2  8 =1 b. x2 + x2  6 x +9 = 2 x + 3

g. 3x2 +5 x +8  3 x2 +5 x +1 =1 2
c. x + x +1 = 2 x  4x + 4

h. 1+ x + 2 = 2 x + 2 +5 d. x2  x + 2 = x2 + 4x + 4 + x

94 Algebra 8
Sometimes we need to solve two or more equations simultaneously. A set of equations like
this is called a system of equations. There are no concrete rules that we can follow to solve
systems of equations, but let us look at some general strategies.

EXAMPLE 53 Solve the system of equations.


x+y=5
xy = 6

Solution We can write y = 5 – x. Now substitute this value of y in the second equation.
x(5 – x) = 6
x2 – 5x + 6 = 0
(x – 2)(x – 3) = 0
x = 2 or x = 3
If x = 2 then y = 3.
If x = 3 then y = 2.
Therefore, the solutions of the system are (2, 3) and (3, 2).

EXAMPLE 54 Solve the system.


x2 + y2 = 65
x  y = 28

Solution 1 Multiply the second equation by 2 and then add and subtract the resulting equations.
x2 + y2 = 65 x2 + y2 = 65
2x  y = 56 2x  y = 56
+
________________________ –______________________
x2 + 2xy + y2 = 121 x2 – 2xy + y2 = 9
(x + y)2 = 121 (x – y)2 = 9
x + y = 11 x – y = 3
Therefore, we have four cases.

Quadratic Equations 95
Case 1
__________ Case 2
__________ Case 3
__________ Case 4
__________
x + y = 11 x + y = 11 x + y = –11 x + y = –11
x–y=3 x – y = –3 x–y=3 x – y = –3
+
____________ +
_____________ +
_____________ +
_____________
2x = 14 2x = 8 2x = –8 2x = –14
x=7 x=4 x = –4 x = –7
y=4 y=7 y = –7 y = –4

The solutions of the system are therefore (7, 4), (4, 7), (–7, –4), (–4, –7).

 x2 + y2 = 65
Solution 2 
 x  y = 28

28
y= , x  0, substitute y in the first equation.
x
2
 28 
x2 +   = 65
 x 

28 2
x2 + = 65
x2
x4  65 x2 + 784 = 0

(x2  49)( x2  16) = 0

x2 = 49 or x2 = 16

x = 7 or x = 4

28
By substituting these x values in y = , we get the y values.
x
The solutions of the system are (7, 4), (–7, –4), (4, 7), (–4, –7).

EXAMPLE 55 Solve the system.


x2 = 13x + 4y
y2 = 4x + 13y

96 Algebra 8
Solution Let us subtract the two equations side by side.
x2 – y2 = 9x – 9y
(x – y)(x + y) = 9(x – y)
x – y = 0 or x + y = 9
Case 1
______________________________________________________
x–y=0; x=y
Let us substitute x in the equation y2 = 4x + 13y.
y2 = 4y + 13y
y2 – 17y = 0
y1 = 0 or y2 = 17
x1 = 0 or x2 = 17
Therefore we have two solutions, (0, 0), (17, 17).
Case 2
______________________________________________________
x+y=9; x=9–y
Let us substitute x in the equation y2 = 4x + 13y.
y2 = 4(9 – y) + 13y
y2 – 9y – 36 = 0
(y – 12)(y + 3) = 0
y3 = 12 or y4 = –3
x3 = –3 or x4 = 12
Therefore we have two more solutions, (–3, 12), (12, –3).
Hence, the solutions of the system are (0, 0), (17, 17), (–3, 12), (12, –3).

EXAMPLE 56 Solve the system.


 x+ y x  y 5
 x  y + x+ y = 2

 2 2
 x + y = 20

Solution x – y  0 ; x  y and x + y  0 ; x  –y.


Let us multiply both sides of the equation by 2(x – y)(x + y).
2(x + y)2 + 2(x – y)2 = 5(x – y)(x + y)
2x2 + 4xy + 2y2 + 2x2 – 4xy + 2y2 = 5x2 – 5y2
–x2 + 9y2 = 0.

Quadratic Equations 97
 2
 x + 9y = 0
2

Now, we have the system  .


 x2 + y2 = 20
Let us add these two equations side by side.
10y2 = 20
y2 = 2 ; y = ñ2
x2 = 18 ; x = 3ñ2
Hence, the solutions of the system are (3ñ2, ñ2), (–3ñ2, ñ2), (3ñ2, –ñ2), (–3ñ2, –ñ2).

Check Yourself 13
Solve the systems of equations.
 x2  y = 2  x2 + 3xy  10y2 = 0
1.  2. 
 x+ y = 4  x2 + 2xy  y2 = 28
Answers
1. (2, 2), (–3, 7) 2. (4, 2), (–4, –2), (5ñ2, –ñ2), (–5ñ2, ñ2)

One person’s constant is another person’s variable.

EXERCISES 3 .5
1. Solve each system of equations. 2. Solve each system of equations.

 x2 + y2 + 6x+ 2y = 0 3x+ xy  y2 = 0
 x  y = 12 9x2  y2 = 44 a.  b. 
a.  b. 
 x+ y+ 8 = 0 2 x2  3xy+ y2 = 0
 x  y = 108 3x  y = 11
 1 1
6 x + xy  y = 0
2 2  x+ y + x  y = 2
1 1 1
 x = 3  y  + = c.  d. 
c.  d.  x y 6 3x2  xy  y2 = 0  3 4
 y2  x = 39   x+ y + x  y = 7
 x+ y = 25 
 x+ y  xy = 2  x3  y3 = 8
 x2  3y2 = 13  x2 + y2 = 18 e.  f. 
e.  f.   xy(x+ y ) = 48  x  y = 2
 xy = 4  xy = 9
 x+ y 2(x  y)
  =1 3x2  2 xy  y2 = 7
 x+ y+ xy = 5  x2 + 3x  4y = 20 g.  x  y x+ y h. 
g.  h. 
 xy+ x  y = 13  2  x2 + xy+ 8y2 = 14
 x  2 x+ y = 5
2
 x  5xy+ 2y = 4
2

98 Algebra 8
CHAPTER REVIEW TEST 3A
1. Find the solutions of the quadratic equation 5. The sum and product of the roots of the equation
2
2x + 5x + 3 = 0. 2x2 – hx + 2k = 0 are 4 and –3, respectively.
Find the values of h and k.
1 1 3
A) and 3 B) 3 and C) 1 and 
2 2 2 A) 8 and –6 B) 4 and –3 C) –3 and 4
1 1 D) –3 and 8 E) 8 and –3
D)  and –3 E) 1 and
2 2

6. What is the sum of the roots of the equation


2. –1 is a root of x2 + mx + n = 0 and
x4 – 13x2 + 36 = 0?
m + n = 5. Find m.
A) 13 B) 18 C) 1 D) 0 E) –13
A) 1 B) –1 C) 2 D) –2 E) 3

3. Which one of the following is false for the 7. Which one of the following equations has roots
equation x2 – 7x + 1 = 0? –2 and 3?

A) 2x2 – x – 6 = 0 B) x2 + x – 6 = 0
A) The roots are not integer.
B) The roots have the same sign. C) x2 – x – 6 = 0 D) x2 + x + 6 = 0

C) The roots are rational. E) 2x2 + x – 6 = 0

D) The roots are both positive.


E) The sum of the roots is positive.

25
8. The sum of a number and its reciprocal is .
12
Find the number.
4. Find the discriminant of x2 – 2ñ5x + 4 = 0.
4 1
A) 1 B) –1 C) D) – E) –3
A) 2 B) 4 C) 36 D) 16 E) 24 3 2

Chapter Review Test 3A 99


9. What are the roots of the equation 13. What is the solution of the equation x = ñx + 6?
1 3
 + 2 = 0? A) –9 B) 4 C) –4 D) 16 E) 9
x2 x

1 1 1 4
A) 1 and B) and 1 C)  and
2 2 2 3

1
D) – and – 1 E) –1 and 1
2
14. Which one of the following is not a solution of the
system
 5 x2  6 xy+ 5 y2 = 29
 ?
 7 x2  8 xy+ 7 y2 = 43

A) (2, 3) B) (–2, –3) C) (–2, 3)

10. One of the roots of the equation ax2 + bx + c = 0 D) (3, 2) E) (–3, –2)
is twice the other root. What is the relation
between the coefficients a, b, and c?

A) 4b2 = 4c B) 2b2 = 9ac C) 2b2 = 9a


D) b2 = 8ac E) 9b2 = 2ac 15. Two technicians can complete a mailing job in 12
hours while working together. Alone, the first
technician can complete the mailing job seven
hours faster than the other technician. How long
will it take each technician to complete the mailing
alone?

11. The diagonal of a rectangle is 5 cm, and the area A) 14 and 21 B) 21 and 28 C) 7 and 14
is 12 cm2. Find the perimeter of the rectangle. D) 11 and 18 E) 15 and 22
A) 12 B) 24 C) 10 D) 14 E) 15

16. The equations 6x2 – 18x + 12 = 0 and


mx2 – nx + 1 = 0 have the same solution. Find
m and n.

3 1 3 1 3 1
A)  and  B)  and C) and
2 2 2 2 2 2
12. x – 1  x+ 1 + 1 = 0. Find the value of 4x.
1 3 3 1
D) and E) and 
A) 5 B) 4 C) 0 D) 16 E) –4 2 2 2 2

100 Algebra 8
CHAPTER REVIEW TEST 3B
1. x1 and x2 are roots of the equation 5. The longest side of a right triangle is 3 cm less
x2 + (2 – x1)x + 7x2 = 0. What is x1 + x2? than twice the length of the medium side. The
shortest side is 3 cm. Find the lengths of the other
A) –2 B) 5 C) 7 D) 10 E) –14 two sides.

A) 8 and 15 B) 7 and 14 C) 8 and 10

D) 4 and 5 E) 6 and 8

2. Which one of the following is true for the


equation |x|2 + |x| – 6 = 0?

A) There is only one root. 6. A triangle has area of 2 cm2. The base of the
triangle is 3 cm longer than its height. Find the
B) The product of the roots is –3.
length of the base and the height of the triangle.
C) The product of the roots is 4.
D) The sum of the roots is 0. A) 5 and 2 B) 4 and 1 C) 6 and 3

E) The sum of the roots is 1. D) 7 and 4 E) 8 and 5

7. Which one of the following equations has roots


3. The sum of the squares of the roots of the equation 1 – ñ3 and 1 + ñ3?

x2 + 2hx = 3 is 10. Find the value of h. A) x2 – 2x – 2 = 0 B) x2 + 2x – 2 = 0

A)
1
B) –3 C)
3
D) 
1
E) –1 or 1 C) x2 – x – ñ3 = 0 D) x2 – 2x + 2 = 0
2 2 2 E) x2 – 6 = 0

8. The sum and product of two numbers are 21 and


4. Find the discriminant of 2x – 2x + 5 = 0.
2
104, respectively. What is the bigger number?

A) –36 B) 4 C) 25 D) –16 E) 9 A) 16 B) 22 C) 13 D) 14 E) 19

Chapter Review Test 3B 101


9. What are the roots of the equation 1 1

2
13. What is the solution to the equation x 2 + x 4 = 2?
 x   x 
3  +7   – 6 = 0?
 x+ 1   x+ 1  A) 9 B) 1 C) –4 D) 4 E) 16

3 1 1 4
A)  and 2 B) and 2 C)  and
4 2 2 3
1
D)  and –3 E) –1 and 2
2
14. Which one of the following is not a solution of the
system
 x2 + y2 = 26
 ?
 x2  y2 = 24

A) (1, –5) B) (–5, –1) C) (–5, 1)

10. Which number is a solution to the equation D) (5, 1) E) (5, –1)

x2 + |x – 1| – 3 = 0?

2 3
A) 3 B)  C) D) 0 E) –1
2 2

15. A pipe can fill a pool four hours faster than another
pipe. The slower pipe filled the pool for seven
hours, then the other pipe was opened. The pipes
then filled the pool in two hours, working together.
How long would each pipe take to fill the pool
alone?
11. x – y = 12 and x  y = 108 are given. Find x + y.
A) 14 and 18 hours B) 8 and 12 hours
A) ±20 B) ±24 C) ±28 D) ±30 E) ±32 C) 11 and 15 hours D) 9 and 13 hours
E) 10 and 14 hours

16. The roots of the equation x3 – 3x + 7 = 0 are p,


12. x – 5 = x – 3 is given. Find the value of x. q, and r. What is the product (p + 1)(q + 1)(r + 1)?

A) 5 B) –7 C) 1 D) 7 E) –4 A) –8 B) –9 C) 7 D) 8 E) 9

102 Algebra 8
A. CARTESIAN PRODUCT AND ANALYTIC PLANE
1. Ordered Pairs
Everyone knows that in a football match different scores, like 3 – 1 and 1 – 3, have a different
meaning. When the score is written as 3 – 1, it is clear that the home side is the winner. But
when it is written as 1 – 3, we understand that the away side is the winner. A similar approach
is also valid for ordered pairs (3, 1) and (1, 3), that is, they don't have the same meaning.

Definition ordered pair


Given that a and b are any two elements, (a, b) is called an ordered pair where a is the first
component and b is the second component.
For example, below we have ordered pairs where components are numbers, names, colors, etc.:
ordered pair first component second component
(28, December) 28 December
(René, Descartes) René Descartes
(–9, 1) –9 1
(black, white) black white
(170 cm, 70 kg) 170 cm 70 kg
(Descartes, René) Descartes René
Note that the pairs (René, Descartes) and (Descartes, René) are not the same since their
components are in different order.

Note
The ordered pair (a, b) is not the same as (b, a) since they are written in different orders.

What is the difference between the two clocks above?

104 Algebra 8
EXAMPLE 1 Give three examples for the ordered pair (x, y) supporting the equation 2x – y = 4.

Solution Let x = 0, then y = –4.


Let x = 5, then y = 6.
Let x = –1.5, then y = –7.
So three such ordered pairs may be (0, –4), (5, 6), (–1.5, –7).
Here, note that we can find an infinite amount of such ordered pairs and although (0, –4) is
an answer, (–4, 0) is not!

EQUALITY OF ORDERED PAIRS

Two ordered pairs are equal if, and only if, corresponding components are equal to each
other. That is, (a, b) = (c, d) if, and only if, a = c and b = d.

EXAMPLE 2 Given that (2x – 1, 7) = (3, 3y – 8), find x and y.

Solution (2x – 1, 7) = (3, 3y – 8)


2x – 1 = 3 and 7 = 3y – 8
x = 2 and y = 5

Note
(x1, x2, x3) is named as an ordered triple.
(x1, x2, x3, x4) is named as an ordered quadruple.
(x1, x2, ..., xn) is named as an ordered n-tuple.

Check Yourself 1
1. How many different ordered pairs are given below?
(5 hours, 20 minutes) (28, December, 2003) (20 minutes, 5 hours) (Italy, Rome)
2. Given that (6, 2x – y) = (x + y, –3), find x and y.
3. Complete the following ordered pairs (x, y) so they support y + 1 = 3x.
(2, ?) (?, 0) (0, ?)
Answers
1. 3; (28, December, 2003) is an ordered triple! 2. 1; 5 3. (2, 5), (1/3, 0), (0, –1)

Functions 105
2. Set Notation
A set is a collection of objects. Each one of these objects that
form a set is called an element of that set.
For example, the months of a year form a set with 12
elements. All even numbers form another set with an
infinite amount of elements. All dogs with seven legs is
another set with no elements. Think about the students in
your class. Is it a set? How many elements does it have?

set of strawberries!

Notation
1. We usually name sets with capital letters like A, B, C, etc.
2. If a1, a2, a3, ..., an are elements of a set A, we list all elements of this set as
A = {a1, a2, a3, ..., an} and denote the number of elements by n(A).
3. The symbol  means “is an element of ”. The symbol  means “is not an element of ”.
For example if A = {3, 5, 10}, then 3  A and 4  A.
4. The symbol  means “intersection” and the symbol  means “union”.
For example if A = {1, 2, 3} and B = {2, 5}, then A  B = {1, 2, 3, 5} since the union
is the set of all elements that are either in A or B, and A  B = {2} since the intersection
is the set of all elements that are both in A and B.
5. To denote an empty set, that is a set with no elements, we use or { }.
6. The symbol  means “is a subset of ”. For example if B = {–2, –1, 3, 7, 10} and
C = {–1, 3, 10}, then C  B since each element in set C is also element of set B.

EXAMPLE 3 Given that A = {1, 4, 5, 7} and B = {all odd numbers between 2 and 8},
a. find n(A) and n(B) .
b. find A  B and A  B.
c. is it correct that 9  A  B?
d. is it correct that A  B = ?
Solution We know that A = {1, 4, 5, 7}. If we list the elements of B, we get B = {3, 5, 7}.
a. n(A) = 4 and n(B) = 3.
b. A  B = {1, 3, 4, 5, 7} and A  B = {5, 7}.
c. Clearly 9 is not an element of A  B. So 9  A  B is correct.
d. Since n(A  B) = 2, A  B =  is incorrect.

106 Algebra 8
Note
Certain letters are reserved for important number sets.
These are , , , and :
 is the set of real numbers.
      
 is the set of rational numbers.
 is the set of integers.
 is the set of natural numbers.
Note that all these sets contain an infinite amount of
elements, so it is impossible to list them.

Notation
A = [a, b] denotes the set of all real numbers between a and b making it
[a, b]
inclusive.
a b
A = (a, b) denotes the set of all real numbers between a and b making it
(a, b)
exclusive.
a b
A = [a, b) denotes the set of all real numbers between a and b, where b is
[a, b)
excluded.
a b
A = (a, b] denotes the set of all real numbers between a and b, where a is
(a, b]
excluded.
a b
Instead of A = [a, b] we can also use the notation A = {all x   such that
a  x  b} or A = {x  | a  x  b}.
Similar notation can be used for the other sets described above.

EXAMPLE 4 Given that A = [–2, 3) and B = (0, 4), find A  B and A  B.

Solution

A A

-2 0 3 -2 0 3

B B

0 4 0 4

AÈB AÇB

-2 0 4 0 3

Here we cannot list all of the elements of the given sets since they contain an infinite amount
of elements. A  B contains elements that are in A or in B so A  B = [–2, 4). A  B
contains elements that are both in A and B so A  B = (0, 3).

Functions 107
Check Yourself 2
1. Given that A = {1, 2, 5, 9, 12} and B = {all x   that are less than 14 and divisible by 3}
find n(B), A  B, A  B.
2. Given that A = [–5, 3] and B = (2, 4], find A  B and A  B.
Answers
1. 4, {1, 2, 3, 5, 6, 9, 12}, {9, 12} 2. [-5, 4], (2, 3]

3. Cartesian Product
Definition cartesian product
Let A and B be two non-empty sets. The set of all ordered pairs, whose first component is from
A and whose second component is from B, is called the cartesian product of A and B and is
denoted by A  B.

For example, if A = {1, 2, 3} and B = {x, y}, then


A  B = {(1, x), (1, y), (2, x), (2, y), (3, x), (3, y)} and

A  B is read “A cross B”, B  A = {(x, 1), (x, 2), (x, 3), (y, 1), (y, 2), (y, 3)}.
not “A times B”. Here, A  B and B  A are clearly different sets. The only common property between them is
that they have the same number of elements, which is equal to the product of number of
elements of A and B. Note that A  B = B  A only when A and B are equal sets.

NUMBER OF ELEMENTS OF A CARTESIAN PRODUCT

Given two sets A and B, n(A  B) = n(B  A) = n(A)  n(B).

EXAMPLE 5 Given that M = {a, b, c} and N = {1, 2, 3, 4}, find the number of elements of M  N and list
them.
Solution n(M  N) = n(M)  n(N) = 3  4 = 12
M  N = {(a, 1), (a, 2), (a, 3), (a, 4), (b, 1), (b, 2), (b, 3), (b, 4), (c, 1), (c, 2), (c, 3), (c, 4)}

Listing the elements of a cartesian product, as shown above, is called the list method.
The term Cartesian is used in the
name of the French mathematician Sometimes a set may have an infinite amount of elements which will result in an endless list.
and philosopher René Descartes.
For example, think about making a list for A  B where A = {yes, no} and B = {all even
numbers}.
Even though there are a finite number of elements, we may want to see the whole
representation rather than the list. For these cases the coordinate method is the most
efficient way to show the representation. To represent all the elements of a cartesian product
by the coordinate method we choose the horizontal axis for the first component and the
vertical axis for the second component.

108 Algebra 8
EXAMPLE 6 Given that C = {black, white} and D = {1, 2, 3, 4, 5, 6}, represent C  D by:

a. the list method. b. the coordinate method.

Solution a. By the list method, we have


C  D = {(black, 1), (black, 2), (black, 3), (black, 4), (black, 5), (black, 6), (white, 1),
(white, 2), (white, 3), (white, 4), (white, 5), (white, 6)}.
b. By the coordinate method, we have

D
6
5
4
3
2
1
C
black
white

CD

Here note that the horizontal axis denotes


A chessboard is a cartesian product of
elements of C and the vertical axis denotes
{A, B, C, D, E, F, G, H} and {1, 2, 3, 4, 5, 6, 7, 8}.
elements of D. Since n(C  D) = 12, we
have 12 points plotted.

EXAMPLE 7 Given that A = {1, 2, 3} and B = {all x   such that 3  x  5}, represent A  B and B  A.

Solution Since A  B and B  A contain an infinite amount of elements, using the list method is not
possible. But by coordinate method we can represent all of the elements as shown below:
B
5 A

3 3
2
1
A B
1 2 3 3 5
A´B B´A

How many points are plotted on each plane?

Functions 109
EXAMPLE 8 Given that A = (4, 7) and B = [1, 3] represent A  B.

Solution By the coordinate method we have


B
3

1
A
4 7
A´B

Note that the vertical boundaries of the rectangle are not included in A  B.

4. Analytic Plane
To graph the cartesian products whose elements are ordered pairs of real numbers, we need
René Descartes
(1596-1650) a coordinate system. The rectangular or cartesian coordinate system consists of a horizontal
number line, the x-aaxis or the abscissa, which we label as x, and a vertical number line, the
y-aaxis or the ordinate, which we label as y.
The plane on which such a coordinate system is constructed is called an analytic plane or
xy-pplane. Axes divide the analytic plane into four parts which are called quadrants. The
French mathematician and intersection point of axes is called the origin.
philosopher, René Descartes
believed science and mathematics
could explain and predict events ILLUSTRATION OF ANALYTIC PLANE
in the physical world. Descartes
developed the Cartesian coordinate
system for graphing equations y
and geometric shapes. Modern
maps use a grid system that can
be traced back to Cartesian
2 nd 4
quadrant 3 1quadrant
st

graphing techniques. It is said 2 (1,2)


(although the story is probably a origin 1
myth) that Descartes came up
with the idea for his coordinate -4 -3 -2 -1 -1 1 2 3 4 x
system while lying in bed and
-2
watching a fly crawl on the
ceiling of his room. 3 rd -3
quadrant -4 4 quadrant
th

Just as every real number corresponds to a point on the number line, every pair of real
numbers corresponds to a point on the analytic plane. For example the pair (1, 2)
corresponds to the point that lies one unit to the right of the origin and two units up. The
first component, which lays on the x-axis, is 1. The second component, which lays on the
y-axis, is 2. The point where the components meet is called the plot. Locating this point on
an analytic plane is named plotting or graphing the plot.

110 Algebra 8
EXAMPLE 9 Plot the pairs (0, –2), (3, 1), (–4, 2), (1, 0) on an analytic plane.
y
Solution Note that the first component is on the x-axis and the 4
second component lays on the y-axis. 3
(-4,2) 2
1 (3,1)
(1,0)
-4 -3 -2 -1 -1 1 2 3 4 x
-2 (0,-2)
-3
-4

The coordinate system that maps use stems from the Cartesian coordinate system.

Check Yourself 3
1. Given that A = {1, 2} and B = {a, b, c, d}, represent A  B and B  A by the list method
and the coordinate method.
2. Given that M = {all x   such that 1 < x  5} and N = {all x   such that 2  x  7},
express M  N by the coordinate method. Is it possible to use the list method?
3. Plot the pairs (1, –2), (0, 1), (3, 3), (–4, –1) on an analytic plane.

Answers
B
1. {(1, a), (1, b), (1, c), (1, d), (2, a), (2, b), (2, c), (2, d)} d
A
c
{(a, 1), (a, 2), (b, 1), (b, 2), (c, 1), (c, 2), (d, 1), (d, 2)} b 2
a 1
A B
1 2 a b c d
2. N 3. y
3
7

1
-4 1 x

2 3
-1
M -2
1 5 ; no

Functions 111
B. RELATIONS
1. Representation of a Relation
If A and B are two non-empty sets, then every non-empty subset of B
the cartesian product A  B is a relation defined from A to B. Canada
Russia
For example, if A = {Ottowa, Beijing, Moscow, Ankara} and
Turkey
B = {China, Turkey, Russia, Canada} we can describe a relation, China
say , from A to B in the following way: A

Ankara
Moscow
Beijing
Ottowa
 = {all pairs (x, y) such that x is the capital of y} and can be
listed as follows: {(Ottowa, Canada), (Beijing, China), (Moscow,
Above all elements of A  B
Russia), (Ankara, Turkey)}. Here  is a subset of A  B. Note that is illustrated. Red ones are
elements of relation .
we can choose any subset of A  B to find another relation. Since
each subset is a collection of ordered pairs, a relation can simply be
defined as the collection of ordered pairs.

Definition relation
A relation is a set of ordered pairs. The set of all the first components is called the domain
and the set of all the second components is called the range of the relation.

For example, {(0, 5), (1, –3), (–1, –3), (2, 0), (–3, 5)} is a relation since it is a set of ordered
pairs. Its domain is {–3, –1, 0, 1, 2} and its range is {–3, 0, 5}.
The relation above is represented by a list. This relation can also be represented by a table,
by a map, or by a graph as seen below:

2nd
1st component 2nd component 1st component 2nd component component
5
0 5 ·0
1 –3 ·1 ·5
· -1 · -3
–1 –3 -1 1
·2 ·0 -3
2 0
2 1st
· -3 component
–3 5
-3
representation by a table representation by a map representation by a graph

112 Algebra 8
EXAMPLE 10 month season
January Winter
September Fall
April Spring
March Spring
June Summer
October Fall
February Winter
May Spring

Given the following relations represented


by the table above, answer the following:

a. How is the first component related with the second component?


b. Represent it by a list.
c. Represent it by a map.
d. Represent it by a graph.
e. Find the domain.
f. Find the range.

Solution Before we find the solution, to get rid of confusion, let us assume that we are on the
northern hemisphere since seasons are different on the southern hemisphere of the earth.

a. The first component is the month, and the second component is the season which
contains the given month.
b. {(January, Winter), (September, Fall), (April, Spring), (March, Spring), (June, Summer),
(October, Fall), (February, Winter), (May, Spring)}
c. month season d. season

· January Summer
· Winter
· September Spring
· April Fall
· Fall
· March Winter
· June · Spring month
April
March
June

May
January
September

October
February

· October
· February · Summer
· May

e. The domain is {January, September, April, March, June, October, February, May}.

f. The range is {Winter, Fall, Spring, Summer}.

Functions 113
EXAMPLE 11 Given the domain {–2, –1, 0, 1, 2} and the relation containing ordered pairs of the form
(x, y) such that x < y  2 and y  ,
a. represent the relation by a list.
b. represent the relation by a graph.
c. find the range.

Solution a. Choose x = –2 from the domain. Related y values should support –2 < y  2 and y  .
So for x = –2, we have y = –1, 0, 1, 2. That gives us the elements (–2, –1), (–2, 0),
(–2, 1) and (–2, 2). We proceed in the same way for x = –1, 0, 1, 2 and get the final list
as {(–2, –1), (–2, 0), (–2, 1), (–2, 2), (–1, 0), (–1, 1), (–1, 2), (0, 1), (0, 2), (1, 2)}.
b. y
2
1

-2 -1 -1 1 x

c. The range is {–1, 0, 1, 2}.


How can we represent this relation by a map?

EXAMPLE 12 Given the domain [–1, 2] and the relation containing ordered pairs of the form (x, y) such
that y = x + 1,

a. if possible, represent the relation by a list.


b. represent the relation by a graph.
c. find the range.

Solution a. Since the domain contains an infinite amount of y


elements it is impossible to list all of the elements of that
relation.
To draw a line, 3
1. select two numbers b. We just sketch the line y = x + 1, but note that we take
for x and find the the part of the line whose x values are between –1 and 2
y value for each.
2. plot those two points of (remember the domain!). -1 2 x
the form (x, y).
3. connect them with a c. As it is seen on the graph y can take any value between 0
straight line and extend and 3, inclusive, therefore the range is [0, 3].
the line on each side.

Note
If the domain of a relation contains an infinite amount of elements we cannot represent it by
a list, table or map.

114 Algebra 8
EXAMPLE 13 The relation between blood types is represented
by the map on the right. 0

a. Explain the relation verbally.


b. Represent the relation by a list. A
B
c. Find the domain.
d. Find the range. AB

Solution a. All blood types can give blood to the same type. Additionaly type 0 can give blood to any
type and type AB can get blood from any type.
b. {(0, 0), (0, A), (0, B), (0, AB), (A, A), (A, AB), (B, B), (B, AB), (AB, AB)}
c. The domain is {0, A, B, AB}.
d. The range is also {0, A, B, AB}.

Check Yourself 4
1. Represent the relation {(Monday, sunny), (Tuesday, rainy), (Wednesday, sunny),
(Thursday, cloudy), (Friday, cloudy)} by mapping and graphing.
2. Represent the relation having ordered pairs of the form (x, y) such that y2 = x in the
domain {0, 1, 4} by listing and graphing.
3. Given the domain [–3, 4] and the relation containing ordered pairs of the form (x, y) such
that y = –2x + 3 represent the relation by graphing and find its range.
Answers
1.
· Monday · sunny cloudy
· Tuesday rainy
· Wednesday · rainy sunny
· Thursday
· cloudy
Monday
Tuesday

Thursday
Wednesday

Friday

· Friday

2. {(0, 0), (1, –1), (1, 1), (4, –2), (4, 2)} 3. y

y 9
2
1
x
1 2 3 4
-1 4
-2 -3 x

-5 ; [–5, 9]

Functions 115
2. Inverse of a Relation
As we learned previously, a relation is a set
of ordered pairs. The inverse of a set of
ordered pairs is obtained by interchanging
the places of the first and the second
components. Since the first and the
second components are interchanged
when we are finding the inverse of a
relation, the domain and the range are
also interchanged.

EXAMPLE 14 Given the relation {(–2, 3), (0, 1), (–4, 0), (2, 3)} containing ordered pairs of the form (x, y),

a. list the inverse relation.


b. find the domain and range of the inverse relation.
c. graph both of these relations.

Solution a. We simply change places of components to find {(3, –2), (1, 0), (0, –4), (3, 2)}
b. The domain of an inverse relation is {0, 1, 3} and its range is {–4, –2, 0, 2}
c. Let us plot the set of points {(–2, 3), (0, 1), (–4, 0), (2, 3)} and {(3, –2), (1, 0), (0, –4),
(3, 2)} on the same analytic plane using different colors:
y
4 y=x
3
2
1

-4 -3 -2 -1 -1 1 2 3 4 x
-2
-3
-4

Note that each point and its inverse is symmetric with respect to the line y = x.

GRAPH OF INVERSE OF A RELATION

The graph of inverse of a relation is symmetric to the original graph with respect to the
line y = x.

116 Algebra 8
EXAMPLE 15 Plot the graph of the inverse of the relation whose graph is given below.
y

Solution Since the graph of inverse of a relation is symmetry of the original graph with respect to the
line y = x, we first draw the axis of symmetry, that is y = x, and then the reflection of the
relation. So we get the following graph:

y y=x

Note that the domain and the


range of the relation and its
inverse contain an infinite
amount of elements.

EXAMPLE 16 Given the relation containing ordered pairs of the form (x, y) such that y = x + 2, find the
rule for the inverse relation.

Solution If the relation was given by a list it would be enough to interchange x and y. Note that here
it is impossible to list the elements of the relation since it contains an infinite amount of
elements. However we can apply the same technique directly on the given rule for relation.
The rule for relation is y = x + 2. If we interchange x and y we have x = y + 2. Leaving y
on one side alone we get the rule for the inverse relation as y = x – 2.

Note
If the rule for a relation is known, then the rule for its inverse is obtained by interchanging
x and y.

Functions 117
Check Yourself 5
1. Given the relation {(1, 2), (–4, 7), (1, 0)}, find the inverse relation, its domain and the
range.
2. Plot the graph of the inverse of the relation whose graph is given below.
y

3. Given the relation containing ordered pairs of the form (x, y) such that y = 3x – 1, find
the rule for the inverse relation.
Answers

2. y x +1
1. {(2, 1), (7, –4), (0, 1)}; {0, 2, 7}; {–4, 1} 3. y =
3

118 Algebra 8
EXERCISES 4 .1
A. Cartesian Product and Analytic 5. For the given sets A and B list A  A, B  B, A  B,
Plane B  A.
1. Find the unknowns in the following ordered pairs. a. A = {all x  + such that x2 < 15},
a. (3x + 5, 4) = (2, y) B = {all x   such that x2 – 1 = 0}
b. (x – 2y, 3x + y) = (1, 3) b. A = {months of winter in northern hemisphere},
B = {2, 5, 6}

2. Let A = {2, 3, 4}, B = {3, 7, 8, 9},


6. For the given sets A and B, plot A  B on an
C = {1, 3, 5, 9, 10}.
analytic plane.
a. Find A  B. b. Find n(A  C).
a. A = {1, 2, 3}, B = {4, 6}
c. Find B  C. d. Find A  B  C.
b. A = {all x   such that 1 < x < 4} and
e. Find A  B  C. f. Find n((A  B)  C).
B = {2, 3, 4}
c. A = {all x   such that –2 < x  3} and
B = {all x   such that 2  x  4}

3. For the given sets A and B find A  B and A  B.


a. A = {all x   such that –3 < x  1},
B = {all x   such that –1 < x < 2}
B. Relations
b. A = {all x   such that x > 2},
7. List the following relations which have ordered
B = {all x   such that –3  x < 5} pairs in the form (x, y).

a. y = x2, the domain is {0, 1, 2, 3}.


b. 2x – 3y = 1, the domain is –3 < x < 4 such
that x  .
4. Using the following find A and B.
a. A  B = {(1, a), (1, b), (1, c), (2, a), (2, b), (2, c)}
b. B
4 8. Represent the following relations having ordered
pairs of the form (x, y) by mapping.
2
a. {(1, b), (2, d), (3, c), (5, e)}
A
1 2 b. x – y = 1, the domain is {1, 2, 3, 4, 5}.

Functions 119
9. Represent the following relations having ordered Mixed Problems
pairs of the form (x, y) by a graph.
12. Plot all pairs (x, y) that satisfy the equation
a. {(1, 3), (2, 1), (1, 2), (2, 3)}
y = x – 1 on an analytic plane.
b. y = 2x + 1, the domain is [0, 3].
c. y = x, the domain is .
d. y  x – 1, the domain is .

13. In this figure a relation is


a
represented by mapping.
10. Find the inverse relation, domain and range of List the elements of the
b d
the inverse relation for the following relations inverse relation.
having ordered pairs in the form (x, y). c

a. {(winter, December), (fall, September),


(summer, June), (spring, March)}
b. y = 2x – 3, the domain is [–20, 34]

c. y = x2 + 2, the domain is [–3, 3]


14. How many elements does A  B have if A denotes
days in a week and B denotes months in a year?

11. Graph the inverse relation for the following:


a. y

(0,2) 15. Prove that n(A  B) = n(A) + n(B) – n(A  B).


(-3,0) x

b. y
(2,2) 16. Given that A = {1, 2, 3}, B = {a, b}, and

C = {blue, red}, list A  B  C. How can you show
(0,0) (4,0) x A  B  C by the coordinate method?
(2,-2)

c. y
(2,4)
(-1,4)
17. Given that A = {–1, 0, 3}, B = [–5, 5], and

C = [1, 4), plot the difference of A  B from
(0,0) (3,0) x A  C.

120 Algebra 8
Many board games use principles of the cartesian product and the 10
9
cartesian coordinate system. The Game of Battleship is a navy game 8
where two players try to sink each other's hidden ships. On a piece of 76
paper two players draw 10 × 10 tables, the first of which represent 54
the player's ocean and the second which represents their opponent's 3
2
ocean. Horizontal grids are named by letters and vertical grids are 1
named by numbers. Each player should place five ships in his own A B C DE F GH I J
C3 = “HIT”
hidden ocean as follows: “Carrier”(5 squares), “Battleship”(4 squares), G7 = “MISS”

“Cruiser”(3 squares), “Submarine”(2 squares), and “Destroyer”(1 square). Ships may be


placed in any horizontal or vertical position - but not diagonally. Taking turns, players call
out their “shots” by telling an ordered pair denoting a location in their opponent’s ocean
and attempting to “hit” the opponent’s ships in order to sink them. When a shot is
called, the opponent immediately tells whether if it is a “hit” or “miss”. If the shot is a
“hit”, the opponent tells the player what kind of ship was hit. Players note shots of their
opponent in their own view and their shots in the enemy’s view to decide a further
strategy of play. Strategy and some luck must be combined to be the first to locate and
sink all 5 of your opponent’s ships in order to win the game.

The salvo game version is recommended for more experienced players. It differs mainly
in how many shots are taken in a turn by each player. Each player at the start takes a
salvo of 5 shots in his turn. Whenever a player has had one of his ships sunk, he loses
one shot for his next salvo. As his ships are removed from the game, the shots for each
salvo are reduced.
The advanced salvo game offers a challenge for the expert player. It is played as salvo,
except after a salvo of shots is called, the opponent simply announces how many hits
were made - but not where or on what ships.
Try to remember the board games you know. Do they have any relation with the cartesian
coordinate
Functions system? What can be the components of an ordered pair? 121
A. DEFINING A FUNCTION
1. Definition
There are many variables around us. Some of them are closely related to each other and some
of them are not. For instance, the number of students in a classroom and the quantity of
oxygen in the same classroom are closely related. But the number of students in a classroom
and the temperature outside are not related at all. Functions are used to show this relation
between variables. Using that relation we can estimate the results for possible cases.
Below, the numbers on the right are related to the numbers on the left:
11
24
39
4  16
You can easily guess the rule that relates the number on the left to the number on the right.
It is: “square the number”. So the relation converts a number x to another number x2. We
can symbolize this as:
x  x2
So,
If x = 10, “the square of the number” is 100 (1st sentence)
If x = –4, “the square of the number” is 16 (2nd sentence)
If x = 0.5, “the square of the number” is 0.25 (3rd sentence)
Obviously, a way of expressing the result of the rule as shown above is not very practical or
mathematical. Writing it in the form 10  100 is also not good since it is not clear which
rule we are using. That is, the meaning of “” can be confusing. (it may be “add 90 to the
number” or “multiply the number by 10” as well). In order to talk about this rule, in our case
“square the number”, we should name it as f. When we apply this rule to x, we get x2. So f
is the rule that converts x into x2. Symbolically,
f(x) = x2.
Let us rewrite the above sentences once more:
f(10) = 100 (1st sentence)
f(–4) = 16 (2nd sentence)
f(0.5) = 0.25 (3rd sentence)

122 Algebra 8
We can write each of the numbers above on the “left” and “right” as ordered pairs like
(10, 100), (–4, 16), (0.5, 0.25). Note that for each number on the “left”, which is the first
component, there corresponds just one number on the “right”, which is the second
component. We know that the mathematical relation is a set of ordered pairs. Whenever the
first component of an ordered pair is associated with exactly one second component we name
that relation a function.

Definition function
A function f is a rule that assigns to each element x in set A exactly one element y or f(x)
in set B. Set A is called the domain and set B is called the range of the function f. We name
x as the independent variable or the argument, and y as the dependent variable since the
value of y depends on x.

A function can also be


Euler's abstract
definition of function
thought of as a set of B = {(1, 2), (2, 6), (3, 9)} FUNCTION
in 1755 in his ordered pairs whose
Institutiones Calculi first components are
Differentialis: Different x-values
all different. The set of
If some quantities so depend on
other quantities that if the latter all the first components
are changed the former undergo of the ordered pairs is C = {(1, 3), (1, 4), (4, 6)} NOT A FUNCTION
change, then the former quantities
are called functions of the latter. the domain of the
This denomination is of broadest
function. The set of all Same x-values
nature and compromises every
method by means of which one the second components
quantity could be determined by
others. If, therefore, x denotes a of the ordered pairs is the range of the function. Consider the set, A = {(cat, dog), (chicken,
variable quantity, then all duck), (cat, mouse)}. The set A would not be a function because the first component, cat, is
quantities which depend on x in
any way or are determined by it paired with 2 different second components. Consider the set B = {(1, 2), (2, 6), (3, 9)}. This
are called functions of it. is a function since each first component has only one second component paired with it. The
domain of B is the set {1, 2, 3} and the range of B is the set {2, 6, 9}. C = {(1, 3), (1, 4), (4, 6)}
would not be a function since 1 has two components paired with it, 3 and 4.

EXAMPLE 17 State whether the following relations define a function or not.

a. {(0, 2), (0, 3), (1, 6), (2, 4), (3, 5)}
b. {(–3, 1), (–1, –1), (0, 1), (1, 3), (2, –2)}
c. A relation having ordered pairs of the form (radius, area of circle)
d. A relation having ordered pairs of the form (name, surname)
e. {(1, z), (2, d), (4, f)} with the domain {1, 2, 3, 4}

Functions 123
Solution a. In order to have a function for each value in the domain we ·2
·0
should have exactly one element assigned in the range. ·1
·3
Since 0 is assigned both to 2 and 3, this is not a function. ·4
·2
·5
·3
·6
b. The domain is {–3, –1, 0, 1, 2}. Since there is only one value
assigned for each value in the domain, this is a · -3 · -2

function. Note that in the image on the right –3 and 0 are · -1 · -1


·0
both assigned to 1, but this does not prevent it from being a ·1
·1
function. The elements from the domain may be assigned to
·2 ·3
the same value from the range. The important point is that
each element of the domain must not have more than one different value from the range.
c. For each given radius there is exactly one possible circle area. This relation is a function.
d. Two people with the same name can have different surnames. This relation is not a function.
e. As in the previous examples if nothing else is stated as the domain, the set of all the first
components is the domain. For this example it would be obvious to think that the domain
is {1, 2, 4}. This relation is a function if we didn't see the phrase “with domain {1, 2, 3, 4}”.
But the definition of a function states that every element in a domain must be assigned
by exactly one element. We can see that the element “3” from the domain remains
unassigned. Therefore this relation is not a function.

Note
Any function is a relation but any relation is not always a function.

To each person there corresponds just


one birth date.
Have you ever heard of a person without
a birth date? Or a person with two birth
dates?

To each satellite there corresponds just


one planet.
Have you ever heard of a satellite without
124
a planet? Or a satellite with two planets? Algebra 8
Notation
If the letter f represents a function, then the notation f(x) means “apply the rule f to the
number x” and we read f(x) as “f of x”.

EXAMPLE 18 Given the function with the rule “multiply a number by 3 and add 5”, formulize it and find
its value when the number is
a. –3 b. –1 c. 0 d. 2
Solution Clearly, we have a long rule for a mathematician. If we denote the number by x and the rule
by f, then we have f(x) = 3x + 5 and we need to find f(–3), f(–1), f(0), f(2).
a. Substituting –3 in the place of x, f(–3) = 3  (–3) + 5 = –4.
b. Substituting –1 in the place of x, f(–1) = 3  (–1) + 5 = 2.
f(x) does not mean
c. Substituting 0 in the place of x, f(0) = 3  0 + 5 = 5.
“f times x”.
d. Substituting 2 in the place of x, f(2) = 3  2 + 5 = 11.

EXAMPLE 19 Given the function f(x) = |2x – 3|,

a. find f(1). b. find x, if f(x) = 0.


Solution a. f(1) = |2  1 – 3| = |–1| = 1 “| |” stands for absolute value.
Absolute value of a number is
b. We are looking for x value(s) for which f(x) = 0.  itself if it is nonnegative,

That means |2x – 3| = 0.  its negative if it is negative.


e.g. |25| = 25, |–0.7| = 0.7
Solving this equation we get that x = 1.5.

A common way to consider functions is through the idea of


a function machine. Imagine this machine as having an input,
where items are entered, and an output, where results are
obtained. If a number is dropped into the input, a function
machine will output a single value.

Functions 125
EXAMPLE 20 Given the function f(x) = x2 – 3x, find f(–2) + f(4).

Solution To find f(–2) + f(4) we use the formula f(x) = x2 – 3x twice by substituting –2 and 4 in the
place of x.
f(–2) = (–2)2 – 3(–2) = 10
In general,
f(4) = 42 – 3  4 = 4
f(a) + f(b)  f(a + b).
So, f(–2) + f(4) = 10 + 4 = 14.

Notation
In mathematics, there are different notations used for functions. The expressions below are
the most frequent notations and have the same meaning:
f(x) = x2
y = x2
Since this is just a matter of notation we may also use letters like g or h to denote a function
and t, s, u, v to denote its argument. For the previous example if we had chosen g as
the function name and u as the variable, then the formula would have been g(u) = u2 – 3u
which would definitely give us the same result.

EXAMPLE 21 Given the function g( x) =


2 x +1
x2  1
, find g(2), g(2a), g(a + 3), g(–x), g(anything).

2  2 +1 5
Solution Substituting 2 in the place of x, g( 2) = = .
22  1 3
2  2a +1 4a +1
Substituting 2a in the place of x, g( 2a) = = .
( 2a)2  1 4a2  1

2  ( a + 3) + 1 2 a +7
Substituting a + 3 in the place of x, g( a + 3)  = 2 .
( a + 3)2  1 a + 6a + 8
2  (  x)+1 2 x+1
Substituting –x in the place of x, g(  x) = = 2 .
(  x)2  1 x 1
2  ( anythi ng ) +1
Substituting anything in the place of x, g( anything ) = .
(anything )2  1

Notation
f : A  B means function has domain A and range B.
This notation is rarely used. When it is not used, it means that the domain is all possible real
numbers for which the function is defined and the range is all possible corresponding values.

EXAMPLE 22 Given the function g :    such that g(u) = u + 5, find g(0), g(–2), g(–0.5).

126 Algebra 8
Solution Substituting 0 in the place of u, g(0) = 0 + 5 = 5.
Substituting –2 in the place of u, g(–2) = –2 + 5 = 3.
We cannot substitute –0.5 in the place of u since the function has integers as its domain
(note that –0.5  ), so g(–0.5) is undefined.
What would you say about g(–0.5) if the domain of function was ?

Expressing a function with the help of one formula may not always be possible. Sometimes
we need two, three or even more formulae to describe a function. We face these kind of
situations frequently and name such functions as piecewise defined functions.

Note
A piecewise function is defined so that for any value in the domain there corresponds
exactly one formula.

EXAMPLE 23 Given that f(2x + 1) = x + 7, find f(3).

Solution We cannot substitute 3 in the place of x. If we do so, we get f(2  3 + 1) = 3 + 7 which gives
f(7) = 10. But we need f(3) not f(7)! To find it we will look for the value of x which will make
the inside of the expression f(2x + 1) equal to 3. Solving 2x + 1 = 3, we find x = 1.
Substituting 1 in the place of x, we get f(2  1 + 1) = 1 + 7 which gives f(3) = 8.

EXAMPLE 24 1
Given that f ( ) = x  1, find
x
a. f(2). b. f(x).

Solution a. We must use the same method of the previous example. To find f(2) we must find the
1
value of x which will give 2 when put in the place of the expression
. In other words we
x
1 1
must solve = 2, which means x = . Substituting this value in the given formula we
x 2
have
1 1 1
f ( 1 )   1 or f (2)   .
2 2 2
1 1 1 1
b. Let  a, so x  . Rewriting the formula f ( )  x  1 in terms of a we have f ( a)   1.
x a x a
Since this is just a matter of notation instead of letter “a” we can choose any other letter
1
we like. Since we must find f(x), let us use “x” in place of “a”. So f ( x)   1.
x
Note that since we now have the formula for f(x) we are able to find any value of f
1
without repeating the procedure in part a. Try to find f(2) directly using f ( x)   1.
x
What will you get?

Functions 127
Check Yourself 6
1. Is the relation having ordered pairs in the form of (month, its season) a function? What
about the relation with ordered pairs in the form of (season, a month of it)?
2. Given the function f(x) = x2 – 5x, find f(0), f(3), f(–a), f(x + 1), f(a) + f(b).
 x2  x if x > 0
3. Given the function f ( x) =  , find f(–2), f(0), f(5).
4 if x  0
4. Given that f(2x – 3) = x2 – x, find f(1).
Answers
1. yes, no 2. 0, –6, a2 + 5a, x2 – 3x – 4, a2 – 5a + b2 – 5b 3. 4, 4, 20 4. 2

2. Applied Problems
Functions are widely used for modelling real life data. In modelling functions, we must be
able to translate the verbal description into mathematical language. We do this by assigning
symbols to represent the independent and dependent variables and then finding the function
that relates these variables. Once we model a real life situation, we are able to estimate the
result for any specific value.

EXAMPLE 25 The area of a rectangle is 40 m2.


a. Express its perimeter P as a function of its width.
b. Using the required function find the perimeter of the rectangle if its width is 5 meters.
40
Solution a. Let x m be the width of the rectangle. Since its area is 40 m2 , its length is m. We
x
know that the perimeter of a rectangle is twice the sum of its length and width. So the
40
b perimeter can be expressed as P( x) = 2( x + ).
x
a
Given a rectangle with 40
sidelength a and b b. Simply we need to find P(5)= 2(5+ ) = 26. The perimeter of the rectangle is 26
5
Perimeter = 2(a + b)
meters.
Area = a  b

EXAMPLE 26 On a certain route an airline carries


8000 passengers per month, each
paying $50. The airline wants to
increase the fare. However, the
market research estimates that for
each $1 increase in fare, it will lose
100 passengers. Express the airline's
monthly income as a function of
increase in fare.

128 Algebra 8
Solution Let x be the increase in fare. Then
the new fare = 50 + x,
the number of passengers = 8000 – 100x (each $1 increase is a decrease of 100 in passengers).
So if they increase the fare by x, the new monthly income will be
(new fare) × (number of passengers) = (50 + x)(8000 – 100x)
Let f be the income function.
So f(x)= (50 + x)(8000 – 100x) or f(x) = –100x2 + 3000x + 400000.

EXAMPLE 27 A telephone company does not charge any


money for calls that lasts at most 15 seconds,
$0.2 per minute for calls that last at most 4
minutes and $0.15 multiplied by unit time after
4th minute. Express the call cost as a function in
terms of minutes.

Solution Let x be the call length in minutes and f(x) be the cost function.
Clearly x > 0.
When x  0.25 (15 seconds = 0.25 minutes), f(x) = 0.
When 0.25 < x  4, f(x) = 0.2x.
When x > 4, besides 0.2 × 4 which is the cost of first 4 minutes, the caller will pay $0.15
multipled by extra time after the 4th minute.
So, f(x) = 0.8 + (x – 4)0.15.
Since we have three different cases, the cost function will be expressed piecewisely:

0 if 0 < x  0.25

f ( x) = 0.2 x if 0.25 < x  4

0.15 x +0.2 if x > 4

Check Yourself 7
1. The area of a circle is found by multiplying the square of its radius by . Write the formula
for the area of the circle function S(r) where r is the radius.
2. A gardener has 140 m of fencing for her rectangular vegetable garden. Express the area
as a function of one side length.
Answers
1. r2 2. –x2 + 70x

Functions 129
3. Graph of a Function
a. Plotting Ordered Pairs
Visualization makes great use to get familiar with a specific function. The best way to
visualize a function is to graph it. The graph of a function f is the collection of ordered pairs
(x, f(x)) or (x, y) such that x is in the domain of the function.

ILLUSTRATION OF THE GRAPH OF A FUNCTION

y = f(x)
(x, f(x))
f(x)

x x

Keep in mind that x is the directed distance from the y-axis, and f(x) is the directed distance
from the x-axis.

EXAMPLE 28 Given the graph of a function on the right, y

f ( 2)+ 2 f (3)
a. find . 1
f (0)
-2 2 3
b. find x, if f(x) = 0. x
-1

y = f(x)

Solution a. In this example we have no formula to use but only the graph. To find the given value we
should first find each of f(–2), f(3) and f(0).
f(–2) means the y-coordinate of the point whose x-coordinate is –2. Clearly the answer is
0. With the same procedure we find that f(3) = –1 and f(0) = 1.
f ( 2) + 2 f (3) 0 + 2  ( –1)
Therefore, =  –2.
f (0) 1
b. To find the values of x for which f(x) = 0, we should find all the points whose
y-coordinate is 0 and then take x-coordinates of those points as an answer. Clearly, there
are two such points (–2,0) and (2,0) so the required x values are –2 and 2.

130 Algebra 8
EXAMPLE 29 Plot the graph of f(x) = x – 2 with the domain {–2, –1, 0, 1, 2, 3}.

Solution In this example the domain contains a finite number of elements. Substituting –2, –1, 0, 1,
2 in the place of x, we get
f(–2) = –4, f(–1) = –3, f(0) = –2, f(1) = –1, f(2) = 0, f(3) = 1
which gives us the ordered pairs (–2, –4), (–1, –3), (0, –2), (1, –1), (2, 0), (3, 1).
So we get the following graph:
y
2 y = f(x)
1

-4 -3 -2 -1 -1 1 2 3 4 x
-2
-3
-4

EXAMPLE 30 Plot the graph of f(x) = x – 2.

Solution Note that the difference of this function from the one in the previous x f(x)
example is that its domain is any real number for which the function is –2 –4
defined. So we must have an infinite amount of points plotted on our –1 –3
graph. Let us evaluate f(x) for some values of x (see the table on the 0 –2
If domain is not given
with the formula, then it right). This will give us the following line as graph: 1 –1
is the set of all possible
real numbers for which y 2 0
the function is defined. 2 y = f(x) 3 1
1

-4 -3 -2 -1 -1 1 2 3 4 x
-2
-3
-4
Graph of f(x) = mx + n is
a line.

Functions 131
b. Intercepts of a Function
If the graph of a function f crosses the x-axis, then the function has an x-intercept. To find
An x-intercept is the first the x-intercepts, we need to find all the x-values that support the equation f(x) = 0. Since
component of the ordered an equation may have more than one solution, the graph of a function can cross the x-axis
pair (a, 0) where a is any
real number. more than once.

Note
The x-intercept(s) of a function are also called the zeros or the roots of the function. A
function can have more than one x-intercept.

x-intercepts are on the x-axis.

EXAMPLE 31 Find the zeros of the following functions:

a. f(x) = 2x + 10 b. g(x) = x2 – 9

Solution a. Zeros of the function support the equation f(x) = 0 which means 2x + 10 = 0. So at
x = –5 function has a zero.
b. Solving g(x) = 0 which means x2 – 9 = 0, we get x = 3 and x = –3.

Similarly, if the graph of a function f crosses the y-axis, then the graph of the function has a
y-intercept. To find the y-intercepts, we need to let x = 0, and then solve for y. Since we are
A y-intercept is the second
component of the ordered talking about a function, f(0) can have at most one value. That means that the graph of a
pair (0, b) where b is any function can cross the y-axis at most once.
real number.
Note that we used the phrase “at most” since it is possible to have no y-intercept if 0 is not
in the domain of function.

132 Algebra 8
Note
A function can have at most one y-intercept.

y-intercepts are on the y-axis.

EXAMPLE 32 Find the y-intercept of the function f ( x) =


x+ 3
x2 +1
.

0+ 3
Solution To find the y-intercept we substitute x = 0. That gives us f (0) = = 3.
02 +1

c. Vertical Line Test for Functions


By definition, a function needs to have at most one y-value
assigned to each x-value. That is, a vertical line can cross
the graph of the function at most once for any x-value.

VERTICAL LINE TEST FOR FUNCTIONS

A set of points in the coordinate plane is the graph of a function if, and only if, no
vertical line crosses the graph at more than one point.

EXAMPLE 33 Which of the following are graphs of functions? Why or why not?
a. y b. y c. y d. y

x x x x

Functions 133
Solution a. As demonstrated below, no vertical line intersects the graph at more than one point. So
the graph belongs to a function.
y

b. Since we can find a vertical line which intersects the graph at more than one point, this
graph does not belong to a function.
y

c. Since we can find a vertical line which intersects the graph at more than one point, this
graph does not belong to a function.
y

d. Since no vertical line intersects the graph at more than one point, this graph belongs to a
function.
y

134 Algebra 8
EXAMPLE 34 Plot the graph of a function whose x-intercepts are –3, 2, 4 and whose y-intercept is 3 if the
graph passes through the point (1, 1.5).

Solution Our graph should cross the x-axis at –3, 2, 4 and the y-axis y

at 3. Moreover, no vertical line should intersect the graph at y=f(x)


3
more than one point and the point (1,1.5) should be on the
1.5
graph. On the right we have a possible graph. Note that we
can plot an infinite amount of different graphs using these -3 1 2 4 x
data.

Check Yourself 8
1. Plot the graph of the function f(x) = 2x + 4.
2. Find the x-intercept(s) of the function f(x) = 3x – 1.
3. Find the y-intercept of the function g(x) = x2 – 2x + 7.
4. State whether the following graphs belong to functions or not. Explain your answer.
y y y

x x x

5. Plot the graph of any function with x-intercepts –2, 1 and y-intercept 4 so that the graph
passes through the point (3, –1).

Answers
y 1 y
1. 2. 3. 7 4. yes, no, yes 5.
3 4
4

-2 -2 1 3
x x
-1

Functions 135
4. Domain of a Function
If a function f does not model data or come with conditions, its domain is the
largest set of real numbers x for which f(x) is a real number.
For example, an area of a circle with radius x can be modelled such that
f(x) = x2. Since the radius must be positive, the domain is the set of all
positive numbers.
The domain of a function may be stated explicitly. For example, if we write
f(x) = x + 3 x  [2, 5),
then the domain is the set of all real numbers x such that 2  x < 5.
You can think of candies in the jar as
x values and the domain. Or if we write
x2  4 x + 3
f ( x) = f :   .
2 x +1
then the domain is the set of all integers.
If the function is given by an algebraic expression and the domain is not stated explicitly, then
the domain is the set of all real numbers for which the expression is defined as a real number.
For example, the function f(x) = óx – 2 is defined when x – 2  0. So its domain is the set
of all real numbers more than or equal to 2, that is x  [2, ).
Sometimes to find the domain of a function, we find all values that make the function
undefined and throw them out of the set of real numbers.
1
For example, the function f ( x) = is not defined at x = 0, so its domain is any real number
x
x except 0, that is, x  (–, 0)  (0, ) or x  R \ {0}.

Notation
To express the domain of a function f(x), we write “x  …” or “D( f ) = …”.

RULES FOR FINDING THE DOMAIN OF A FUNCTION

1. The polynomial functions have any real number as their domain.


2. The rational functions have any real number except the ones that make the
denominator zero as their domain.
3. Even degreed root functions (square root, fourth degree root, etc.) have any real
number that makes the expression under the root non-negative as their domain.
4. Odd degreed root functions (cubic root, fifth degree root, etc.) have any real number
as their domain.
5. If a function contains the combination of the above functions, then the domain is
found by taking the intersection of all conditions.

136 Algebra 8
EXAMPLE 35 Find the domain of the following functions:
1
a. f(x) = x2 + 2x – 8 b. f ( x) = c. f ( x) = 2 x +1 d. f ( x) = 3 x  7
x+ 2

Solution a. The polynomial functions are defined for any real number x. So D( f ) = .
b. We have a rational function denominator of which should not be equal to zero which
means x + 2  0, that is x  –2. So D( f ) = (–, –2)  (–2, ).
c. The square root is defined when the expression under it is nonnegative.
1 1
That means 2x + 1  0, therefore, x   . So, D( f ) =[  , ).
2 2
d. The cubic root is defined for any real number x. So D( f ) = .

EXAMPLE 36 Find the domain of the following functions:


1 1
a. f ( x) = 2 b. f ( x) =  x  5 + 3 x2  2 x
x  2x 6x

1 1
c. f ( x) = 4 (12  3 x)( x2 +7) + d. f ( x) = – 4 2 x +7 + x3 – 5 x
2
x + 2x 3– x

Solution a. Since we have a fraction, the denominator should be non-zero. That means x2 – 2x  0
or x(x – 2)  0. That gives x  0 or x  2. So D( f ) = (–, 0)  (0, 2)  (2, ).
1
b. means 6 – x  0. x  5 means x – 5  0. 3
x2  2 x always gives real number no
6x
matter what x is.
6  x  0
To find the domain we should solve the system  .
 x  5  0
So D( f ) = [5, 6)  (6, ).
1
c. 4
(12  3 x)( x2 +7) means (12 – 3x)(x2 + 7)  0. means x2 + 2x  0.
x2 + 2 x
To find the domain we should solve the system
(12  3 x)( x2 +7)  0 12  3 x  0
 or  since x2 +7 is always positive.
 x + 2 x  0  x( x + 2)  0
2

So, D( f ) = (–, –2)  (–2, 0)  (0, 4].

1 1
d. – 4 2 x +7 + x3 – 5 x gives a real number when  0 and 2x + 7  0.
3– x 3– x

3 – x > 0
To find the domain it is enough to solve the system  .
2 x +7  0
So, D( f ) = [–3.5, 3).

Functions 137
EXAMPLE 37 Write any function by using the following domains:

a. D( f ) = (–, 4] b. D( f ) = [1, 2)

Solution a. Any real number x that is less than or equal to 4 will work for our function, that is x  4
or 4 – x  0. If we think 4 – x as inside of a square root function, this will give us the
domain D( f ) = (–, 4]. So f ( x) = 4  x is such a function.
b. If we think the problem backwards x  [1, 2) should be a solution that we get from a

 x  1 x – 1  0
system of inequalities  or  which can be a condition for the function
 x < 2 x – 2 < 0
1
f(x) = 2004 x – 1+ .
x–2
Note that these are not the only possible answers.

Check Yourself 9
1. Find the domain of the function f ( x) = 3  x2 + 2 x +8.
x+ 4
2. Find the domain of the function g( x) = x  1+ .
x2  4
3. Write any function by using the domain D( f ) = (–2, 4].

Answers

1.  2. [1, 2)  (2, ) 3. f ( x) = 4 – x
x+ 2

138 Algebra 8
B. PROPERTIES OF FUNCTIONS
1. Equal Functions

EXAMPLE 38 Are the functions f ( x) =


x2  1
x 1
and g(x) = x + 1 the same? Why?

x2  1
Solution We know that = x +1. But this doesn’t mean that f(x) = g(x). To understand this
x 1
fact it is enough to see that D( f ) = (–, 1)  (1, ) and D(g) = . Since D( f )  D(g), the
x2  1
functions f ( x) = and g(x) = x + 1 are not the same.
x 1

EQUALITY OF FUNCTIONS

Two functions f(x) and g(x) are equal if, and only if,
1. f(x) = g(x),
2. D( f ) = D(g).

EXAMPLE 39 Are the functions f(x) = (x0.5)4 – (x0.25)4 and g(x) = x2 – x equal? Why?

Solution Clearly, f(x) = (x0.5)4 – (x0.25)4 = x2 – x = g(x)


However, note that in the original
f(x) = (ñx )4 – (ñx
4
)4, which means D( f ) = [0, ).
Since D(g) =   D( f ), functions f and g are not equal.

2. Even and Odd Functions


Let us consider the function f(x) = x2. Clearly,
f(–2) = f(2) = 4
f(–5) = f(5) = 25.
It is not hard to realize that the function f gives the same value for any number and its
negative in its domain. So we can say that f(–x) = f(x) for any x  D( f ). These kind of
functions are called even functions.
Let us consider the function f(x) = x3. Clearly,
f(–2) = –f(2) = –8
f(–5) = –f(5) = –125.
For this function we can say that f(–x) = –f(x) for any x  D( f ). These kind of functions are
called odd functions.

Functions 139
Definition even, odd functions
A function f is even if, for each x in its domain, f(–x) = f(x).
A function f is odd if, for each x in its domain, f(–x) = – f(x).

EXAMPLE 40 Classify whether the following functions are even or odd.


2
a. f(x) = 3x4 – 4 b. f(x) = 5x3 – 2x c. f ( x) =
2x  7
x2 +1
d. f ( x) = e. f ( x) = x + 2
3x
Solution We should evaluate f(–x) and decide if it is the same of the given function f(x) or its negative.
a. f(–x) = 3(–x)4 – 4 = 3x4 – 4 = f(x). This function is even.
b. f(–x) = 5(–x)3 – 2(–x) = –5x3 + 2x = –(5x3 – 2x) = – f(x). This function is odd.
2 2
c. f (– x) = =
2(  x)  7 2 x  7
This function is neither even nor odd since f(–x)  f(x), nor is f(–x)  – f(x).
(  x)2 +1 x2 +1 x2 +1
d. f (– x) = = = =  f ( x). This function is odd.
3(  x) 3x 3x

e. f (– x) = (– x)+ 2 = x + 2. This function is neither even nor odd.

Note
A function is even, odd or neither. Generally functions that we face are neither even nor odd.

EXAMPLE 41 Classify whether the following functions are even or odd.


x3  2 x 2x  1 x7 x7
a. f ( x) = b. f ( x) = c. f ( x)  
xx 2 x +1 x 1 x 1

(  x)3  2( x) x3 + 2 x ( x3  2 x) x3  2 x


Solution a. f (  x) = = = = = f ( x).
(  x)  x x x x x xx
This function is even. Here note that |–x| = |x| since the absolute value of a number is
equal to the absolute value of its negative.
1 1  2x
x 1
2 1 2 x x 1  2x 2 x 1
b. f (  x) =  x = = 2 x= =– x =  f ( x). This function is odd.
2 +1 1 1+ 2 1+ 2 x
2 +1
+1
2x 2x
x  7  x +7 x +7 x 7  x +7 x 7 
c. f (  x) = + = + =  +  =  f ( x).
 x +1 x  1 ( x  1) ( x +1)  x  1 x +1 
This function is odd.

140 Algebra 8
If f is even, then the points (x0, y0) and (–x0, y0) are on the graph of the function f, which
means the graph of an even function is symmetric with respect to the y-axis.

GRAPH OF AN EVEN FUNCTION

y
y=f(x)
y0

-x0 x0 x

If f is odd, then the points (x0, y0) and (–x0, –y0) are on the graph of the function f which
means the graph of an odd function is symmetric with respect to the origin.

GRAPH OF AN ODD FUNCTION

y
y=f(x)
y0

-x0
x0 x

-y0

EXAMPLE 42 Classify whether the following functions are even or odd:

a. y b. y c. y d. y

x x x x

Solution a. The graph is symmetric with respect to the y-axis making this function even.
b. The graph is symmetric with respect to the origin making this function odd.
c. Using the vertical line test we can easily state that the graph does not belong to a
function. So we cannot talk about the function being even or odd.
d. The graph has no symmetry with respect to the y-axis or the origin. The function is
neither even nor odd.

Functions 141
EXAMPLE 43 Complete the following graphs if they belong to an even or
an odd function.

a. y b. y

x x

Solution In order to complete the graph for an even function just imagine the y-axis as a mirror and
draw the reflection of each point on the other side.
In order to complete the graph for an odd function connect each point with the origin and
extend that line to the other side of the origin until you have the same distance.

a. y y b. y y

x x x x

Check Yourself 10
1. Are the functions f ( x) = ( x  3)( x + 4) and g( x) = x  3  x+ 4 equal? Explain, why.

2. Classify whether the following functions are even or odd:


x2 + 2x4
f(x) = x7 + 4x f ( x) = f(x) = 3x + 3–x – 2|x|
x4

3. Complete the graph on the right if it belongs to y

a. an even function.
b. an odd function. x

Answers
1. no 2. odd, neither, even 3. a. y b. y

x x

142 Algebra 8
3. Increasing, Decreasing and Constant Functions
It is very useful to know where the graph of a function rises and where it falls. The graph
shown below rises, falls, then rises again as we move from left to right, that is as the argument
gets larger.
y
D
B
y=f(x)
C
A

a b c d x

It rises from A to B, falls from B to C, and rises again from C to D. The function f is said to
be increasing when its graph rises and decreasing when its graph falls. If there is no rise or
fall in the graph, then we say the function is constant.

Definition increasing, decreasing, constant function


A function f is increasing on an interval I if f(x1) < f(x2) for any x1 < x2 in I.
A function f is decreasing on an interval I if f(x1) > f(x2) for any x1 < x2 in I.
A function f is constant on an interval I if f(x1) = f(x2) for any x1 < x2 in I.

GRAPH OF INCREASING, DECREASING, CONSTANT FUNCTION


y y y
y=f(x) y=f(x)
y=f(x)
f(x1)
f(x2)
f(x2) f(x1) f(x2)
f(x1)
x1 x2 x x1 x2 x x1 x2 x
An increasing function A decreasing function A constant function

Functions 143
EXAMPLE 44 Investigate the following functions for increase and decrease:

a. f(x) = 4 b. f(x) = 2x + 5 c. f(x) = –x + 4

Solution Let us plot their graphs to see the rise and fall.
y y y

y=4 5
4 4

-2.5 4
x x x
y = –x + 4
y = 2x + 5

a. f(x) = 4 is constant on D( f ).
b. f(x) = 2x + 5 is increasing on D( f ).
c. f(x) = –x + 4 is decreasing on D( f ).

Note
A linear function is always
A line in the form
y = mx + n 1. increasing if its slope is positive,
has m as its slope.
2. decreasing if its slope is negative,
3. constant if its slope is zero.

EXAMPLE 45 State the intervals on which the function, whose graph y


is given on the right, is increasing, decreasing or
constant.

-5 -2 1 x

Solution Note that when the question is about the intervals of increase and decrease we are interested
in the argument, that is, the x-axis. Looking at the graph we see that there is a rise until
x = –5 (note that since the beginning of the graph is not fixed by a point, this rise begins at
minus infinity), there is a constant behaviour until x = –2, there is a fall until x = 1, and
finally there is a rise “until” plus infinity. Mathematically our answer is as follows:
The function is increasing on (–, –5] and [1, ), decreasing on [–2, 1], constant on [–5, –2].
Note that we find the largest possible interval of increase and decrease. So the intervals have
closed brackets whenever possible.

144 Algebra 8
EXAMPLE 46 Prove that f ( x) =
5
2 x +1
is decreasing on (–, –0.5).

Solution In this problem we will use the definition. Let x1 < x2 < –0.5, then we must show that
f(x1) > f(x2) on (–, –0.5).
5 5 10( x2  x1 )
f ( x1 )  f ( x2 ) =  =
2 x1 +1 2 x2 +1 (2 x1 +1)(2 x2 +1)

 x2  x1 > 0
 10( x2  x1 )
Since x1 < x2 < 0.5, 2 x1 +1< 0 . So, > 0.
 (2 x1 +1)(2 x2 +1)
2 x2 +1< 0
5
It means that f(x1) – f(x2) > 0 or f(x1) > f(x2). Thus, f ( x) = is decreasing on (–, –0.5).
2 x +1

EXAMPLE 47 Investigate f(x) = x2 for increase and decrease.

Solution Note that f(x) is an even function. So let’s investigate it in [0, ).
If 0  x1 < x2, then f(x2) – f(x1) = x22 – x12 = (x2 – x1)(x2 + x1).
 x2  x1 > 0
Since 0  x1 < x2, then  . So (x2 – x1)(x2 + x1) > 0.
 x2 + x1 > 0
It means that f(x2) – f(x1) > 0 or f(x1) < f(x2).
Thus, f(x) = x2 is increasing on [0, ). Since f(x) is even, it is decreasing on the other side
of the y-axis (Think about the symmetry with respect to the y-axis). So the function is
increasing on [0, ), decreasing on (–, 0].

Check Yourself 11
1. Find a if f(x) = (a – 3)x + 4 is constant on D( f ).
2. State the intervals on which the function whose y

graph is given on the right is increasing, decreasing 2


or constant. 3 5 x

4
3. Prove that f ( x) = is increasing on (2, ).
2x
4. Investigate f(x) = x3 for increase and decrease.
Answers
1. 3 2. increasing on [2, 3], decreasing on (–, 2], [5, ), constant on [3, 5]
3. consider f(x1) – f(x2) when 2 < x1 < x2 4. increasing on 

Functions 145
4. Reading Graph of a Function
Graphs give us the opportunity to see
the general details of a function at one
glance. In order to get the necessary
information it is important to read a
graph properly. We read a graph the
same way that we would read a book,
from left to right. The behaviour of the
function is determined by the y-values,
but the intervals are reported in terms
of the x-values.

Notation
1. If a point is plotted by a full circle, it means that the point is included in the graph.
2. If a point is plotted by an empty circle, it means that the point is not included in the graph.
3. If the endpoint of a graph is a full or empty circle, it means the part of the graph on that
side stops at that point; otherwise it means that the graph continues up to infinity.

EXAMPLE 48 Answer the following using the graph below:


y y=f(x)

-6 -4 5
-7 -3 6 x
-1

-3

a. Find f(–6), f(0), f(ñ3), f(–5), f(–4).


b. Find the domain and the range of f.
c. Find the x and the y-intercepts of f.
d. Solve f(x) = –3.
e. Solve f(x)  0.
f. Find the intervals on which f is increasing, decreasing and constant.
g. Is f even or odd?
h. Find the minimum and the maximum value of f .

146 Algebra 8
Solution a. We just find the corresponding y coordinate on the graph for each given x value, that is
for –6, 0, ñ3, –5, –4. Clearly, f(–6) = –1, f(0) = –3, f(ñ3) = –3.
f(–5) and f(–4) are undefined since no part of the graph has x coordinate being equal to
–5 or –4.
b. Graphically finding the domain means writing all of the values of x for which a vertical line
intersects the graph of f . So, D( f ) = (–, –6]  (–4, ).
Similarly finding the range means writing all of the values of y for which a horizontal line
intersects the graph of f . So, E( f ) = [–3, ).
c. Finding the x-intercepts means finding the values of x for which the graph crosses the
x-axis. So, the x-intercepts are –7, –3 and 6.
Finding the y-intercept means finding the value of y for which the graph crosses the
y-axis. So, the y-intercept is –3. y
y=f(x)
d. To solve f(x) = –3 means finding values
of x for which y is equal to –3. We see 5
that there are an infinite amount of x
solutions: x  [0, 5].
-3

e. To solve f(x)  0 graphically, you must y


y=f(x)
ask for which values of x is the graph of
f above or at the same level with the line
y = 0. So, the answer is -7 -3 6 x
x  (–, –7]  [–3, 0)  [6, ).

f. Recall that intervals are reported in y


terms of the x-values and for increase, y=f(x)
decrease or being constant we write the
largest possible interval. -6 -4 5
x
f is increasing on (–4, 0) and [5, )
f is decreasing on (–, –6]
f is constant on [0, 5]
g. f is neither even nor odd since its graph has no symmetry with respect to the y-axis nor
the origin.
h. The maximum value of f is the highest possible y value that the graph can reach. In our
graph since it moves as high as possible we say that the maximum value doesn’t exist or
the maximum value is plus infinity.
The minimum value of f is the lowest possible y value that the graph can reach. In our
graph the minimum value is –3.

Functions 147
EXAMPLE 49 Answer the following using the graph below:

3
2

y=f(x)
1 2
-6 -4 -2 -1 3 5 7 x
-1

-3

a. Find the domain and the range of f.


b. Find the x and the y-intercepts of f.
c. Solve f(x) = –1.
d. Solve –3 < f(x)  –1.
e. Solve f(x) > 2.
f. Solve f(x)  2.
g. Find the intervals on which f is increasing, decreasing and constant.
h. Is f even or odd?
i. Find the minimum and the maximum value of f .
j. When does f have its minimum and maximum value?

Solution a. D( f ) = [–6, 7) and E( f ) = [–3, 3].

b. The x-intercepts are –1, 3 and the y-intercept is –1.

c. x  {0, 2}.

d. x  [0, 1)  (1, 2].

e. x  [–6, –2).

f. x  [–6, –2]  {5}.

g. f is increasing on [1, 5], decreasing on [–4, 1] and [5, 7), constant on [–6, –4].

h. f is neither even nor odd.

i. The maximum value of f is 3 and the minimum value is –3.

j. f has its maximum value when x  [–6, –4] and has its minimum value when x = 1.

148 Algebra 8
Graphs are widely used in many areas like statistics, economics, engineering, medicine,
meteorology, etc. Interpreting information from them in real situations is the same as reading
a graph. In fact those graphs belong to the functions that are the models of daily situations.

EXAMPLE 50 Describe a situation that can be modelled by


the graph shown.
Heart Rate

Time

Solution The graph shows the heart beat rate of a person in a time interval. It increases for a period
of time and then decreases back to its initial condition. The person may have dealt with a
sport activity during the interval of increase and after finishing that activity, the heart beat
rate decreased to its normal condition.

EXAMPLE 51 Answer the following questions using the graph below:


a. In what period did the car sales occur?
Car Sales
b. How many cars were sold in April? 35
30
c. In which month were 15 cars sold? 25
Number 20
d. In which month were the sales the best? of cars
15
10
e. In which month were the sales the worst? 5
0
f. In which two months were the sales same? Jan Feb Mar Apr May Jun
Month
g. In which months did the sales increase?
h. In which months did the sales decrease?

Solution a. The first six months of the year, that is from January to June, inclusive.
b. 25 c. March
d. May e. June
f. January and February g. April and May
h. March and June

Functions 149
Check Yourself 12
Answer the following using the graph below:
y
y=f(x)

(-4,1) (5,1)
(2,0)
(0,0) x

(1,-3)

1. Find the domain and the range of f .


2. Find the x the and the y intercepts.
3. Find the intervals on which f is increasing and decreasing.
4. Solve f(x)  1.

Answers

1. D( f ) = (–, 2)  (2, 5), E( f ) = [–3, ) 2. 0; 0


3. increasing on (1, 2) and (2, 5), decreasing on (–, 1) 4. x  [–4, 2)  (2, 5)

150 Algebra 8
EXERCISES 4 .2
A. Defining a Function 2 x +1 if x >1
1. State whether the following relations are functions 
5. Given that f ( x) = 3 if x =1,
or not. 
3x if x <1
a. {(1, x), (2, y), (3, x)} with the domain {1, 2, 3}
f (2)+ f (1)
b. {(a, b), (a, c), (b, a), (c, a)} with the domain find .
f ( 1)
{a, b, c}
c. {(1, 1), (2, 2), (3, 1), (4, 1)} with the domain
6. Plot the graphs of the following functions within the
{1, 2, 3, 4, 5}
given domain.

a. f(x) = –x2, D( f ) = {–2, –1, 0, 1, 2}


2. Express the following rules in a function notation. b. g(x) = 2 – x, D(g) = {–1, 0, 1, 2, 3}
a. Multiply by 5, then add 2. c. h(x) = ñx, D(h) = {0, 1, 4, 9}
b. Square the number then subtract twice the d. f(x) = 2x + 1, D( f ) = 
number.
c. Divide by two, then add three times its cube. 7. Find the x and the y intercepts of the following
functions.

3. Express the following rules in words. a. f(x) = 3x – 7


b. f(x) = (x – 6)(x + 1)
a. f(x) = 2x – 4 b. g( x) = x +1
2x  4
x c. f ( x) =
c. h( x) = x +5
3x +1
d. f ( x) = x + 4  5

4. Find the required values for the following functions. 8. State whether the following graphs belong to a
2
a. If f(x) = 2x – 5x, find function or not.
f(1), f(–3), f(–x), – f(x), f(ñx), f(x + h). a. y b. y c. y
3x  1
b. If g( x) = , find
x2 + 2
x x
g(0), g(4), g(3x), 2g(–x), g(x2), g(2a + b). x
u 2 +1 d. e. f.
c. If h( u ) = 3 , find y y y
u + 2u
h(–1), h(0), h(2u), h(x + 1), h(u2), h2(u).
x x
(notation h2(u) is used in the place of (h(u))2 which means
the square of the function.) x

Functions 151
9. Find the domain of the following functions.
5 x
h. f ( x) =
a. f(x) = 2x + 1 x 1
2
b. f(x) = x – 2x + 5
1 3
x +1 i. f ( x) = 
c. f ( x) = 2 5x
x4
x x2  7 x +10
d. f ( x) = 2 j. f ( x) =
x 9  x2 +6 x  9
e. f ( x) = 3x  6
1 2 11. Write a formula for the function by using the
f. f ( x) =  
x x+ 3 following domains.
1 a. [3, 5)
g. f ( x) =  x 5
6x b. [–4, –2)  (–2, 0)
h. f ( x) = x  5 + 30  3 x c. [–2, 1]  {3}
1
i. f ( x) = + 1 x
5x  1 12. For which values of x are the following functions
1 equal?
j. f ( a) = 3
+ 1+ a  3  2 a
a 1 a. f(x) = 2x – 3 and g(x) = 5x + 1
3u  4 1 b. f(x) = x + 22 and g(x) = x2 + x + 6
k. f ( u ) = + + u3
u  2 3u  10
3x  1
c. f ( x) = and g(x) = 3
x2
10. Find the domain of the following functions.
 B. Properties of Functions
a. f ( x) = 15  2 x  8 x2 13. Classify whether the following functions are even,
odd or neither.
1 x+ 4
b. f ( x) = 2 + a. f(x) = x3 + x b. f(x) = x5 + x3 – 1
x x2 5  3x
x 1
x+ 3 c. f(x) = 5 – x2 + x4 d. f ( x) =
c. f ( x) = x2 +1
x3  x2  x +1
1 x
e. f ( x) = f. f ( x) =
4x + 3 4
x 3 1+ x2
d. f ( x) =
x  2x  8
2 3
x  2x 10 x  10  x
g. f ( x) = h. f ( x) =
x2 +1 10 x +10  x
e. f ( x) = x2 +6 x +5 + x2 + x + 2
 x + 2 if x  0
i. f ( x) = 
f. f ( x) = x2 (1  x)( x  2)  x  2 if x < 0
1 3x2 +1 if x > 0
g. f ( x) = j. f ( x) = 
( x  1)2 (3x  x2 )

 3x  1 if x < 0
2

152 Algebra 8
14. Given their graphs classify whether the following 16. Plot the graph of a function by the following
functions are even, odd or neither. intervals of increase and decrease.
a. b. c. a. Increasing on (–, 2] and [5, ), constant on
y y y
[2, 5]
b. Increasing on [–5, 2] and [6, 9], decreasing on
x x x [2, 6] and [9, 11]
c. Increasing on (–, 3] and [7, ), decreasing on
d. y e. y f. y [4, 5], constant on [3, 4] and [5, 7]

x x x 17. Prove that:



a. f(x) = 3 – 2x is decreasing on .
4
b. f ( x) = is increasing on (2; ).
2x
15. Complete the following graphs to get an even 21x  9 1
c. f ( x) = is increasing on (–; ).
function and to get an odd function. 3x  1 3
4x + 31
a. (-4,6) y d. f ( x) = is decreasing on (–7; ).
x +7
e. f(x) = x4 – 8x is increasing on [2; ).
(-7,4)
(0,0)
x
18. Given that a  f(x) = x  f(x) + bx + 2 and

(-2,-2) g(x) = x + 5, find g(ab) if f is a constant function.

19. f(x) = ax3 + bx + 1, a  0, b  0 and f(10) = 4,


b. 
y find f(–10).
(4,4)
(0,4)
20. Answer the following using the graph below:
y
x
3
(7,-4) y = f(x)
5
-6 -2 3 6 x
(5,-6)
a. Find the domain and the range.
c. y
(2,4)
b. Find the intervals on which the function is
(7,2)
increasing and decreasing.
(-2,0) c. Find the x and the y intercepts.
(5,0) x d. Solve f(x – 2) = 0.
(0,-2)
e. Solve f(x) < 0.

Functions 153
21. Answer the following using the graph below: 23. Two ships leave the port at the same time, one
y y = f(x) sailing east at a rate of 9 km/h and the other
sailing south at 12 km/h. If t is the time (in hours)
2
after their departure, express the distance d
-5 -3 3 5
between the ships as a function of t.
-10 -6 6 10 x

a
-7

a. Find the domain and range of f(x).

b. Solve f(x) = 0. b d

c. Find f(0).

d. Find f(–10).
N
e. Solve f(x)  0.

f. Solve f(x) > 2.


24. The freezing point of water is 0° C, or 32° F, and
g. Solve f(x) = 2. the boiling point is 100° C, or 212° F.

h. Is f(x) even or odd? a. Express the Fahrenheit temperature F as a


linear function of the Celsius temperature C.
i. Write down the intervals where f(x) is
b. What temperature increase in F corresponds
increasing, decreasing or constant.
to an increase in temperature of 1° C?
j. Find the minimum and the maximum value of
f(x).
25. A cross section of a rectangular pool with the
k. Find all the x values for which f(x) = –7. 
dimensions 60 m by 20 m is shown in the figure.
l. Find all the x values for which f(x)  (–7; 2). The pool is being filled with water at a rate of
10 m3 / min.
60
1
h 3
15
Mixed Problems

x  1 if x > 3 a. Express the volume V of the water in the pool


 as a function of time t.
22. Given that f ( x) =  x2  2 x  3 if 1< x  3,
 b. Express V as a piecewise function of the depth
 x + 4 if x  1 h at the deep end for 0  h  2 and then for
find f ( f ... f (0)...). 2<h3
 
2004 times c. Express h as a piecewise function of t.

154 Algebra 8
26. The graph below gives the weight of a certain x +1 
30. If f   = x, find f(–1), f(3), f(0).
person as a function of age. Describe in words how  x 1
this person's weight has varied over time. What do
you think happened when this person was 40
years old?

100 31. If f(x) = 2x9 – 3x7 + ax – 3 and f(11) = 2, find


Weight 75
f(–11).
(kg)
50

25

10 20 30 40 50 60 70 Age
(years) 32. Find the intervals on which the function

f(x) = x2+|5x+1|+5 is increasing and decreasing.

27. The graph gives a salesman’s distance from his


home as a function of time on a certain day.
Describe in words what the graph indicates about
1 1
his travels on this day. 33. If f  x   = x3  3 , find f(–x).
  x x

Distance
from home
(km)

34. If f(x) = ax2 + bx + c and


8 A.M. 10 NOON 2 4 6 P.M. Time 
(hours) f(x – 1) + f(x) + f(x + 1) = x2 + 1, find f(2).

28. Find the range of the following functions.


2  f ( x +1)+1
a. f(x) = 4 – x2 b. f ( x) = x +1+ 3 35. If f : +  , f(25) = 1, f ( x) = ,
 2
1 x3 find f(3).
c. f ( x) = 2 d. f ( x) =
x +6 x+ 4

29. If f(x) + f(x + 1) = 2x + 3 and f(2) = 3, find 1


 36. If g( x)+ 2  g   = x, find g(x) in terms of x.
f(99).   x

Functions 155
A. BASIC OPERATIONS
Two functions f and g can be combined to form new functions f + g, f – g, f  g, and f / g in
a similar way that we add, subtract, multiply, and divide the real numbers.
For example, we define the function f + g as (f + g)(x) = f(x) + g(x). This new function is
called as a sum of functions f and g. Its value at a given x value is found by adding the value
of f and g at that x value. The operation is similar for the case of the difference, the product
and the quotient. Clearly, the new function is defined only when f and g is defined, that the
domain of the new function is also the intersection of the domains of f and g. In case of the
quotient the values that make denominator equal to zero must be excluded from the domain.

SUM, DIFFERENCE, PRODUCT AND QUOTIENT OF FUNCTIONS

Let f and g be two functions with domains A and B respectively. Then


Operation Definition Domain
Addition ( f + g)(x) = f(x) + g(x) AB
Subtraction (f – g)(x) = f(x) – g(x) AB
Multiplication (fg)(x) = f(x)g(x) AB
Division (f/g)(x) = f(x)/g(x) A  B, g(x)  0

EXAMPLE 52 Given that f(x) = x2 – 4 and g(x) = x + 3, find


a. f + g, f – g, fg, f / g and their domains. b. (f + g)(1), (f – g)(1), (fg)(1), (f / g)(1).

Solution a. Note that D( f ) =  and D(g) = . So we have


( f + g)( x) = f ( x)+ g( x) = x2 – 4 + x + 3 = x2 + x – 1 with the domain ,
( f – g )( x) = f ( x) – g( x) = x2 – 4 – ( x + 3) = x2 – x – 7 with th e domain ,
( fg)( x) = f ( x)g( x) = ( x2 – 4)( x + 3) = x3 + 3 x2 – 4 x – 12 with the domain ,
f ( x ) x2 – 4
( f / g )( x) = = with domain ( ,  3) (–3, ) since – 3 makes t he
g( x) x+ 3
denominator of the new function equal to zero.
2
b. ( f + g)(1) = f (1)+ g(1) = 1 – 4 + 1+ 3 =1
( f – g )(1) = f (1) – g(1) = 12 – 4 – (1+ 3) = –7
( fg )(1) = f (1)g(1) = (12 – 4)(1+ 3) = –12
( f / g )(1) = f (1) / g(1) = (12 – 4 ) /(1+ 3) = –3 / 4

156 Algebra 8
EXAMPLE 53 Given that f(x) = 2, g(x) = 3x2 – 11, h(x) = 2 x  6 and m( x) =
f ( x)+ g( x)
h( x)
+ f ( x)g( x),
find m(5) and the domain of m.

f ( x) + g( x) 2 + 3x2 – 11 3 x2  9
Solution m( x) = + f ( x)g( x) = + 2(3x2 – 11) = +6 x2  22.
h( x) 2x – 6 2x  6

3  52  11
m(5) = +6  5 2  22 =160.
2  56

The domain of function m can directly be found by considering the final formula for m(x).
The only condition that we should deal with is 2x – 6 > 0. So, D(m) = (3, ).

B. COMPOSITION OF FUNCTIONS
Aside from the four basic operations there is a very important way of combining functions to
get a new function. We call this a composition. This operation is illustrated as follows:
Here we see two different functions f and g f
g. Their common point is that the range
of one is the domain of the other. Range of g Range of f

Because of that fact the domain of the –2


2 –1
function g is linked with the range of the 0
3 0
1
function f. Consider x = –2: 5 1
3
g(–2) = 3
Domain of g Domain of f
f(g(–2)) = f(3) = 0
In other words, –2 finally maps to 0. Note f(g(x))
that to find the final mapping we first use h
g and then f. If we name that final
mapping as h, then h(–2) = 0 where –2
–1
f(g(x)) means 0
h(x) = f(g(x)). 0
g is applied first, 1
f is applied second. That final mapping h is another function 1
3
and is the composition of f and g as
Domain of h Range of h
shown below on the right:

As an algebraic example if f(x) = ñx and g(x) = 3x + 1, we know that f(x) is the rule “take
the square root of the number x” and g(x) is the rule “multiply the number x by 3 and add
Read f(g(x)) as
1”. In that case f(g(x)) means you must “first multiply the number x by 3 and add 1, then take
“f of g of x”.
the square root of the result” so we can define h( x) = f ( g( x)) = g( x) = 3x +1. Here we
say that h(x) is a composition of f(x) and g(x), that is h(x) is a composite function.
The domain of f(g(x)) is the set of all x in the domain of g such that g(x) is in the domain of f.

Functions 157
Composite functions are composed of “bridges” that link the domain of “first” with the range of “last”.

Definition composition
f(g(x)) is defined as composition of f and g. It is also denoted by (f  g)(x) or simply f  g.

fog

g f
x g(x) f(g(x))

Arrow diagram for (f o g)(x)

EXAMPLE 54 Given that f(x) = x2 + 3x, g(x) = x – 4, find f(g(x)) and g(f(x)).

Solution 1 f(g(x)) = g(x)2 + 3g(x) = (x – 4)2 + 3(x – 4) = x2 – 8x + 16 + 3x – 12 = x2 – 5x + 4


g(f(x)) = f(x) – 4 = (x2 + 3x) – 4 = x2 + 3x – 4
So f(g(x)) = x2 – 5x + 4 and g(f(x)) = x2 + 3x – 4.
What can you say about f(g(x)) and g(f(x))? Is it true that they are the same?

Solution 2 f(g(x)) = f(x – 4) = (x – 4)2 + 3(x – 4) = x2 – 5x + 4


g(f(x)) = g(x2 + 3x) = (x2 + 3x) – 4 = x2 + 3x – 4

Note
In general, f(g(x))  g(f(x)).

158 Algebra 8
EXAMPLE 55 Let f(x) = x + 1, g(x) = x3. Find:

a. (f  g)(2)

b. (g  f)(1)

c. (f  f)(5)

d. (g  g)(–1)

Solution 1 a. (f  g)(x) = f(g(x)) = g(x) + 1 = x3 + 1


(f  g)(2) = 23 + 1 = 9
b. (g  f)(x) = g(f(x)) = (f(x))3 = (x + 1)3 Composite functions are formed of functions that are
3 inside another function.
(g  f)(1) = (1 + 1) = 8
c. (f  f)(x) = f(f(x)) = f(x) + 1 = x + 2
f  g means g is applied
first, f is applied second (f  f)(5) = 5 + 2 = 7
and in general f  g  g  f.
d. (g  g)(x) = g(g(x)) = (g(x))3 = x9
(g  g)(–1) = (–1)9 = –1

Solution 2 Note that these values can be calculated without finding the formula.

a. In case of ( f  g)(2) we may first calculate g(2) = 8 and then f(8) = 9 to find the answer
since ( f  g) (2) = f(g(2)) = f(8) = 9.

b. (g  f )(1) = g( f(1)) = g(2) = 8

c. ( f  f )(5) = f( f(5)) = f(6) = 7

d. (g  g)(–1) = g(g(–1)) = g(–1) = –1

EXAMPLE 56 Let f ( x) =
x
x 1
, g(x) = x5 and h(x) = x2 + x. Find f  g  h.

Solution 1 Here we have a composition of three functions.


g( h( x)) ( h( x))5 ( x2 + x)5
( f  g  h)( x) = f ( g( h( x))) = = = 2 .
g( h( x)) – 1 ( h( x)) – 1 ( x + x)5  1
5

( x2 + x)5
Solution 2 ( f  g  h)(x) = f(g(h(x))) = f(g(x2 + x)) = f((x2 + x)5) = .
( x2 + x)5 – 1
Functions 159
EXAMPLE 57 Write the function h( x) = 5x + 3 as a composition of two functions.

Solution Let us read what the formula tells us to do: “first add 3 to five times the number x and then
take the square root of all”. Let f(x) = 5x + 3 and g(x) = ñx. Note that f is applied first, g is
applied second. So h(x) = (g  f)(x).

The composite function form is never unique.


For example, consider the previous example:
If n is any nonzero integer, we could choose
f(x) = (5x + 3)1/n and g(x) = xn/2.
Thus, there are an infinite amount of composite
function forms. Generally, our aim is to choose a
formula such that the expression for the function
is simple, as in the above example.

58
1
EXAMPLE Given f(x) = 3x + 1 and g(x) = 2x – m such that (f  g)(x) = (g  f )(x), find g( ).
10

Solution ( f  g)(x) = f(g(x)) = 3  g(x) + 1 = 3(2x – m) + 1 = 6x – 3m + 1


(g  f )(x) = g( f(x)) = 2  f(x) – m = 2(3x + 1) – m = 6x + 2 – m
1
Since ( f  g)(x) = (g  f )(x), 6x – 3m + 1 = 6x + 2 – m which gives m = – .
2
1 1 7
So we have g(x) = 2 x + . Therefore, g( ) = .
2 10 10

EXAMPLE 59 If the rule for function f  g  h is “first take the square root of a number plus twenty-five,
second divide the new number plus 2 by itself, third take the cube of the newest number”
and p(x) = 3x2 + 6x. Find (f  g  h)(x) and (h  p  f)(–2).

Solution First of all let us find formulae for f, g and h.


Since f  g  h means h is applied first, g second and f third we have
x+ 2
h( x) = x + 25, g( x) = , f ( x) = x 3.
x
3
 x + 25 + 2 
According to the rule, ( f  g  h)(x) =  .
 x + 25 

Now let us find (h  p  f)(–2):
(h  p  f )(–2) = h(p( f(–2))) = h(p(–8)) = h(144) = 13.

160 Algebra 8
EXAMPLE 60 Given that f(g(x)) = 4x – 1 and g(x) = x + 2, find f(x).

Solution Since f(g(x)) = 4x – 1 and g(x) = x + 2 we have f(x + 2) = 4x – 1.


Let x + 2 = a. Then x = a – 2.
Substituting x = a – 2 in f(x + 2) = 4x – 1 we get
f(a – 2 + 2) = 4(a – 2) – 1 or f(a) = 4a – 9.
Since this is just a matter of notation in the final formula instead of a we choose x to get
f(x) = 4x – 9.

Check Yourself 13
x +1
1. Given that f ( x) = and g(x) = x, find f + g, f – g, fg, f / g.
2x  4
2. Given that f(x) = x2 and g(x) = 3x + 4, find (f  g)(1) and (g  f)(1).
3. Write the function h(x) = (2x – 7)5 as a composition of two functions such that
h(x) = (f  g)(x).
Answers
2 2 2
1. 2 x  3x +1 , 2 x +5 x +1 , x + x , x2+1 2. 49, 7 3. f(x) = x5, g(x) = 2x – 7
2x  4 2x  4 2x  4 2x 4x

Functions 161
C. INVERSE OF A FUNCTION
1. One-to-one Functions
Consider the two functions represented by a map as shown below:

f g

·1 ·4 ·1
·4
·2 ·5 ·2
·3 ·6 ·3 ·5

Each number in the domain of f takes a different value


from the range. But it is not the same for the function g
since 1 and 2 take the same value in the range. We
name a function one-to-one if each element in the
domain corresponds to exactly one element from the
range. So here f is one-tto-o
one and g is not one-to-one.
To understand whether a given formula belongs to a
one-to-one function or not we need an algebraic tool.
The following definition for one-to-one function meets
our needs:

Definition one-tto-o
one function
one if, for each x1  x2 in its domain, f(x1)  f(x2).
A function f is one-tto-o

EXAMPLE 61 Which one of the following is a one-to-one function?

a. f(x) = 2x – 1 b. f(x) = x2

Solution Let us use the definition for one-to-one function to find out whether they support the
definition or not:
a. Let x1  x2. Then f(x1) = 2x1 – 1 and f(x2) = 2x2 – 1. Let’s assume that f(x1) = f(x2), so
2x1 – 1 = 2x2 – 1 which means x1 = x2. But this is wrong since we let x1  x2! So our
assumption that f(x1) = f(x2) is also wrong. That means when x1  x2, we have f(x1)  f(x2).
Therefore, the function is one-to-one.
b. Clearly, if we choose –2  2 we get f(–2) = f(2) = 4. Although we can find an infinite
amount of such examples, just one of them is enough to decide that function is not
one-to-one.

162 Algebra 8
We know that the graph of a relation is a function if any vertical line
crosses the curve at most once. Similarly, a function is one-tto-o
one if
any horizontal line crosses the curve at most once. For example:
y y

x x

Graph of Graph of
a function which is not one-to-one a one-to-one function

HORIZONTAL LINE TEST FOR THE ONE-TO-ONE FUNCTIONS

A function is one-to-one if, and only if, no horizontal line crosses its graph at more than
one point.

Note that in the previous example we could have found the answers without using the
definition but just by plotting their graphs and applying the horizontal line test.

EXAMPLE 62 Which one of the following graphs belong to one-to-one functions?

a. y b. y c. y

x x x

Solution y y y

x x x

a. This graph does not belong to a one-to-one function.


b. This graph does not belong to a function, so we can not talk about one-to-one function.
c. This graph belongs to a one-to-one function.

Note
A function that is only increasing or decreasing in its domain is one-to-one.

Functions 163
Check Yourself 14
3x  1
1. Prove that f ( x) = is a one-to-one function.
5
2. Draw a graph which belongs to a one-to-one function.
3. Draw a graph which does not belong to a one-to-one function.

Answers
1. Compare f(x1) and f(x2) when x1  x2.
2. Consider a function which is either increasing or decreasing.
3. Consider a function which is sometimes increasing and sometimes decreasing.

2. Definition for The Inverse of a Function


As mentioned earlier in this book, many mathematical relations can be modelled as functions.
For example,

C = f(r) = 2r V = g(a) = a3


The circumference of a circle The volume of a cube is
is a function of the radius r. a function of the edge a.
In many cases we are interested in reversing this correspondence determined by a function:

C
r  m C   a  n V   3 V
2
The radius of a circle is a function The edge of a cube is a
of the circumference C. function of the volume V.

As illustrated above, reversing the relation between two quantities produces a new function.

164 Algebra 8
Recall that a function is a relation that assigns to each element in the domain exactly one
element from the range.

f
x0 y0

Domain Range

In the figure above we see an arrow diagram for function f. So “Using the rule f we say that
x0 becomes y0”. We symbolize this as “f(x0) = y0”.

Now consider this question: Using the rule f what gives us y0? Of course the answer is x0. So
–1
Read f (x) as “Using the rule f to get y0 we need x0”. We need a new notation so that “x0” will be our
“f inverse of x”. answer. And we symbolize this fact as “f –1(y0) = x0”.
1
f –1(x) doesn’t mean .
f ( x)
1
The reciprocal is f –1
f ( x)
x0 y0
written as (f(x))–1.

Range Domain

Note
The domain of a function is the same as the range of its inverse.
The range of a function is the same as the domain of its inverse.

The domain and the range change their places in an inverse of a


function just as the server and the opponent change their places
in a table tennis game.

Functions 165
EXAMPLE 63 Given the function f = {(0, 2), (–1, 4), (4, 6), (5, 5)},

a. find f –1. b. find f –1(4).

Solution a. Interchange the x and y-coordinates of each ordered pair of f to find f –1:
–1
f = {(2, 0), (4, –1), (6, 4), (5, 5)}
b. To find the value of f –1(4), notice that f –1(4) is the second coordinate when the first
coordinate is 4 in the function f –1. So f –1(4) = –1.

EXAMPLE 64 If f(x) = 2x – 3, find f –1(5).

Solution If we substitute 5 in place of x we find f(5) = 2  5 – 3 = 7 but not f –1(5)!


Finding f –1(5) means finding the x value for which f(x) = 2x – 3 gives 5 as a result.
2x – 3 = 5, so x = 4 or f –1(5) = 4.
If f –1 also supports the definition for a function, we will call f –1 as the inverse of f. But this is
not always possible. For example, consider the relation below:
· -3 f · -2
· -1 · -1
·0
·1
·1
·2 ·3

Domain Range

Clearly, this is a function since each element is assigned to exactly one element. Now let’s
consider its inverse:

· -3 f –1 · -2
· -1 · -1
·0
·1
·1
·2 ·3

Range Domain

As we see f –1(1) = –3 or f –1(1) = 0 is not possible for a function (just one element must be
assigned for each element in the domain). So although f is a function, f –1 is not a function.
That means the inverse of f doesn’t exist.

Note
It is not true that every function has an inverse.

For the inverse of a function to be defined, it is necessary that different elements in the domain
always give different values. As we know such functions are called one-to-one functions.

166 Algebra 8
As another example let us consider the following function graph of which is given below:
y

y = f(x)

To find the inverse, we use the same procedures that we used for relations. Drawing the
reflection with respect to y = x we get the following picture (below left):
y y
y = f –1(x) y = f –1(x)

y = f (x)

x x

y=x

We know that a set of points in the coordinate plane is the graph of a function if, and only if,
no vertical line crosses the graph at more than one point. But that rule doesn’t hold for f –1
(look at the picture above right). That means the inverse of f doesn’t exist. Note that the
inverse we draw will be a function if f has no horizontal line crossing the graph at more than
one point, that is if f is one-to-one.
We know that the range of a function becomes the domain of its inverse. We also know that
no element from the domain of a function must be left unassigned.
Sometimes the range of a function is explicitly so that it is larger than the function’s its real
range, that is some elements of the range are not used. In that case when we talk about the
inverse some elements in the domain of inverse will be unassigned. This will result in an
absence of the inverse function.
For example, if f(x) = x + 2 such that f:   , although the range seems to be  it is in
fact  (Note that when we put any integer in x + 2, we always get other integers).
To guarantee that we will not face such functions we must be sure that any element in the
range is assigned by an element from the domain. Such functions are called onto functions.

CRITERIA FOR EXISTENCE OF INVERSE OF A FUNCTION

A function has an inverse if, and only if, it is one-to-one and onto.

If a function is given by only formula or graph (where no explicit range is given), it is


naturally an onto function. So there is no need to think about this detail to define its inverse.

Functions 167
Now we can give the formal definition for the inverse of a function:

Definition inverse function


Let f be a one-to-one and onto function with the domain A and the range B. Then its inverse
function f –1 has the domain B and the range A such that f(x) = y  f –1(y) = x.
By definition the inverse function f –1 undoes what f does. That is, if we take x, apply f, and
then apply f –1, we arrive back at x where we started. Similarly, f undoes what f –1 does. That
is why f and f –1 are the inverses of each other.

Note
f(f –1(x)) = x and f –1(f(x)) = x.

EXAMPLE 65 Prove that f(x) =x3 and g(x) = 3 x are inverses of each other.

Solution When we find f(g(x)) = ( 3 x )3 = x, we can see that f(x) and g(x) are the inverses of each
other. Note also that g(f(x)) = x.

EXAMPLE 66 If f(x) = x + 2 and f –1(x) = x – a, find a.

Solution We know that f(f –1(x)) = x. So f(x – a) = x or (x – a) + 2 = x. That gives a = 2.

3. Finding The Inverse of a Function


Given the graph of a function, to find the inverse we use the following procedure:
1. Verify that the function is one-to-one by applying the horizontal line test.
2. Take symmetry of the graph of the function with respect to the line y = x.

EXAMPLE 67 Answer the following using the graph on the right: y (7,11)
y = f(x)
a. Does f have an inverse? Why?
(6,6)
b. Draw the graph of the inverse function f –1.
c. Find f –1(-6), f –1(-3), f –1(0), f –1(6), f –1(11). (0,-3) (4,0) x
–1
d. Find the domain and the range of f .
(-10,-6)

Solution a. Applying the horizontal line test we can see that f is one-to-one. So it has an inverse
y

168 Algebra 8
b. We draw the line y = x and reflect f with respect to it to draw the graph of f –1:
y

(11,7)
–1
(0,4) (6,6) y = f (x)
(-3,0)
x

(-6,-10)

c. f –1(6) = –10, f –1(–3) = 0, f –1(0) = 4, f –1(6) = 6, f –1(11) = 7.


d. f –1 can take any x between –6 and 11, inclusive, so the domain is [–6, 11]. The value y of
f –1 can be any number x between –10 and 7, inclusive, so the range is [–10, 7].

Given the formula for a function, to find the inverse we use the following procedure:
1. Verify that the function is one-to-one.
2. Solve the equation y = f(x) for x and interchange x and y in the end.

EXAMPLE 68 Given that f(x) = 2x + 5, answer the following:


a. Does f have an inverse? Why?
b. Find the formula of the inverse function f –1.
c. Find f –1(19), f –1(1).
d. Find the domain and the range of f –1.

Solution a. f is a linear function which is always increasing. That means it is one-to-one and so it has
an inverse.
y5
b. Let y = 2x + 5. Then x = .
2
x5 x5
Interchanging x and y we have the inverse function as y = or f –1(x) = .
2 2
x5 19  5 1 1 5
c. Using the formula f 1( x) = we get f 1(19) = = 7 and f (1) = = 2.
2 2 2
Note that we could also solve equations 2x + 5 = 19 and 2x + 5 = 1 to find f –1(19) and
f –1(1), respectively.

d. Clearly, D( f –1) =  and E( f –1) = .

Functions 169
FINDING THE INVERSE OF A FUNCTION

 To plot the graph of the inverse of a function, reflect the graph of the function with
respect to the line y = x.
 To find the formula for the inverse of a function, solve the equation y = f(x) for x and
interchange x and y.

EXAMPLE 69 If possible draw the graphs of inverses for the following functions:

a. y b. y
y = g(x)
(-3,3)
y = f(x) (2,3)
(0,4)

(-6,0) x (0,0) x

Solution a. Clearly (horizontal line test), f is one-to-one. So the inverse is the symmetry of f with
respect to the line y = x: y
y = f –1(x)

y = f (x)

(0,4)

(-6,0) (4,0) x

(0,-6)

b. g is not one-to-one (horizontal line test). We cannot draw the graph of the inverse
function since it doesn’t exist.

EXAMPLE 70 Given that the following one-to-one functions, find their inverses.
2x – 6
a. f(x) = 4 – 3x b. f ( x) = c. g(x) = x3 – 5
3
Solution Since we know that the functions are one-to-one, we can find their inverses directly

4– y 4– x 4– x
a. Let y = 4 – 3x. Then x = . Interchanging x and y: y = or f –1( x) = .
3 3 3
2x – 6 3y +6 3x +6 3 x +6
b. Let y = . Then x = . Interchanging x and y: y = or f –1( x) = .
3 2 2 2
c. Let y = x3 – 5. Then x = 3 y +5. Interchanging x and y: y = 3 x +5 or g –1( x) = 3 x +5.

170 Algebra 8
EXAMPLE 71 If possible find the inverses of the following functions:
2x + 4
a. f ( x) = b. f ( x) = 3 2 x  1+7 c. f(x) = x2 – 2x
3x  5

2x + 4
Solution a. We can verify that f ( x) = is one-to-one as follows:
3x  5
Let x1  x2. Then assume that f(x1) = f(x2). So,
2 x1 + 4 2 x2 + 4
=
3x1  5 3x2  5
6 x1x2  10 x1 +12 x2  20 = 6 x1x2 +12 x1 10 x2 20
22 x2 = 22 x1
x2 = x1.

But this is not correct since we let x1  x2. This means that if x1  x2, then f(x1)  f(x2).
So f is one-to-one. Now let’s find its inverse:
2x + 4
f ( x) = y =
3x  5
3xy  5y = 2x + 4
3xy  2 x = 4+5 y
x(3y  2) = 4+5 y
4+5 y
x=
3y  2

4+5 x 4+5 x
Interchanging x and y we have y = or f 1( x) = .
3x  2 3x  2
b. We leave the verification of the fact that f is one-to-one to student and proceed to finding
its inverse.

y = 3 2 x  1+7
y  7 = 3 2x 1
( y  7)3 = 2 x  1
( y  7)3 +1
=x
2

( x  7)3 +1 ( x  7) 3 +1
Interchanging x and y we have y = or f 1( x) = .
2 2
c. Choosing x = 0 and x = 2 we can realize that f will give the same value. So f is not
one-to-one. Proof of this using definition is left to the student in exercises. As a result f
does not have an inverse.

Functions 171
Check Yourself 15
1. If f(2) = 3, f(5) = 6, f(6) = –1, find f –1(–1), f –1(3), f –1(6).
y
y = f(x)
2. If possible draw the inverse of f graph of which is given on the right.

2x + 3 (-3,2)
3. Find the inverse of f ( x) = .
7
(0,0) x
Answers (1,-3)
y 7x  3
1. 6, 2, 5 2. 3. f 1( x) =
2
(–3, 1) (0, 0) x

(2, –3)

The functions which are not one-to-one can


have their domains restricted so that they
become one-to-one. As a result their inverses
will then be functions.

EXAMPLE 72 Given that f(x) = x2 + 1,


a. find the largest possible domain so that the inverse exists.
b. find f –1(x) using the new domain.

Solution a. The inverse exists if we have a one-to-one function. Note that for each value of x and its
negative we have the same value of y. So, if we ignore the negative x values, we will have
a one-to-one function. That means to have an inverse, domain must be restricted to [0, ).
b. Let y = x2 + 1 and we will try to solve the equation for x: x2 = y – 1, so x =  y – 1.
If we interchange x and y we have y =  x – 1. Note that the range of the inverse
function is the domain of the original function, that is [0, ). So y = x – 1. Therefore,
1
the inverse function is f ( x) = x – 1.

172 Algebra 8
EXAMPLE 73 Prove that f(x) = x2 – 2x – 2 has an inverse if D( f ) = [1, ) and then find its inverse.

Solution f(x) = x2 – 2x – 2 = x2 – 2x + 1 – 3 = (x – 1)2 – 3.


Here (x – 1)2 is equal to the same number for x – 1 = x0
and x – 1 = –x0 for any x0 > 0.
This fact prevents f(x) = (x – 1)2 – 3 from being a
one-to-one function. But if we guarantee that x – 1 is
never negative, then the function will be one-to-one. And
that is possible when x – 1  0, x  [1, ).
To find the inverse, we solve y = (x – 1)2 – 3 for x:
y = ( x  1)2  3

y + 3 = ( x  1)2

y + 3 =| x  1|

Since x  1  0 we have y+ 3 = x 1 or y+ 3 +1= x.

Interchanging x and y we have f 1( x) = x + 3 +1.

EXAMPLE 74  x +1 
If f 1 
 x
3 –1
 = x , find f (2) + f(8).

x +1
Solution Let us find f –1(2). We don’t have the formula for f –1(x) but for f 1( ). The expression
x
inside the brackets must be equal to 2 since we are looking for f –1(2).
x +1
When we solve = 2, we find x = 1.
x
x +1
That means when x = 1, f 1( ) = f 1(2) =13  1.
x
1 x +1
Finding f(8) is simple. Note that f is inverse of f –1. So if we need f(8), the result of f ( ) = x3
x
x +1 x +1
must be equal to 8, that is f 1( ) = 8, where will be f(8) for the corresponding x value.
x x
x +1 2+1 3
So, x3 = 8, x = 2 and = = .
x 2 2
3 5
Finally, f 1(2)+ f (8) =1+ = .
2 2

Functions 173
EXAMPLE 75 Using the graph below find (f  g–1  f –1)(5) + (f  g  f)(–3).
y

y = f(x)

1 x

y = g(x)

Solution Note that we don’t have the graph of any inverse function and we don’t need them at all. To
find the value of a function just find the y-value for the given x-value on the graph. To find
the value of an inverse function just find the x-value for the given y-value on the graph.
(f  g–1  f –1)(5) = f(g–1(f –1(5))) = f(g–1(4)) = f(0) = 2
(f  g  f)(–3) = f(g(f(–3))) = f(g(0)) = f(4) = 5
So (f  g–1  f –1)(5) + (f  g  f)(–3) = 2 + 5 = 7.

EXAMPLE 76 If (f  g)(x) = 5x – 2 and f(x) = 4x – 3, find g–1(x).

Solution (f  g)(x) = f(g(x)) = 4  g(x) – 3 = 5x – 2


5x +1
Solving the last equation for g(x) we have g( x) =
4
5x +1 4y  1
To find the inverse, we solve y = for x: x =
4 5

1 4x  1
Interchanging x and y we have g ( x) = .
5

EXAMPLE 77 Given f(x) = 2x – 7, find f –1(4x + 1).

Solution Let us find f –1(x): f ( x) = 2 x – 7 = y


y +7
x=
2
x +7
f –1( x) =
2
(4 x +1)+7
Now let us find f –1(4x + 1): f –1( 4x +1) = = 2x+ 4
2

174 Algebra 8
EXAMPLE 78 Given that f(x) = 3x + 1 and g(x) = 2x + 2, find

a. (f  g)–1 b. g–1  f –1

Solution a. Let us find f  g.


y = (f  g)(x) = f(g(x)) = 3(2x + 2) + 1 = 6x + 7
y–7
To find the inverse, x =
6
x–7
(f  g)–1(x) = .
6
b. Let us find f –1 and g–1.
y = f(x) = 3x + 1
y –1
To find the inverse, x =
3
x –1
f –1(x) = .
3
y = g(x) = 2x + 2.

y–2
To find the inverse, x =
2

x–2
g–1(x) = .
2
x–1
–2
x–7
(g–1  f –1)(x) = g–1( f –1(x)) = 3 =
2 6

Note that we have (f  g)–1(x) = (g–1  f –1)(x).

Is it always correct? Why?

Check Yourself 16
1. Given that f(x) = x2 – 4, restrict the domain so that the inverse exists and find its formula.
2. Given that f(2x + 1) = x – 5, find f –1(0).
3. Given that f(g(x)) = 2x + 1 and g(x) = x + 4, find f –1(x).
x –1
4. Given that f(x) = and g(x) = 3x, find (f  g)–1 and g–1  f –1.
4
Answers
x +7 4x +1 4 x +1
1. [0,  ), x+ 4 2. 11 3. 4. ,
2 3 3

Functions 175
EXERCISES 4 .3
A. Basic Operations 5. Find the required values using the given data:
1. Find f + g, f – g, fg, f / g for the following functions. 3x +5 x+ 2
a. f ( x) = and g( x) = ,
a. f(x) = x + 1, g(x) = x – 1 2 x +1 x 1
b. f(x) = x3 + 3x2, g(x) = x2 + 5x ( f  g  f )(0) = ?

c. f ( x) = x + 3, g( x) = x + 2  x +1 
b. f(2x + 1) = 3x – 1 and g  2
 = x +1,
 x 1
d. f ( x) = x  1, g( x) = x +1 ( f  g)(–1) = ?
x+ 3 x2
c. f (3x  1) = and g( x+1) = ,
2x  1 x 1
1 ( f  g)(1) = ?
2. If f ( x) = 3x + 4, g( x) = x 2 + x, h( x) = , find
x
2 x +1 if x > 2
the following functions. d. f ( x) =  and
 x  2 if x  2
 
a. (f + g)(3) b.  f  h  ( 1)  x if x > 0
 g  g( x) = 
c. (hg – f)(4)  x if x  0
( f  g  f )(–2) = ?

B. Composition of Functions 2 x +1
6. If f(x) = x3 – 3x2 + 3x – 1 and g( x) = ,
x+ 3
3. Find f  f, f  g, g  f, g  g for the following
solve ( f  g)(x) = 0.
functions.

x +1 7. If ( f  g)(x) = 5g(x) + 4 and


a. f ( x) = , g( x) = x2 + x
x 1 (g  f )(x) = 3f(x) – 1,
2
b. f(x) = x – 2x + 1, g(x) = x + 1 find a such that ( f + g)(a) = 19.
2x + 3 x+ 3
c. f ( x) = , g( x) =
x 3x  2  
1 x
8. If f ( x) = , find  f  f  ...  f  ( x).
 1+ x   
 2004 times 
4. Express h(x) in terms of f(x) and g(x) such that
C. Inverse of a Function
h(x) = (f  g)(x).
9. State whether the following relations are one-to-one
a. h( x) = 2 x  4 functions or not.

 x4
7 a. {(x, 1), (y, 1), (z, 2), (k, 3)}
b. h( x) =  
 5  b. {(1, 1), (2, 2), (3, 3), (4, 1)}
2 3 2
c. h(x) = (2x – x) – (2x – x) + 4 c. {(a, b), (b, a), (c, d), (d, c)}

176 Algebra 8
10. State whether the following functions are one-to-one 13. Plot the graphs of inverse functions for the
or not. following functions, if possible:

a. f(x) = x2 – 1, D( f ) = {0, 1, 2, 3} a. y b. y
(4,5) (0,4)
b. f(x) = x2, D( f ) = +
c. f(x) = x2
d. f(x) = 3x + 5 x x
(2,-2)
x 1 (-8,-3) (-5,-5)
e. f ( x) = (5,-5)
x2
c. y d. y
(5,7) (-6,7)

(-5,4) (0,3)
(4,1)
(-2,-1) x (0,0) x
(2,-3)
11. State whether the following graphs belong to
(-6,-3)
one-to-one functions or not. (6,-5)

a. y b. y e. y f. y

(-5,3) (0,5)

x x x x
(4,-2)
(6,-5)

c. y

14. Find the required values using the given data for
x the functions with the inverse.
2 x +5 1
a. f ( x) = , f (3) = ?
5
b. f –1(x) = x2 – 3, D(f –1) = [0, ), f(–3)= ?
f ( x)+1 1
c. 2 f ( x)  1= , f (2) = ?
12. Find the inverses of the following one-to-one x2
functions: 4 x  1 1
d. f (3x +1) = , f (3) = ?
x2
3x + 4
a. f(x) = 6 – 4x b. f ( x) = 3 
x 1 e. f(x2 + x + 1) = x – 2, D( f ) =  ,   ,
4 
c. f(x) = 4x3 – 5 d. f ( x) = 3  5 x  1+9 f –1(0) = ?

Functions 177
15. Find the following using the given data for 18. Find the following using the graph of functions
functions with inverses: 
below:
 2 x +1  y = f –1(x) y
1
a. f   = x +1, f ( x) = ?
 x 1 
3x  2 1
b. f ( x  1) = , f ( x) = ?
x + 21 1

 3x  5  1 1 x
c. f   = x, f (2 x) = ?
 x +1  y = g(x)

Mixed Problems
a. (g  f –1)(–2) + ( f –1  g  f )(2)
16. Find the following using the given data for b. ( f –1  g)(2) + ( f  g)(3)

functions with inverses:

a. f(x) = 3x + 1, g(x) = 2x + 3, (f  g–1)(x) = ? 19. Prove that



1 x+ 2 a. f(x) = x2 – 2x is not one-to-one.
b. f ( x) = , g( x) = , ( g 1  f )( x) = ?
x 2 x +1 b. f(x) = 2x3 + 4 is one-to-one.
2 x +1 1 3x +1
c. f 1( x) = , g ( x) = , ( f 1  g1 )( x) = ?
x3 x 2 20. If f and g are even functions such that
x  2 1 2 f(0) = g(4) = –2, g(0) = f(4) = 4,
d. f 1( x) = , g ( x) = , ( f  g)1( x) = ?
x x2 f(–2) = g(–2) = 0, find
e. f(x) = 2x + 3, (g  f )(x) = 4x – 5, g(x) = ? ( g  f )(4)+( f  g)(–4)+( g  f )(–4)
.
( f  g)(2)+( g  f )(2)+( f  g)(4)
3x +1
f. g( x) = , ( f  g)( x) = 2 x+ 3, f 1( x) = ?
x 1
2 x +1 21. Prove that if f and g are even, then f + g, f – g, fg,
g. g( x) = , ( g 1  f )( x) = 2 x  1, f 1( x) = ? 
x f / g are also even.

17. Find the required values using the given data for 22. Prove that if f and g are odd, then f + g, f – g are

functions with inverses: 
odd and fg is even.
–1
a. f(x) = x + 1, g(x) = 3x – 4, ( f  g)(2) = ?
b. f(x) = x – 2, g(x) = 2x + 1, ( f  g–1)(3) = ?
2
23. Find the inverses of the following functions:

x+ 3
c. f ( x) = , g( x) = 6x +1, ( f  g)1(2) = ? a. f(x) = x2 + 6x, D( f ) = (–, –3]
x2
b. f(x) = x2 – 8x + 5, D( f ) = [4, )
2x + 3  1  2 x +5 
d. f   = x, g   = 3x  2,
 x +1   x+ 3 
24. Prove that (g  f )–1 = f –1  g–1.
1
( f  g )(3) = ? 

178 Algebra 8
Cryptography is the art of secret writing. Its aim is to protect information by
transforming it into a coded language which unwanted eyes are unable to
use. This transformation, however, must be done in a reversible way so that
individuals intended to view the information may do so.
Some of the classical methods of cryptography are as follows (However, none
of them are used today, because they are considered either insecure or
impractical).

Simple Substitution:
When we encode the message MATH IS COOL , it becomes
NBUIAJTADPPM. Here, the rule is that every letter is substituted by the
letter that follows it. That is, A by B, B by C, “space” by A, etc.

Mixing:
Take a key sequence consisting of the first few natural numbers in mixed
order, for example 3, 5, 1, 2, 4. The rule is that the first letter will move to
the 3rd place, the second to 5th place, the third to 1st place, the fourth to
2nd place, the fifth to 4th place, and so on for the next five letters.
So MATH IS COOL becomes THM A CIOS OL.
Note that the “space” character is also treated as a letter.

Method of Vigenere:
Pair each letter with a number with its order in the alphabet as follows:

A B C D E F G H I J K L M N O P Q R S T U V W X Y Z
1 2 3 4 5 6 7 8 9 10 11 12 13 14 15 16 17 18 19 20 21 22 23 24 25 26 27
Functions 179
Our message MATH IS COOL becomes 13 1 20 8 27 9 19 27 3 15 15 12
Choose a key word, for example, FUNCTION which is 6 21 14 3 20 9 15 14.
Now, add 6 to the first letter of our encoded message, 21 to the second, 14 to the
third, and so on.
The new message will be read as 19 22 34 11 47 18 34 41 9 36 29 15.
If the number is more than 27, divide it by 27 and take the remainding
number. Finally, the message will become SVGKTRGNIIBO. Find the pair for
these numbers from the table above. The whole procedure is summarized in
the table below.

13 1 20 8 27 9 19 27 3 15 15 12 convert the message using table.

convert the keyword and write it as the


6 21 14 3 20 9 15 14 6 21 14 3
same length as the original message.

19 22 34 11 47 18 34 41 9 36 29 15 add the top two rows.

divide the previous row by 27 and


19 22 7 11 20 18 7 14 9 9 2 15
write the remainding number.

Note that all of these methods are functions that are defined from a limited
domain to a range. We can invent even more complicated methods by
applying a few of them together for the same message (Think about
the composition of functions!). Here it is very important to know the rules.
To decode a crypted message we just apply the rule in reverse order (Think
about the inverse of a function!).
Below is an encoded message. We first applied the Method of Vigenere with
the key word CAT (each letter is paired by its order in the alphabet as “space”
being the 27th letter), and then we applied the simple substitution.
GQHKTVXWFEVCSPM
Try to decode it!
What will happen if the rule is a function that has no inverse? What happens
if we assign the same number to two different letters? If a few of the rules are
applied in order, by which rule should we start the decoding procedure? Try
to develop your own method of encryption.
180 Algebra 8
Definition quadratic function
A function f(x) is a quadratic function if
y = f(x) = ax2 + bx + c, a0
where a, b, and c are real numbers.

For example, f(x) = 2x2 + 3x + 1, y = x2 – 1, y = –ñ2x2 and f(x) = 2x – x2 are all quadratic
functions. y = 3x – 4 is not a quadratic function, because a = 0.

The domain of a quadratic function is the set of all real numbers.

The graph of a quadratic function f(x) = ax2 + bx + c is called a parabola.

a>0 a<0

y axis of symmetry y V

c
The sign of a has no
effect on the size or
shape of the parabola,
the sign of a determines x-intercept
whether the parabola x1 x2
opens upward or x1 x2 x x
downward.
x-intercept

c
vertex
y-intercept V

a>0 a<0 When a > 0, the parabola opens upward. When a < 0, the parabola opens downward.

The point V in the parabolas above is called the vertex of the parabola. The vertex is the
lowest or the highest point of the parabola.
The vertical line drawn through the vertex is called the axis of symmetry. It divides the curve
into two symmetrical halves.
The points x1 and x2 are called the x-iintercepts of the parabola. They are the zeros of the
function.
The point c is called the y-iintercept of the parabola.

182 Algebra 8
A. GRAPHING y = ax2
How can we find the graph of the quadratic function y = ax2? If we have the function, we can
plot the graph by making a table of values. To find the values, we substitute different values
of x into the equation to obtain the corresponding y values. These x and y values provide the
coordinates for points which we can plot to form the shape of the graph.
Let us graph the function y = ax2.
If a > 0, we get the table of
x – –2 –1 0 1 2 +
ordered pairs opposite.
y + 4a a 0 a 4a +
Then we plot the points
on a graph and draw a
parabola through them.
y
4a

3a

2a

-2 -1 1 2 x

y = ax2, a > 0

We can see that the vertex of the parabola is at the origin (0, 0), and the axis of symmetry
lies along the y-axis (the line x = 0).
If a < 0, we get a different
x – –2 –1 0 1 2 +
set of ordered pairs.
y – 4a a 0 a 4a –

-2 -1 1 2 x
a

2a

3a

4a

Quadratic Functions 183


EXAMPLE 1 Sketch the graphs of the functions.

a. y = x2 , y = 2x2 and y = 1 x2 b. y =  x2 , y = 2x2 and y =  1 x2


2 2

Solution a.
x – –2 –1 0 1 2 

As |a| increases, the y = x2  4 1 0 1 4 


parabola becomes
narrower. As |a|
decreases, the parabola y = 2x2  8 2 0 2 8 
becomes wider.
1 2 1 1
y= x  2 0 2 
y 2 2 2
ax2 x2
1 x2
2 x2 2x2 y

3
a>1 x
y x2 ax2 2

-4 -3 -2 -1 1 2 3 4 x

0<a<1 x

b.
x – –2 –1 0 1 2 

y = –x2 – –4 –1 0 –1 –4 –
y= ax2 + bx
y y y = –2x2 – –8 –2 0 –2 –8 –

1 2 1 1
y= – x – –2 – 0 – –2 –
2 2 2
–b x –b x
a a
a>0
y
y y -4 -3 -2 -1 1 2 3 4
x

–b x –b x
a a
a<0

–x2 – 1 x2
–2x2 2

184 Algebra 8
Check Yourself 1
Graph the functions.

1. y = 3x2 2. y = 1 x2 3. y = –3x2 4. y = – 1 x2
3 3

B. GRAPHING y = ax2 + bx + c
1. Vertex Point
Let x1 and x2 be zeros of the polynomial ax2 + bx + c.
a>0 a<0
b Then ax2 + bx+ c = a(x  x1 )(x  x2 )
–
2a
b 
– b+    b   
2a = a  x    x  
 2a   2a 
b
–
2a
  b     b+   b 2   
= a  x2  x  
  x  2a + 4a2 


b   2a    
–
2a

b  xb+ x  + xb  x  b2  
– = a  x2 + + 2  2 
2a 2a 4a 4a
 

 bx b 2  
= a  x2 + + 2  
 a 4a  4a
b
–
2a
2
 b  
= a  x+  
 2 a  4 a

b 
= a(x  h)2 + k where h =  and k =  .
2a 4a

This is called the completed square form of the quadratic function.


If a quadratic function has
two known x-intercepts,
we can find the COMPLETED SQUARE FORM
x-coordinate of the vertex
point by averaging them. The completed square form of a quadratic function is f(x) = a(x – h)2 + k.
x1 + x2 b The vertex of a parabola of a function in complete square form is the point (h, k).
h= =–
2 2a

b 4ac  b 2
At the vertex point V(h, k), h =  and k = f(h) or k = .
2a 4a
Since (h, k) is the vertex of a parabola, then the x-coordinate of the vertex point is the equa-
tion of the axis of symmetry, x = h.

Quadratic Functions 185


If a > 0, then the y-coordinate of the vertex is the minimum value of the function. If a < 0,
then the y-coordinate of the vertex is the maximum value of the function.
a>0 a<0
maximum value of
y y the function
D V(h,k)
–
4a
b
–
2a
x
The trajectory of a shell
fired from a cannon is a
parabola. To reach the b x
–
maximum value range with 2a
a cannon, it is shown in D
calculus that the barrel of –
4a V(h,k)
the cannon must be set at
45°. minumum value
of the function

EXAMPLE 2 Find the coordinates of the vertex point of each function. Write the equation of the axis of
symmetry, and determine the maximum or minimum value of the function.

a. y = x2 – 2x – 3 b. y = –x2 + 4x + 5 c. y = x2 + 3x + 1

Solution a. V(h, k) is the vertex point.


b 2
h  1
2a 2
k = f(h) = 12 – 2  1 – 3 = –4
So the vertex is V(1, –4).
The equation of the axis of symmetry is x = 1.
Since a = 1 > 0, the vertex is the minimum value and k = –4 is the minimum value of
the function.

b 4
b. h =  = =2
2a 2  (–1)
k = f(h) = –22 + 4  2 + 5 = 9
So the vertex is V(2, 9).
The equation of the axis of symmetry is x = 2.
Since a = –1 < 0, the vertex is the maximum value and k = 9 is the maximum value of
the function.

186 Algebra 8
b 3 3
c. h =  = =
2a 2 1 2
2
 3  3 5
k = f (h) =    + 3     + 1 = 
 2  2 4

 3 5
So the vertex point is V   ,   .
 2 4
3
The equation of the axis of symmetry is x =  .
2
5
Since a = 1 > 0, the vertex is the minumum value and y =  is the minimum value of
4
the function.

EXAMPLE 3 The line 3x + 1 = 0 is the axis of symmetry of the parabola y = –2x2 + mx – 1.


Find the maximum value of the function.

1
Solution 3x + 1 = 0 ; x =  is the equation of axis of symmetry.
3
1 b m 4
h=  =  = ; m= 
3 2a 2  (–2) 3

Since a = –2 < 0, the vertex is the maximum value.


2
 1 4 1 7
k = f (h) =  2         1 = 
 3 3 3 9

7
So k =  is the maximum value.
9

EXAMPLE 4 The minimum value of the function y = x2 – 8x + n is –4. What is the value of n?

b 8
Solution Given k = –4 and h =  = = 4 ; h = 4.
2a 2 1
k = f (h) ;  4 = 42  8 4 + n ; n = 12

Check Yourself 2
Find the vertex of each parabola.

1. y = x2 – 10x + 20 2. y = 1 x2 + x – 8 3. y = 4 – x2 4. y = 4x + x2
2
Answers
1. (5, –5) 2. (–1, –8.5) 3. (0, 4) 4. (–2, –4)

Quadratic Functions 187


2. Intercepts
Consider the function y = f(x) = ax2 + bx + c, a  0.
If x = 0, then y = c. The point (0, c) is called the y-intercept of the graph. We mean that the
parabola intersects the y-axis at this point.
If y = 0, then ax2 + bx + c = 0. Let x1 and x2 be roots of this equation. The points (x1, 0)
and (x2, 0) are called the x-intercepts of the graph. We mean that the parabola intersects the
x-axis at these points. There are three possibilities for the x-intercepts.
1. If  > 0, the parabola intersects the x-axis at two distinct points.
2. If  = 0, the parabola is tangent to the x-axis.
3. If  < 0, the parabola does not intersect the x-axis.

D<0

D=0 a<0
D>0

x
D>0

a>0 D=0

D<0

EXAMPLE 5 Find the x- and y-intercepts of each function.

a. y = x2 – 9 b. y = –4x2 + 5x c. y = 3x2 + 4x – 2
d. y = x2 + 3x + 7 e. y = 4x2 + 20x + 25

Solution a. x = 0 ; y = –9
So (0, –9) is the y-intercept.
y = 0 ; x2 – 9 = 0
x1 = –3, x2 = 3
So (–3, 0) and (3, 0) are the x-intercepts.
b. x = 0 ; y = 0
So (0, 0) is the y-intercept.
5
y = 0 ; –4x2 + 5x = 0 ; x1 = 0 , x2 =
4
5 
So (0, 0) and  , 0  are the x-intercepts.
4 

188 Algebra 8
c. x = 0 ; y = –2
So (0, –2) is the y-intercept.
y = 0 ; 3x2 + 4x – 2 = 0
 = 42 – 4  3  (–2) = 40

4  2 10 2  10 4 + 2 10 2 + 10
x1 = = , x2 = =
6 3 6 3
2  10 2 + 10
So ( , 0) and ( , 0) are the x-intercepts.
3 3

d. x = 0 ; y = 7
So (0, 7) is the y-intercept.
y = 0 ; x2 + 3x + 7 = 0
 = 32 – 4  1  7 = –19 < 0
So the parabola does not cut the x-axis, i.e. there are no x-intercepts.
e. x = 0 ; y = 25
So (0, 25) is the y-intercept.
y = 0 ; 4x2 + 20x + 25 = 0
 = 202 – 4  4  25 = 0
20 5
x1 = x2 =  =
8 2
 = 202 – 4  4  25 = 0
5
So the parabola is tangent to the x-axis at the point (– , 0).
2

EXAMPLE 6 The function y = x2 – (m + 2)x + 5 + m is given. Find the value(s) of m for which the
parabola

a. does not cut the x-axis. b. is tangent to the x-axis.


c. intersects the x-axis at two distinct points.

Solution  = (m + 2)2 – 4(5 + m)


= m2 – 16
 = m2 – 16 = 0 ; m = 4
Let us consider the sign of .
m –¥ –4 4 ¥

m2 – 16 + – +

Quadratic Functions 189


a. If m  (–4, 4), the parabola does not cut the x-axis.
b. If m = 4, the parabola is tangent to the x-axis.
c. If m  (–, –4)  (4, ), the parabola cuts the x-axis at two distinct points.

Check Yourself 3
Find the x- and y-intercepts of each parabola.

1. y = –x2 + 3x – 4 2. y = 3x2 – 6x + 3 3. y = 3x – x2
Answers

1. no x-intercepts, (0, –4) 2. (1, 0), (0, 3) 3. (0, 0), (3, 0)

3. Sketching a Graph
To sketch the parabola y = ax2 + bx + c, follow the steps.
1. Check a.
When a > 0, the graph has a minimum point and the parabola opens upward.
When a < 0, the graph has a maximum point and the parabola opens downward.
2. Find the coordinates of the vertex point, V(h, k).
3. Find the x- and y-intercepts.
4. Sketch the graph of the parabola.

EXAMPLE 7 Graph the functions.

a. y = x2 – 3x – 10 b. y = –2x2 – 8x c. y = x2 + 4x + 5
d. y = –3x2 + 12 e. y = x2 – 2ñ2x + 2

Solution a. a = 1 > 0, so the parabola opens upward. y


3
b 3 2
h=  = ,
2a 2 -2 5 x

3 2 3 49
k = f (h) = ( )  3   10 =  ;
2 2 4 f(x)=x2 – 3x – 10

3 49
V( , – ) is the vertex point. -10
2 4 49
-
4 V
x = 0 ; y = –10, (0, –10) is the y-intercept.
y = 0 ; x2 – 3x – 10 = 0 ; x = –2, x = 5 ;
(–2, 0) and (5, 0) are the x-intercepts.

190 Algebra 8
b. a = –2 < 0, so the parabola opens downward. y
V
8 8
h=  = 2,
2  (–2)
f(x)=–2x2 – 8x
k = f (h) = 2(–2)  8 (–2) = 8 ;
2
V(–2, 8)

x = 0 ; y = 0, (0, 0) is the y-intercept.


y = 0 ; –2x2 – 8x = 0 ;
x = 0, x = –4 ; (–4, 0) and (0, 0) are the x-intercepts. -4 -2 x

c. a = 1 > 0, so the parabola opens upward. y

4
h=  = 2, f(x)=x2+4x+5
2 1 5

k = f (h) = (–2)2 + 4 (–2) 2 + 5 = 1 ; V(–2, 1)

x = 0 ; y = 5, (0, 5) is the y-intercept. 1


V
2
y = 0 ; x + 4x + 5 = 0 ; -2 x

 = 42 – 4  1  5 = –4 < 0 ; the parabola does not cut the x-axis.

d. a = –3 < 0, so the parabola opens downward. y

0 V 12
h=  = 0,
2(  3)

k = f (h) = 3(0)2 +12 = 12 ; V(0, 12)


f(x)=–3x2+12
x = 0 ; y = 12, (0, 12) is the y-intercept.
y = 0 ; –3x2 + 12 = 0 ;
x = 2 ; (–2, 0) and (2, 0) are the x-intercepts.

-2 2 x

e. a = 1 > 0, so the parabola opens upward. y

2 2 f(x)=x2 – 2ñ2x+2
h=  = 2
2 1
2
k = f (h) = ( 2 )2  2 2  2 + 2 = 0 ; V( 2, 0)
V
x = 0 ; y = 2, (0, 2) is the y-intercept. ñ2 x
2
y = 0 ; x – 2ñ2 + 2 = 0 ;
x1 = x2 = ñ2 ; the parabola is tangent to the x-axis at the point (ñ2, 0).

Quadratic Functions 191


EXAMPLE 8 Sketch the graph of the function y = –x2 – 2x + 8 for |x|  3. Find the range of the function.

Solution a = –1 < 0, so the parabola opens downward. y

2 9
h=  = 1, 8
2  (–1)
k = f (h) = (–1)2  2 (–1) + 8 = 9 ; V(–1, 9) 5

x = 0 ; y = 8, (0, 8) is the y-intercept.


y = 0 ; –x2 – 2x + 8 = 0 ; -4 3
-3 -1 2 x
x1 = –4, x2 = 2 ; (–4, 0) and (2, 0) are the x-intercepts.
|x|  3 ; –3  x  3
x = –3 ; y = –9 + 6 + 8 = 5, so (–3, 5) is on the parabola.
x = 3 ; y = –9 – 6 + 8 = –7, so (3, –7) is on the parabola. -7
So the range of the function is [–7, 9].

EXAMPLE 9 Find the values of p for which px2 + 4x + p is greater than zero for all real values of x.

Solution px2 + 4x + p > 0 for all real numbers x. y = px2+ 4x+p


This is possible only if p > 0 and  < 0.
p>0 (1)
2 2
4 – 4p < 0 (2)
x
(1) p > 0
(2) 42 – 4p2 < 0 ; 42 – 4p2 = 0 ; p = 2
p –¥ –2 0 2 ¥

(1) – – + +

(2) – + + –

system

So p  (2, ).

Check Yourself 4
Sketch the graph of each function.

1. y = x2 – 5x + 4 2. y = –x2 + 4x – 3 3. y = 4x2 – 20x + 25 4. y = 3x2 – 2x

192 Algebra 8
Answers
1. y 2. y 3. y 4. y

4 25
1 3 x

2/3 x
-3
1 4 x 5/2 x

4. Shifting Graphs
Let y = f(x) be a function.
Vertical Shifting
k > 0 shift the graph of y = f(x) k units upward. (see Figure 1)
y = f (x ) + k 
k < 0 shift the graph of y = f(x) |k| units downward. (see Figure 1)

Horizontal Shifting
 h > 0 shift the graph of y = f(x) h units to the left. (see Figure 2)
y = f (x+ h) 
 h < 0 shift the graph of y = f(x) |h| units to the right. (see Figure 2)

Reflection
y = –f(x) Reflect the graph of y = f(x) in the x-axis. (see Figure 3)
Horizontal and Vertical Shifting
y = f(x + h) + k Shift the graph of y = f(x) |k| units upward or downward and |h| units
to the left or to the right.
k>0 h>0
y x2+k y y

x2 (x+h)2 x2 (x – h)2 x2

k x2 – k

0 x -h 0 h x 0 x

-k
–x2

Figure 1 Figure 2 Figure 3

EXAMPLE 10 Sketch the graph of the each function using the shifting method.

a. y = x2 + 3 b. y = x2 – 3 c. y = (x – 3)2 d. y = (x + 3)2
e. y = –(x – 4)2 f. y = (x + 2)2 – 3 g. y = –(x – 2)2 + 1

Quadratic Functions 193


Solution a. y b. y c. y d. y
x2+3

x2 x2 x2 (x – 3)2 (x+3)2 x2

3 x2 – 3

0 x 0 x 0 3 x -3 0 x

-3

e. y f. y g. y

x2 (x+2)2 x2
x2

(x+2)2 – 3 1
4
0 x -2 0 x 0 2 x

-3
–(x – 4)2
–x2 –(x – 2)2+1
y

Check Yourself 5
Sketch the graph of the each function by shifting.

1. y = x2 + 4 2. y = –(x + 1)2 3. y = (x – 2)2 – 2 4. y = –(x + 2)2 + 1


Answers
1. y 2. y 3. y 4. y
x2+4 x2 (x–2)2
-1 1
x2 -1 x
4 (x–2)2–2 -2 x
4 -3
2 –(x+2)2+1
–(x+1)2 –x2 2 -4
x x
-2
–(x+2)2 –x2

5. Parabolas with Absolute Value (Optional)

EXAMPLE 11 Sketch the graph of each function.


a. y = |x2 – 3x + 2| b. y = –x2 + 2|x| + 3
c. y = x|x + 2| d. y = |2x2 – 12x + 16| + 2

194 Algebra 8
Solution a. First graph the function y = x2 – 3x + 2. y

Then, take above the part of the graph which is below 2

the x axis, symmetric with respect to x-axis. f(x)=|x2 – 3x+2|


a = 1 > 0, so the parabola opens upward. 1

3 3 1/4
h=  = ,
2 1 2 1 3/2
2 x
-1/4
2
 3 3 1
k = f (h) =    3  + 2 =  ;
2 2 4

3 1
V  ,   is the vertex point.
2 4

x = 0 ; y = 2, (0, 2) is the y-intercept


y = 0 ; x2 – 3x + 2 = 0 ;
x = 1, x = 2 ; (1, 0) and (2, 0) are the x-intercepts.

b. y = –x2 + 2|x| + 3
Case 1
____________________________________________________________________
If x  0, |x| = x ; y = –x2 + 2x + 3
a = –1 < 0, so the parabola opens downward.

2
h=  = 1, k = f (1) = 12 + 2 1+ 3 = 4 ; V(1, 4) is the vertex point.
2  (–1)
x = 0 ; y = 3, (0, 3) is the y-intercept.
y = 0 ; –x2 + 2x + 3 = 0 ;
x = –1, x = 3 ; (–1, 0) and (3, 0) are the x-intercepts.
Case 2
____________________________________________________________________
If x < 0, |x| = –x ; y = –x2 – 2x + 3 y
4 f(x)=–x2+2|x|+3
a = –1 < 0, so the parabola opens downward.
3
2
h=  = 1,
2  (–1)

k = f (–1) = (–1) 2  2 (–1) + 3 = 4 ; -3 -1 1 3 x

V(–1, 4) is the vertex point.


x = 0 ; y = 3, (0, 3) is the y-intercept.
y = 0 ; –x2 – 2x + 3 = 0 ; x = –3, x = 1 ; (–3, 0) and (1, 0) are the x-intercepts.

Quadratic Functions 195


c. y = x|x + 2|
Case 1
________________________________________________________________________
If x + 2  0, x  –2, |x + 2| = x + 2 ; y = x2 + 2x
a = 1 > 0, so the parabola opens upward.

2
h=  = 1, k = f (–1) = (–1) 2 + 2 (–1) = 1 ; V(–1, –1)is the vertex point.
2 1
x = 0 ; y = 0, (0, 0) is the y-intercept.
y = 0 ; x2 + 2x = 0 ; x = –2, x = 0 ; (–2, 0) and (0, 0) are the x-intercepts.

Case 2
________________________________________________________________________
If x + 2 < 0 ; x < –2, |x + 2| = –(x + 2) ; y
2
y = –x – 2x
2 f(x)=x|x+2|
a = 1 > 0, so the parabola opens upward.
1
2
h=  = 1, -1
2  (–1) -3 -2 x
-1
k = f (–1)  (–1) 2  2 (–1)  1 ;

V(–1, 1) is the vertex point.


x = 0 ; y = 0, (0, 0) is the y-intercept.
y = 0 ; –x2 – 2x = 0 ; x = –2, x = 0 ; (–2, 0) and (0, 0) are the x-intercepts.

d. y = |2x2 – 12x + 16| + 2 y


2 f(x)=|2x2 – 12x+16|+2
First graph the function y = |2x – 12x + 16|. 18
Then shift this graph 2 units up. 16

a = 2 > 0, so the parabola opens upward. y=|2x2 – 12x+16|

12
h=  = 3,
22
k = f (h) = 2(3)2  12  3 +16 = 2 ;
V(3, –2)is the vertex point.

x = 0 ; y = 16, (0, 16) is the y-intercept. 4

y = 0 ; 2x2 – 12x + 16 = 0 ; 2

x = 2, x = 4 ; 2 3 4 x
(2, 0) and (4, 0) are the x-intercepts. -2

196 Algebra 8
Check Yourself 6
Sketch the graph of each function.
1. y = |x2 – 6x + 5| 2. y = –x|x| + 4
Answers
1. y 2. y

4 4

2
x
1 3 5 x

C. EQUATION OF A PARABOLA
We have learned how to construct a parabola if we are given its equation. But if we are given
the graph of a parabola, how can we write its equation?
There are three different approaches to finding the equation of a parabola. The approach we
use depends on the information we know.

1. If we know the vertex point of the parabola, V(h, k) then we use the formula y = a(x – h)2 + k
to write the equation of the parabola. We need to know another point on the parabola to
write the equation.

2. If we know the x-intercepts of the parabola then we use the formula y = a(x – x1)(x – x2)
to write the equation of the parabola. We need to know another point on the parabola to write
the equation.

3. If we know any three points on the parabola, then we use the formula y = ax2 + bx + c
to write the equation of the parabola.

EXAMPLE 12 Write the equation of each parabola.

a. y b. y c. y

4 y=g(x)
2 y=f(x)

3 y=h(x)
2 x 2

-3 1 x 3 4 x

Quadratic Functions 197


Solution a. The vertex point of the parabola is V(2, 2), so y = a(x – 2)2 + 2.

Also, the parabola passes through the origin (0, 0), so this point satisfies the equation of
the parabola.

1
0 = a(0  2)2 + 2 ; 0 = 4 a+ 2 ; a= 
2
1
y =  (x  2)2 + 2
2
1
y =  x2 + 2 x
2
This is the equation of the parabola.

b. Let y = ax2 + bx + c.
Since the points (0, 3), (–3, 3) and (1, 4) are on the graph of the parabola, these points
satisfy the equation of the parabola.

 b  b2  4ac
(0, 3), 3 = c
2a
(–3, 3), 3 = 9 a  3 b+ 3 ; 9a 3b = 0

(1, 4), 4 = a+ b+ 3 ; a+ b = 1

9a  3b = 0 (1)

 a+ b = 1 (2)

1 3
From (1) and (2), a= , b= .
4 4
1 2 3
So the equation is y = x + x+3 .
4 4

c. Let y = a(x – 3)(x – 4)

(0, 2) is on the parabola.

1
2 = a(0  3)(0  4) ; a=
6
1
y = (x  3)( x  4)
6
1 2 7
So the equation is y = x  x+2 .
6 6

198 Algebra 8
EXAMPLE 13 x1 and x2 are the roots of the parabola below. Find the value of x1 + x2.
y

3 y=g(x)

-6 x1 x2 2 x

-2

Solution y g(2) = g(–6) = 3

B 3 y=g(x) AB = BC, so the x-coordinate of the vertex point is


A C
6 + 2
h= = 2.
2
-6 x1 x2 2 x b x + x2
h = ; 1 = 2 ; x1 + x2 = 4.
-2 2a 2

EXAMPLE 14 A parabolic concrete bridge support needs


to pass through the points (–50, 0),
(0, 30), and (50, 0). Write the equation of
the parabola formed by the bridge.

Solution Let y = ax2 + bx + c. Since the points (–50, 0), (0, 30), and (50, 0) are on the graph of the
parabola, these points satisfy the equation of the parabola.
(0, 30), c = 30
(–50, 0), 0 = 2500a – 50b + 30 ; 250a – 5b = –3 (1)
(50, 0), 0 = 2500a + 50b + 30 ; 250a + 5b = –3 (2)
From (1) and (2),
–3
a= , b = 0.
250
3 2
So the equation is y = – x +30 .
250

Quadratic Functions 199


EXAMPLE 15 The shape of the famous “Gateway to the West” arch, which is
the spirit of the City of St. Lois can be modeled by a parabola.
1 2 21
The equation for the parabola is y = – x + x.
150 5
a. Sketch the graph of the arch's equation on a coordinate axis.
b. What is the distance from one side of the arch to the other?

1
Solution a. a = – < 0, so the parabola opens downward. y
150 V
661.5
21
h= – 5 = 315,
1
2 ( – )
150
1 21 630
k = f (315) = – 315 2 +  315 315 x
150 5
= 661 .5 V(315, 661.5)
x = 0 ; y = 0 (0, 0) is the y-intercept.
1 2 21
y=0 ; – x + x=0 ;
150 5
x = 0, x = 630 ; (0, 0) and (630, 0) are the x-intercepts.
b. From the x-intercepts, we can see that the distance along the bridge is 630 m.

EXAMPLE 16 A farmer has 120 m of fencing. He wants to put


a fence around three sides of a rectangular plot of
land, with the side of a barn forming the fourth
side. Find the maximum area the farmer can
enclose. What dimensions give this area?

Solution Let x represent the width of the plot. Then, since


there are 120 m of fencing,
x + x + length = 120 ; length = 120 – 2x.
The area of the plot is A = (120 – 2x)  x = 120x – 2x2.
We can see that a < 0, so the graph of the function of the area has its maximum value at the
vertex point.
To make the area as large as possible, let us therefore find the vertex of the graph of the function
A = –2x2 + 120x.
–b 120
The x-coordinate of the vertex is =– = 30. This is the width of the plot at its maximum
2a –4
area. Now we have the solution: A = –2  302 + 120  30 = 1800 m2 and the dimensions of
the plot are 30 m by (120 – 2  30)m, i.e. 30 m by 60 m.

200 Algebra 8
EXAMPLE 17 Find the maximum vertical distance d between the parabola and the line in the figure.

The distance between the vertex point of the parabola y


Solution f(x)=–2x2+4x+3
and the midpoint of the AB line segment in the figure 5

will be the maximum distance. 4


f(x)=x – 2
b –4 3
So h = – = =1
2 a –4 2 d
B
k = f (1) = 5 , V (1, 5). 1

Let us find the coordinates of point A and point B. -2 -1 1 2 3 4 5 x


2 -1
–2 x + 4x+ 3 = x – 2
-2
2 x2 – 3x – 5 = 0
A -3
5
(2 x – 5)(x+1) = 0 ; x = , x = –1 -4
2
5 1
A(–1, –3) and B( , )
2 2 A M B
5 1 (–1, –3) ( 5 , 1)
–1+ –3 +
M( 2, 2 ) = ( 3 , –5 ) 2 2
2 2 4 4
3 –5 1 625 626 626
So d = ( – 1)2 +( – 5) 2 = + = = .
4 4 16 16 16 4

Check Yourself 7
Write the equation of each parabola.

1. y 2. y 3. y
3 4
3
x
2

-3 -2 x
-1 3 x
-4
-1

Answers
1 2 2 1 2 3 1 2
1. y = x – x–1 2. y = – x + x–3 3. y = – x – x+ 2
3 3 4 4 4

The things of this world cannot be made known without a knowledge of mathematics.

Quadratic Functions 201


P ARABOLIC R EFLECTORS
Parabolic curves are used in the design of lighting systems, telescopes, and radar antennas, mainly because of the
reflective property you can see in the figures below.

axis axis

Figure a . A reflecting telescope: Figure b . A parabolic flashlight:


light rays parallel to the a light source at the focus
axis are concentrated at sends out beams of light
the focus. parallel to the axis.

Figure a shows the application of the reflective property of a parabola to create a reflecting telescope. The eyepiece
of the telescope is placed at the focus F of a parabolic mirror. Light enters the telescope in rays that are parallel to
the axis of the parabola. We know from physics that when light is reflected, the angle of incidence equals the angle
of reflection. Hence, the parallel rays of light strike the parabolic mirror so that they all reflect through the focus,
which means that all the parallel rays are concentrated at the eyepiece. This maximizes the light-gathering ability of
the mirror.
Flashlights and automobile headlights (see Figure b) simply reverse this process. A light source is placed at the focus
of a parabolic mirror. The light rays strike the mirror with an angle of incidence equal to the angle of reflection, and
each ray is reflected along a path parallel to the axis. As a result, the light emits a light beam of parallel rays.

Radar utilizes both of these properties. First, a pulse is transmitted from the
focus to a parabolic surface. As with a reflecting telescope, parallel pulses are
transmitted in this way. The reflected pulses then strike the parabolic surface and
are sent back to be received at the focus.

202 Algebra 8
EXERCISES 5 .1
A. Graphing y = ax2 6. The equation of the axis of symmetry of the
1. Graph each set of functions in the same plane. parabola y = 2x2 + (m + 1)x – 4 is 4x – 3 = 0.
Find the minimum value of the function.
a. y = 2x2, y – 4x2 = 0, 2x2 = 3y
b. y = –3x2, y + 4x2 = 0, –3x2 = 2y
c. y = 3x2, 3y = x2, 2y + x2 = 0, y + 2x2 = 0
4
7. – is the minimum value of the function
3
2. Determine whether each point lies on the graph y = 3x2 + 2x – n. Find the value of n.
of the function 4y + 5x2 = 0.

 45  4 
a.  3, –  b.  , 1 c. (–2, –5) d. (4, 10)
 4  5  8. The vertex point of the parabola
y = –3x2 – 2(2k + 5)x + 4 is on the y-axis.
3. If the parabola y = px2 passes through the following Find k.
points, find p.

 4 4 
a. (–3, 3) b.  2,  c. (3, –7) d.  , –2 
 5 5  9. Find the x- and and y-intercepts of each function.

a. y = –x2 + 4x b. y = (x – 3)2
2
4. Graph the function y =  x if x  4. c. y = 8(x + 7)2 + 4 d. y = x2 – 8
2
e. y = 2x2 + 5x + 3 f. y = –x2 – 4x – 5
2 2
g. y = 16x2 + 24x + 9 h. y = – x –4
B. Graphing y = ax2 + bx + c 3
5. Find the coordinates of the vertex point of each
function. Write the equation of the axis of symmetry
and determine the maximum or minimum value 10. The function y = –x2 + (m – 2)x + m – 3 is given.
of the function. Find the values of m for which the parabola

3 2 a. does not cut the x-axis.


a. y = x b. y = –3x2
4 b. is tangent to the x-axis.
c. y = 3x2 + 2 d. y = –x2 – 1
c. cuts the x axis at two distinct points.
e. y = –2x2 + 5x f. y = 0, 7x2 + 0,8x
g. y = 3x2 – 4x + 3 h. y = –x2 + x + 1
i. y = 3(x + 1)(x – 4) j. y = –2(x + 4)2 11. The parabola y = mx2 – (m + 1)x + 2m – 1 passes
2 2
k. y = –(x + 3) – 5 l. y = (x – 1) + 3 through the point A(2, 9). Find the value of m.

Quadratic Functions 203


12. Sketch the graph of each function. 22. Sketch the graph of each function by shifting.

a. y = 3x2 – 9 b. y = 2x2 + 5 a. y = x2 + 4 b. y = x2 – 2
c. y = –3(x + 4)2 d. y = 2(x + 7)2 c. y = –x2 – 2 d. y = –(x + 5)2
e. y = x2 – 2x – 3 f. y = –x2 – 4x e. y = (x – 4)2 f. y = –(x – 3)2
g. y = x2 – 4x + 1 h. y = x2 – 5 g. y = –(x + 1)2 – 2 h. y = (x – 2)2 + 3
i. y = (x – 3)2 + 4 j. y = (x + 3)(4 – x)

23. Sketch the graph of each function.


13. Sketch the graph of the function
y = x2 – 2x – 8 if –1 < x < 5. a. y = |x2 – 4x + 3| b. y = –2x2 + 4|x| + 1
c. y = x|x – 2| d. y = |x2 – 6x + 8| – 1
14. Sketch the graph of the function
y = –2x2 + 4x + 1 if |x – 1| < 2.
C. Equation of a Parabola
15. Find the minimum and the maximum values of 24. Write the equation of each parabola.
the function y = x + 4x + 7 if x  [–4, 1].
2
a. y b. y
3
2
16. Find the minimum or the maximum value of the 1
function if m + n = 16. 5
1 x 6 x

17. Find the minimum or the maximum value of the


function y = m2 + n if m – 2 = n. c. y d. y
3 4
3
18. The parabola y = 2ax + bx – 3b passes through
2

the points (1, –3) and (2, –5). Find the values of
-3 x -1 x
a and b.

19. Write the equation of the parabola which is e. y f. y


symmetric to y = 3(x + 1)2 with respect to the 3 5 3
line x = 2. x

-3
-3 3 x
20. Given f(x) = –3(x – 1)2 + 2, show that f(x)  f(1).

21. f(x) = x2 + 3mx + 4m and f(x)  0 are given.


Find m. -8

204 Algebra 8
25. Write the equation of the parabola through each 30. The figure shows the graph y
set of points. 
of the function
a. A(–1, 0), B(0, 4), C(1, 2) y = mx2 – 7mx + 5.
b. A(0, –4), B(2, 0), C(4, –4) If |AB| = 3, find m.
A B x

26. Write the equation of the parabola through each


vertex point V and point A.

a. V(0, –4), A(3, 5) b. V(2, 0), A(4, 4) 31. Find the area of the y
 y=–2x2+6x
 1 15  trapezoid OABC in
c. V(–1, –4), A(2, 5) d. V   ,   , A(0, –4) C
5 B
 2 4  the figure. 2

A
O x

27. In the figure, y

3  |OB| = |AO|.
y=–x2 – 4x – c
Find c.
A B
O x

32. Find the area of the y



square OABC in the figure.

24
28. In the figure, y B
 C
V
|OB| = 5  |AO| and k
O A 2 x
V is the vertex point
2
of the parabola.
A B
Find k.
O 2 x

33. Find the area y



of the
29. The figure shows the y rectangle D C
 y=x2 – 6x+5
graph of the function ABCD in
y = ax2 + bx + c. the figure.
-5 1 x O A B x
Find the value of
a + b + c.

Quadratic Functions 205


Mixed Problems 39. The figure shows the y
 y=–x2+bx+1
graph of the function 5

34. x1 and x2 are the x-intercepts of the function y = –x2 + bx + 1.


c
 Find the value of
y = mx2 – 4x – 2m – 2. If x1  x1 < 0, find the
a + b + c + d.
possible values of m.
a 2 d x

35. The perimeter of a rectangle is 32 cm. Find its



maximum possible area.

40. In the figure, y



36. The function y = 5x – 3x + k is tangent to the
2
the area of the y=k(x2+5x+4)

line y = 2. Find k. triangle ABC is
C
6 cm2.
Find k. A B O x

37. 3 is the minimum value of the function



y = mx2 – 2mx + 2m + 1. Find m.

38. The figure shows the y



6
41. In the figure, the y
graph of the function 
maximum value of the C
y = –x2 + bx + c.
49
Find the coordinates function is . A B
4
-2 O 5 x
of the point A. 2
Find the area of the
A B x triangle ABC.

206 Algebra 8
An equation is a statement that says two expressions are equal. For example, a = b is an
equation. An inequality is a statement that says two expressions may or may not be equal. For
example, a > b, a  b, and a < b are all inequalities. An equation usually has a finite number of
solutions, but an inequality may have an infinite number of solutions.
We can show the solutions of an inequality as an interval and as a graph. Look at the following
examples of intervals and their graphs.

Interval Inequality Graph

the interval (–, a) x<a a

the interval (–, a] xa a

the interval (a, ) x>a a

the interval [a, ) xa a


Trichotomy Property:
For any two real numbers
a and b, exactly one of the open interval (a, b) a<x<b a b
the following is true:
a < b, a = b or a > b. the closed interval [a, b] axb a b

the half-open interval [a, b) ax<b a b

the half-open interval (a, b] a<xb a b

the interval (–, ) 

Notice in the table that an open circle on a graph shows an open interval, i.e. a line segment
with an open endpoint (a point which is not included in the segment). A filled circle shows
a closed endpoint (a point which is included in the segment).

EXAMPLE 1 Write each inequality using interval notation and show it as a graph.

a. 1  x < 5 b. x > 7 c. –3 < x < 4 d. x  9


1 5
Solution a. The interval notation is [1, 5) and the graph is .
7 –3 4 9
b. (7, ) c. (–3, 4) d. (–, 9]

208 Algebra 8
Definition linear inequality
A linear inequality is an inequality that can be written in one of the forms
ax + b > 0, ax + b < 0
ax + b  0, ax + b  0
for the real numbers a and b, a  0.

Property If both sides of an inequality are multiplied or divided by a negative number, the direction of
the resulting inequality must be reversed.

For example, if we multiply both sides of the inequality a < b by –2, we obtain –2a > –2b.
The order of the inequality is reversed.

EXAMPLE 2 Solve the inequality 2x + 3  5x and graph this solution.

Solution 2x + 3  5x
–3x  –3
1x
1
Therefore, x  [1, ), or .

EXAMPLE 3 Solve the inequality 2 < 3x – 1  8 and graph this solution.

Solution 2 < 3x – 1  8
3 < 3x  9
1<x3
1 3
Therefore, x  (1, 3], or .

Property For k > 0, the following statements are true.


1. |x|  k means –k  x  k.
2. |x| > k means x < –k or x > k.

EXAMPLE 4 Solve the inequality |2x – 1| < 5 and graph this solution.

Solution |2x – 1| < 5


–5 < 2x – 1 < 5
–4 < 2x < 6
–2 < x < 3
Therefore, x  (–2, 3), or –2 3 .

Quadratic Inequalities 209


Check Yourself 1
Solve the inequalities.
1. –x + 7 > 2x + 1
2. 11  x+ 3  17
4
3. |2x – 1| + 2  7
Answers
1. x < 2 2. –47  x  65 3. –2  x  3

FIND THE MISTAKE!

a>4
4a > 16
4a – a2 > 16 – a2
a(4 – a) > (4 – a)(4 + a)
a>4+a
0>4
Can you find the mistake in this working?

Sign Chart
In general, to solve a linear inequality such as ax + b > 0 or ax + b  0 we need to know the
sign of the polynomial ax + b, a  0.
Look at the steps.
First we find the zero of the polynomial:
b
ax+ b = 0 ; x =  .
a
Then we construct a sign chart.
b
x –¥ x0= – ¥
a
ax+b ax+b has sign opposite to a ax+b has the same sign as a

This sign chart shows us:


 b
if x    ,   , the sign of the polynomial is opposite to the sign of a,
 a 
 b 
if x    ,   , the sign of the polynomial is the same as the sign of a.
 a 

210 Algebra 8
EXAMPLE 5 Solve the inequality 3x – 2  0.

2
Solution Find the zero: 3 x  2 = 0 ; x = .
3
Draw the sign chart:
2
x –¥ x0= ¥
3
3x – 2 – +

 2
If x   ,  , then 3 x2 is negative.
 3

2 
If x   ,   , then 3 x2 is positive.
3 
2  2 
Since 3 x – 2  0, x is in the interval  ,   , i. e. x   ,  .
 3   3 

EXAMPLE 6 Solve the inequality 6 – 2x  0 by using a sign chart.

Solution 6 – 2x = 0 ; x = 3

x –¥ 3 ¥

6 – 2x + –

Since 6 – 2x  0, the sign is positive. So x  (–, 3].

Check Yourself 2
Solve the inequalities by using a sign chart.
x+ 14 x  12
1. 5 – 2x < 0 2.  3 3. 6x + (x – 2)(x + 2)  (x + 4)2
6 8
Answers
5
1. x > 2. x  –20 3. x  –10
2

Mathematics, of course, is not the only cornerstone of opportunity in today’s


world. Reading is even more fundamental as a basis for learning and for life. What
is different today is the great increase in the importance of mathematics to so
many areas of education, citizenship, and careers.

Quadratic Inequalities 211


A rectangular room has dimensions 30 m  12 m  12 m. A spider is in the horizontal center of one end wall,
one unit away from the ceiling. A fly is in the horizontal center of the opposite wall, one unit away from the floor.
What is the shortest distance the spider needs to travel (without leaving a surface) to get to the fly (which
remains stationary)?

EXERCISES 6 .1
1. Write each inequality using interval notation. 3. Solve and graph the inequalities.
a. x > –2 b. x  6 a. –2x + 3 < 4
c. x < –7 d. x  4
x x x x
e. 2  x < 4 f. –1 < x  0 b. +  
2 3 4 5
1 3
g. –5 < x < h. 0 ,5  x  3x x
2 2 c.  + 5 < 1+
2 2
x+ 3 x  2 2x
d.   
2. Determine the sign of each polynomial. 4 2 3
e. (3x + 1)2 – (x + 2)(4x – 1) > 5(x – 1)2 + 6x
a. 4x + 1 b. –3x – 5
x 7 f. (a + 3)x – 5  1 (a > –3)
c.  + d. ñ3x – 6
2 3 g. (a4 + 4)x + 3 > 0 (a  )

212 Algebra 8
Definition quadratic inequality
A quadratic inequality is an inequality that can be written in one of the forms
ax2 + bx + c > 0, ax2 + bx + c < 0,
ax2 + bx + c  0, ax2 + bx + c  0,
for the real numbers a, b, and c, a  0.

We have seen how to solve linear inequalities such as


We will never
be equals. 2(x + 3) + 3  5(x – 1).
But how do we solve quadratic inequalities such as
x2 – 5x  –6?
First we write the inequality in standard form, leaving only zero on the right side:
x2 – 5x + 6  0.
In this example, we are looking for values of x that will make the quadratic on the left side
greater than or equal to zero.

Note
If  and  are used in the inequality, then remember that the zeros of the polynomial are
included in the solution set.
First, let us find the zeros of the polynomial x2 – 5x + 6:
(x – 2)(x – 3) = 0

positivepositive=positive x = 2 or x = 3.
positivenegative=negative
negativenegative=positive Then we construct a sign chart for each linear factor of the polynomial, and their product.
x –¥ 2 3 ¥

x–2 – + +

x–3 – – +

(x – 2)(x – 3) + – +

Since x2 – 5x + 6  0, we need to take the positive intervals.


Therefore, the solution is the union of intervals (–, 2] and [3, –), i.e. x  (–, 2]  [3, ).

Quadratic Inequalities 213


We can also construct the sign chart in one step. The zeros of the polynomial divide the real
number line into three intervals, (–, 2], [2, 3] and [3, ). We know that the polynomial has
constant sign in each of these three intervals. If we select a test number in each interval and
evaluate the polynomial at that number, then the sign of the polynomial at this test number
must be the sign for the whole interval.
Let us try testing each interval in our problem. Choose a number from each interval, and
substitute for x in the original inequality. For example, we could choose the numbers 1, 2.5,
and 4.
Test number 1 2.5 4

Value of the polynomial 2 –0.25 5

Sign of the polynomial + – +

With this information, we can draw the sign chart.

x –¥ 2 3 ¥

x2 – 5x + 6 + – +

This is the same as the last line of the previous chart.


We can also use the discriminant of a quadratic to help complete the sign chart. We have
already seen that the discriminant  of a quadratic equation ax2 + bx + c = 0 tells us it the
equation has two real roots ( > 0), one double root ( = 0), or no real solutions ( < 0).

If  < 0, then the polynomial ax2 + bx + c always has the same sign as a.

x –¥ ¥

ax2 + bx + c same sign as a

If  = 0, then of the polynomial ax2 + bx + c has the same sign as a but we must consider
the zero of the polynomial.

b
x –¥ x1=x2= – ¥
2a
ax2 + bx + c same sign as a same sign as a

If  > 0, the polynomial ax2 + bx + c has the opposite sign to a between the zeros of the
polynomial and the same sign as a in other intervals.

x –¥ x1 x2 ¥

ax2 + bx + c same sign as a opposite sign to a same sign as a

214 Algebra 8
EXAMPLE 7 Solve the inequality –x2 – 4 < –5x.

Solution –x2 – 4 < –5x


–x2 + 5x – 4 < 0
(–x + 4)(x – 1) = 0
x = 4 or x = 1

x –¥ 1 4 ¥

–x2 + 5x – 4 – + –

Therefore, x  (–, 1)  (4, ).

EXAMPLE 8 Solve the inequalities.


2x  1
a. 3x + 4  x2 b. x(x + 2) > 35 c. 9x2 – 12x + 4  0 d. x2  <0
4
Solution a. 3x + 4  x2 ; –x2 + 3x + 4  0
 = 9 + 16 = 25 ; x1 = –1, x2 = 4

x –¥ –1 4 ¥

–x2 + 3x + 4 – + –

So x  [–1, 4].
Plot the roots in the sign
chart line in ascending b. 35 < x(x + 2) ; –x2 – 2x + 35 < 0
order.
 = 4 + 140 = 144 ; x1 = –7, x2 = 5

x –¥ –7 5 ¥

–x2 + 3x + 4 – + –

So x  (–, –7)  (5, ).

c. 9x2 – 12x + 4  0
12 2
 = 144 – 144 = 0 ; x1 = x2 = =
18 3
2
x –¥ 3 ¥

9x2 – 12x + 4 + +

2
So the only solution is x = .
3

Quadratic Inequalities 215


2x  1
c. x2  < 0 ; 4x2 – 2x + 1 < 0
4
 = 4 – 16 = –12 < 0.

x –¥ ¥

4x2 – 2x + 1 + + + + +

4x2 – 2x + 1 is always positive, so there is no solution.

EXAMPLE 9 Consider the equation x2 – 2(m + 1)x + 1 = 0. For which values of m does the equation have

a. no real root?
b. one double root?
c. two distinct real roots?

Solution Let us check the sign of discriminant.


 = 4(m + 1)2 – 4 = 4m2 + 8m + 4 – 4 = 4m2 + 8m
4m2 + 8m = 0
m = –2 or m = 0

m –¥ –2 0 ¥

4m2 + 8m + – +

Now we can answer the question.

a. For  < 0, there is no real root, and m  (–2, 0).


b. For  = 0, there is one double root, and m = –2 or m = 0.
c. For  > 0, there are two real roots, and m  (–, –2)  (0, ).

EXAMPLE 10 Solve the inequalities.

a. (3 – x)(x3 – 2x2 – 8x)(x2 + 3) < 0


(3x+ 2)(x  5)
b. 0
x(x – 1)(x2 + x +1)
(x2  x  6)(x  1)17
c. 0
x8 (1  x2 )55

216 Algebra 8
Solution a. First we find all the zeros of the polynomials, then we determine the sign for each poly-
nomial and multiply the signs of each polynomial.
(3 – x)(x3 – 2x2 – 8x)(x2 + 3) = (3 – x)x(x2 – 2x – 8)(x2 + 3)
3–x=0; x=3
x=0
x2 – 2x – 8 = 0 ; x = –2 or x = 4
x2 + 3 = 0 ; no real solution.
x –¥ –2 0 3 4 ¥
3–x + + + – –
x – – + + +
x2 – 2x – 8 + – – – +
x2 + 3 + + + + +
(3 – x)x(x2 – 2x – 8)(x2 + 3) – + – + –

We need a value less than zero, so x  (–, –2)  (0, 3)  (4, ).
b. First we find all the zeros of the polynomials. The equality part of the original inequality is satisfied
for these zeros and they must be included in the final solution set. On the other hand, since
positive division by zero is never allowed, the zeros of x4 – x must not be included in the solution set.
= positive
positive
(3x+ 2)(x  5)
positive
= negative x(x  1)(x2 + x+ 1)
negative
2
negative
= positive 3x+ 2 = 0 ; x = 
negative 3
x5=0 ; x=5
x=0
x  1= 0 ; x = 1
x2 + x+ 1 = 0; no real solution.
2
–
x –¥ 3 0 1 5 ¥
3x + 2 – + + + +
x–5 – – – – +
x – – + + +
x–1 – – – + +
x2 +x+1 + + + + +
(3x + 2)(x – 5)
+ – + – +
x4 – x

 2
We need a value greater than or equal to zero, so x    ,    (0 , 1)  [5 , ).
 3

Quadratic Inequalities 217


(x2  x  6)(x  1)17
c. 0
x8 (1  x2 )55
x2 – x – 6 = 0 ; x = –2 or x = 3
(x – 1)17 = 0 ; x – 1 = 0 ; x = 1
x8 = 0 ; x = 0 (double root)
(1 – x2)55 = 0 ; 1 – x2 = 0 ; x = 1 or x = –1 (1 is also a double root)
If the power is an odd
number, you can ignore x –¥ –2 –1 0 1 3 ¥
it when you calculate the
zero. If the power is an x2 – x – 6 + – – – – +
even number, consider it
just as 2 when you calculate (x – 1)17 – – – – + +
the zero.
x8 + + + + + +

(1 – x2)55 – – + + – –
(x2 – x – 6)(x – 1)17
+ – + + + –
x8(1 – x2)55

We need a value less than or equal to zero, so x  [–2, –1)  [3, ).

The signs of M  N and Let us summarize the key steps to solving any inequality.
M
, N  0 are the same. 1. Write the polynomial inequality in standard form.
N
2. Find all zeros of the polynomial(s).
3. Determine the character of the roots.
4. Determine the sign of the coefficient of leading term of the polynomial(s).
5. Construct a sign chart.
6. In the sign chart, from right to left start with the sign of the coefficient of the leading term
a. After each root change the sign, but if there is a double root do not change the sign.

Let us solve Example 10c in another way.

EXAMPLE 11 Solve the inequality


(x2  x  6)(x  1)17
x8 (1  x2 )55
 0.

Solution x2 – x – 6 = 0 ; x = –2 or x = 3
(x – 1)17 = 0 ; x – 1 = 0 ; x = 1
x8 = 0 ; x = 0 (double root)
(1 – x2)55 = 0 ; 1 – x2 = 0 ; x = 1 or x = –1 (1 is also a double root)

218 Algebra 8
x –¥ –2 –1 0 1 3 ¥
(x2 – x – 6)(x – 1)17
+ – + + + –
x8(1 – x2)55

Therefore, x  [–2, –1)  [3, ).

EXAMPLE 12 Solve the inequality (x2 – 2x – 8)(x2 + x – 12)  0.

Solution Find the zeros of the polynomials.


x2 – 2x – 8 = 0 ; x = 4 or x = –2
x2 + x – 12 = 0 ; x = –4 or x = 3
x –¥ –4 –2 3 4 ¥

(x2 – 2x – 8)(x2 + x – 12) + – + – +

So, x  [–4, –2]  [3, 4].

x+ 2
EXAMPLE 13 Find the domain of the function y =
3x  12 x2
.

Solution 3x – 12x2 > 0


3x – 12x2 = 0
1
x = 0 or x =
4
1
x –¥ 0 4 ¥

3x – 12x2 – + –

 1
So x   0 ,  .
 4

EXAMPLE 14 Solve the inequality (x – 1)2(x – 2)3(x – 3)4(x – 4)5  0.

Solution (x – 1)2 = 0 ; x – 1 = 0 ; x = 1 (double root)


(x – 2)3 = 0 ; x – 2 = 0 ; x = 2
(x – 3)4 = 0 ; x – 3 = 0 ; x = 3 (double root)
(x – 4)5 = 0 ; x – 4 = 0 ; x = 4
x –¥ 1 2 3 4 ¥

(x – 1)2(x – 2)3(x – 3)4(x – 4)5 + + – – +

So x  (–, 2]  {3}  [4, ].

Quadratic Inequalities 219


EXAMPLE 15 Solve the inequality
x4  3x3 + 2x2
x3  5x2
< 0.

x4  3x3 + 2x2
Solution <0
x3  5x2
x2 (x  1)(x  2)
<0
x2 (x  5)
x = 0 (double root)
x = 1, x = 2, x = 5
x –¥ 0 1 2 5 ¥
x4 – 3x3 + 2x2
– – + – +
x3 – 5x2

So x  (–, 0)  (0, 1)  (2, 5).

x  1 x+ 1
EXAMPLE 16 Solve the inequality
x

x 1
< 2.

x  1 x+ 1
Solution   2 < 0 ; x  0 and x  1
x x 1
(x  1)2  x(x+1)  2 x( x  1)
0
x(x  1)

2 x2  x+ 1
<0
x(x  1)

2 x2  x+ 1 = 0

1
x = 1 or x =
2
x(x  1) = 0

x = 0, x = 1

1
x –¥ –1 0 2 1 ¥
–2x2 –x+1
– + – + –
x(x – 1)

 1
So x  (–  , –1)   0 ,  (1, ).
 2

220 Algebra 8
1  2x
17
1 2
EXAMPLE Solve the inequality   .
x+ 1 x2  x+ 1 x3 + 1

1 2 1  2x
Solution   0 ; x  1
x+ 1 x2  x+ 1 x3 + 1
x2  x+ 1  2(x+ 1)  (1  2 x)
0
(x+ 1)(x2  x+ 1)

x2  x  2
0
(x+ 1)(x2  x+ 1)

x2  x  2 = 0 ; x = 1 or x = 2
x + 1 = 0 ; x = –1 (double root)
x2 – x + 1 = 0 ; no real solution
x –¥ –1 2 ¥
x2 –x–2
– – +
(x + 1)(x2 – x + 1)

So x  (–, –1)  (–1, 2].

2 x2 + 3x  2 . (1  x2 )
EXAMPLE 18 Solve the inequality
(x2 + 3x) . x  2
 0.

Solution Since |2x2 + 3x – 2| and |x – 2| are non-negative, just check their roots.
1
2x2 + 3x – 2 = 0 ; x = –2, x =
2
x–2=0; x=2
The absolute value of a 1
number is never negative.
–2 and satisfy the inequality, so these values are in the solution set. However, 2 is not in
2
|x|  0, x   the solution set because it makes the denominator zero.
1 – x2 = 0 ; x = –1, x = 1
x2 + 3x = 0 ; x = 0, x = –3
x –¥ –3 –1 0 1 ¥
|2x2 + 3x – 2|(1 – x2)
– + – + –
(x2 + 3x)|x – 2|

1
So x  (–, –3)  [–1, 0)  {–2, }  [1, 2)  (2, ).
2

Quadratic Inequalities 221


EXAMPLE 19 Find the domain of the function y = 4
x2  6x  16
x2  12 x+ 11
+ 7 3x4  4x3 + x  1 if y is a real number.

7
Solution 3x4  4x3 + x  1 is a real number for all values of x because the index is an odd number.

x2  6 x  16 x2  6 x 16
4 is a real number if 0
x2  12 x+ 11 x2 12 x+11
Therefore, y is a real number if
x2  6x  16
 0.
x2  12 x+ 11
x2  6 x  16 = 0 ; x = 2 , x = 8
x2  12 x+ 11 = 0 ; x = 1, x = 11
x –¥ –2 1 8 11 ¥
x2 – 6x – 16
+ – + – +
x2 –12x + 11

So x  (–, –2]  (1, 8]  (11, ).

20
3 x 1 x 3
3
EXAMPLE Solve the inequality 2 x1 < 8 3 x7 .

1
3
3 x 1 x 3
 3 x 1  3 x 3
Solution 2 x 1
<8 3 x7
;  2 x1  < (2 3 ) 3 x7
 
3 x 1 3x  3
3x 1
2 <2 3 x 7

3x  1 3(x  3)
<
If ab < ac then b < c. 3(x  1) 3x  7
3x  1 3 x  9
 <0
3x  3 3x  7
(3x  1)(3 x  7)  (3 x  9)(3 x  3)
<0
(3x  3)(3 x  7)
12 x  20
<0
(3x  3)(3 x  7)
5
12 x  20 = 0 ; x =
3
3x  3 = 0 ; x = 1
7
3x  7 = 0 ; x =
3

222 Algebra 8
5 7
x –¥ 1 3 3 ¥
12x – 20
– + – +
(3x – 3)(3x – 7)

5 7
So x  (   , 1)   ,  .
 3 3

Check Yourself 3
Solve the inequalities.
2 3 4
1. x2 + 5x – 6 > 0 2. (x + 3)3(x – 1)2(x – 4)  0 3. + 
2  x 2 + x 4  x2
2 x
4. (x  2)( x2 + 4) 1
5.    4x  x
2
<0
x2  4 2
Answers
1. (–, –6)  (1, ) 2. (–, –3]  {1}  [4, ) 3. (–2, 2)  [6, )
1
4. (–2, –ñ2)  (ñ2, 2) 5. (–, 0]  [ , )
2

‘Obvious’ is the most dangerous word in mathematics.

EXERCISES 6 .2
1. Determine the sign of each polynomial. 3. For which values of k does the equation
a. x2 – 5x + 4 b. 2x2 + x – 6 x2 + 2(1 – k)x + 1 = 0 have

c. 2x2 – 3x + 4 d. –16x2 + 8x – 1 a. no real solution? b. one double root?


e. –4x2 + 10x – 25 f. 12x2 + 4ñ3x + 1 c. two distinct real roots?

4. For which values of b does the equation


3x2 + (b + 1)x + 1 = 0 have two distinct real
roots?
2. Solve the inequalities.
a. x2 < 9x – 20 b. 4x – 7x2 > 0 5. For which values of a does the equation

c. x(6x + 7)  0 d. 2x – 6 < 3x2 ax2 + (a + 1)x + 2 = 0 have no real root?

16 4 1 6. The product of a number and four plus the


e. 3 x2 + 8 x   f. x(x  1) + 1 < x+
3 5 5 number is less than 15. Find all possible integer
g. (2x – 1)2 > (x – 5)2 values of the number.

Quadratic Inequalities 223


7. Solve the inequalities. 8. Solve the inequalities.

a. x(x – 1)2 > 0
1 1 1 1 1
a. 1  +  +  <0
b. (2 – x)(3x + 1)(2x – 3) > 0 x x2 x3 x4 x5

c. (3x – 2)(x – 3)3(x + 1)3(x + 2)4 < 0 (a2 + a+ 1)x2 + 4


b. >0
a 4 x  a 4 x2
(x  1)(3 x  2)
d. >0
5  2x 3 x (x2 + x+ 3)
c. <0
2  x+ 2 x2  5x+ 2
(x+ 1)(x+ 2)( x+ 3)
e. >0 x3
(2 x  1)( x+ 4)(3  x) d. 2
2
x  5x+ 6
x4 + x2 + 1
f. <0
x2  4x  5

3x  2
g. <3
2x  3
9. Find the domain of each function.
2 x2 + 18 x  4
h. >2
x2 + 9x+ 8 2
a. y =
3 2
x  49
x+ 1 3 1
i.  
x2 x2 2 1
b. y =
x 4
2x 1  2x
j. 3 2
 3
x +x x  3x2 3x  6
c. y =
x+ 2
 1  3
k.  x    x2    0
 x  2 d. 4
(x4  5x3 + 6 x2 )(1  x2 )

1 3
l. >
3x  2  x 2
7x  4  3x2

(x2 + 4x+ 4)(x  2)1001


m. <0
(x  2)2004 x+ 4
12 3
n. |6x2 – 2x + 1|  1 10. Solve the inequality 2
 2 > 1.
 x + 2x x + 2x  2
x+ 2
o. <3
2x  3

x2  3x+ 2
p. >1
x2 + 3x+ 2
4 3
x2  3x  1 11. Solve the inequality x + 3x + 4x2  8
q. 1 2
< 0.
x2 + x+ 1  x

224 Algebra 8
We saw in Chapter 1 that a set of simultaneons equations to solve is called a system of equations.
A system that of equations includes more than one inequality is called an inequality system.
To find the solution of a system, we solve each inequality separately and then find the
intersection of the solutions.

 x2  7 x  8 > 0
EXAMPLE 21 Solve the inequality system 
 x  4x+ 3 > 0
2
.

Solution First find the zeros of each polynomial.


(1) x2 – 7x – 8 > 0
x2 – 7x – 8 = 0 ; x = –1, x = 8
(2) x2 – 4x + 3 > 0
x2 – 4x + 3 = 0 ; x = 1, x = 3
We need to find values so that both polynomials are greater than zero. Let us check the chart.
x –¥ –1 1 3 8 ¥

(1) + – – – +

(2) + + – + +

system

We can see that both polynomials are greater than zero when x  (–, –1)  (8, ). This
is the intersection of the solutions.

 x2 + x  4
EXAMPLE 22 
Solve the inequality system  x
 x2 < 64
<1
.

x2 + x  4 x2 + x  4  x x2  4
Solution (1) <1; <0 ; <0
x x x
x2 – 4 = 0 ; x = 2 ; x = 0
(2) x2 < 64 ; x2 – 64 < 0
x2 – 64 = 0 ; x = 8
x –¥ –8 –2 0 2 8 ¥

(1) – – + – + +

(2) + – – – – +

system

This time, both polynomials need to be less than zero, so x  (–8, –2)  (0, 2).

Quadratic Inequalities 225


 x5  100 x3
EXAMPLE 23 
Solve the inequality system 

(x+ 9)(5 x  x2  18)
 0
.
 x2  18 x+ 45

Solution (1) x5  100x3


x5 – 100x3  0 ; x3(x2 – 100) = 0
x = 0, x = 10
(x+ 9)(5 x  x2  18)
(2) 0
x2  18 x+ 45
x + 9 = 0 ; x = –9
5x – x2 – 18 = 0 ; no solution
x2 – 18x + 45 = 0 ; x = 3, x = 15
x –¥ –10 –9 0 3 10 15 ¥

(1) – + + – – + +

(2) + + – – + + –

system

So x  [–10, –9]  [10, 15).

EXAMPLE 24 Solve the inequality 2 x  1 < x+ 2.

 2x  1  0 (1)
Solution 
 x+ 2 > 0 (2)

2 x  1 < (x+ 2)
2
(3)

(1) 2x – 1  0 (2) x + 2 > 0 (3) 2x – 1 < x2 + 4x + 4


2x – 1 = 0 x+2=0 x2 + 2x + 5 > 0
1 x = –2 x2 + 2x + 5 = 0
x=
2 since  < 0, there is no real solution
If f ( x)  g( x)
1
 f ( x)  0 x –¥ –2 2 ¥

then,  g( x)  0

(1) – – +
 f ( x)  g (x)
2

(2) – + +

(3) + + +

system

1
So x  [ , ).
2

226 Algebra 8
EXAMPLE 25 Solve the inequality x2 + x  2 > x.

Solution Case 1
__________________ Case 2
__________________
 x < 0 (1)  x  0 (1)
 2  2
 x + x  2  0 (2)  x + x  2 > x2 (2)

If f ( x)  g( x) then (1) x < 0 (1) x  0


2
 g( x) < 0
(2) x + x – 2 = 0 ; (2) x – 2 > 0 ;
 x1 = –2, x2 = 1 x – 2 = 0, x = 2
 f ( x)  0
or
 g( x)  0 x –¥ –2 0 1 2 ¥

 f ( x)  g2 (x) (1) – – + + +

Case 1 (2) + – – + +

system

(1) – – + + +

Case 2 (2) – – – – +
system

So x  (–, –2]  (2, ).

EXAMPLE 26 Solve the inequality (x2 + x + 1)2 – 4(x2 + x + 1) + 3 < 0.

Solution For the inequality, we let t = x2 + x + 1. Then the original inequality becomes
t2 – 4t + 3 < 0. First let us solve the inequality for t.
t2 – 4t + 3 = 0 ; t = 1, t = 3
x –¥ 1 3 ¥

t2 – 4t + 3 + – +

So 1 < t < 3. Now solve for x.


1 < x2 + x + 1 < 3
0 < x2 + x < 2, which gives us the system of inequalities
 x2 + x > 0 (1)
 .
 x2 + x  2 < 0 (2)

Quadratic Inequalities 227


(1) x2 + x > 0
x2 + x = 0 ; x = 0, x = –1
(2) x2 + x – 2 < 0
x2 + x – 2 = 0 ; x = –2, x = 1
x –¥ –2 –1 0 1 ¥

(1) + + – + +

(2) + – – – +

system

So x  (–2, –1)  (0, 1).

EXAMPLE 27 Solve the inequality |x2 – 3x + 2|  2x – x2.

Solution Case 1
________________________________________
 x2  3x+ 2  0 (1)

 x2  3x+ 2  2 x  x2 (2)

(1) x2 – 3x + 2  0
x2 – 3x + 2 = 0 ; x = 1, x = 2
(2) x2 – 3x + 2  2x – x2 ; 2x2 – 5x + 2  0
1
2x2 – 5x + 2 = 0 ; x = , x = 2
2

Case 2
_______________________________________
 x2  3x+ 2 < 0 (1)

  (x2  3x+ 2)  2 x  x2 (2)

(1) x2 – 3x + 2 < 0
x2 – 3x + 2 = 0 ; x = 1, x = 2
(2) –(x2 – 3x + 2)  2x – x2
–x2 + 3x – 2  2x – x2
x–20
x–2=0; x=2

228 Algebra 8
1
x –¥ 2 1 2 ¥

(1) + + – +

Case 1 (2) + – – +

system

(1) + + – +

Case 2 (2) – – – +

system

1 
So x   , 1  (1, 2)  {2}, i.e.
2 

1 
x   , 2 .
2 

Check Yourself 4
Solve the systems.

 x2 – 4x  0  x2  4x+ 3 > 0
1.  2.  3. x2  x > 1+ x
 x – 3  0  x2 (x  7)2 > 0
Answers

1. [4, ) 2. x  0, x  7, (–, 1)  (3, ) 3. ( ,  )
3

In the center of a square pond whose side measures 10 m


grows a plant whose top reaches 1 m above the water level. If
we pull the plant toward the bank, its top becomes even with
the waters surface. What is the depth of the pond and the
length of the plant?

Do not worry about your difficulties in mathematics. I can assure


you that mine are still greater. Einstein

Quadratic Inequalities 229


EXERCISES 6 .3
1. Solve the inequality systems. 4. Solve the inequalities.

 x+ 2 > 0  x  1 > 0 x+ 3
a. 2 x+ 10 < 3x  5 b. 2
a.  b.  4x
2 x  3 < 0  x2  2 x  3 < 0
17  15 x  2 x3
x  2 c. x2  x  12 < x d. >0
 x2 + 3x  10 > 0  x+ 1  0 x+ 3

c.  d.  x4
2 x2 + 11x  6 < 0  x2  1 e. x+ 1  x  2  1 f. <x8
 x  3 > 0 x+2

 x2  4x+ 3 < 0 2 x2 + 2 < 5x 5. Solve the inequalities.


e.  f. 
2 x  4 < 0  x2  x
a. x2  9 < x+ 2
 x+ 2 x 1
2

 x+ 1 > 1  <0 b. 2  3x  x2  2 x+ 3
  3x
g.  h. 
 x  x 1 c. x2  x  2  x  1
 x  2 > 2  x+ 1 < 0
d. 4  1 x < 2  x
2. Solve the inequality systems.
e. x2+ x3<4
 x+ 3
3 x <2 6. Solve the inequalities.


a.  x3 < 16 x b. 4x – 2 < x2 + 1 < 4x + 6 a. b.
2+ x  x  1 1+ x  x  1

 4  x2
 c. 6  x2 < x  1

c. 5x  7 < 4  x + 2 3x < 4 7. Solve the inequality


x5 5  x x  25 
x+ 3  4 x  1 + x+ 8  6 x  1 > 1.
3. Solve the inequalities.
a. |4 – 3x|  2 – x 8. For which values of m are the following inequalities

satisfied for any real number x?
b. |x2 – 3x – 15| < 2x2 – x
c. |2x2 + x + 11| > x2 – 5x + 6 a. 2x2 + 5x + m > x2 – x – 7
d. |x – 4| + |2x + 6| > 10 b. 2x2 + mx2 – 2x + m – 1 > (1 + m)x2 – 4x – m
x3 c. (m + 2)x2 – 3x + m – 2 < 0
e. 2
x2  5x+ 6 d. x2 – 6x + 4m2 > 7
f. |2x + 1| – |5x – 2|  1
9. Solve the inequality
x2  x  12 
g.  2x
x3 (x2 + 6x + 14)2 – 9  (x2 + 6x + 15) + 9 < 0.

230 Algebra 8
CHAPTER REVIEW TEST 6A
1. What is the solution of the inequality 5. What is the solution of the inequality
2 – 3(1 – x) < x + 1.8? 1 1
< ?
x+ 2 x  1
A) x > 1.4 B) x < 0.4 C) x > 0.7
D) x > –0.7 E) x < 1.4 A) x < 0 or x > 2 B) –2 < x < 1
C) x < –2 or x > 1 D) 0 < x < 2
E) –2 < x < 2

2. What is the solution of the inequality


x2 + 5x – 14  0?

A) [–2, 7] B) [–7, 2] C) (–7, 2)


D) [2, 7) E) (–2, 7]
6. What is the greatest integer value of x which satisfies
the inequality x2 – 13x + 36 < 0?

A) 13 B) 7 C) –8 D) 8 E) –13

x2  4
3. What is the solution of the inequality > 0?
x2  1

A) (–, –2)  (1, ) B) (–, –2)  (2, )


C) (–, –1)  (1, ) D)  – {–1, 1}
E) (–, –2)  (–1, 1)  (2, )
7. What is the solution of the inequality
1 1 1
+ + < 0?
x x 2 x3

A) (–, 0) B) (0, ) C)  D)  E) {0}


4. What is the solution of the inequality
(2  3x)(3 x  3)
 0?
3x +1

 1 2  1  2 
A)   ,   [1, ) B)  ;     ; 1
 3 3  3  3 

2   1 2
C)    (1, + ) D)   ,   (1, + )
3  3 3 8. How many integer values of x are there which
satisfy the inequality |x + 2|  2 x +7 ?
 1  2 
E)  ;     ; 1
 3 3  A) 1 B) 3 C) 4 D) 5 E) 7

Chapter Review Test 6A 231


9. What is the solution of the inequality 13. What is the domain of the function
(1+ x2 )2001  (1  x)2002 x2
 0? y= ?
( x + 2)2003  (3  x)2005 20  8 x  x2
A) x > –2 B) x < –2 C) x < 3 A) [–1, 7] B) [–10, 2] C) (–10, 2)
D) x > 3 E) –2 < x < 3 D) [5, 4) E) (–2, 10]

14. What is the solution of the inequality system


10. The equation 2x(ax – 4) – x + 6 = 0 has no real
2
 x2  5x +6  0
roots. What is the smallest possible integer value  ?
 x2 + x  2  0
of a?
A) (2, 3) B) [2, 3] C) (–2, 3)
A) –1 B) 2 C) 3 D) 4 E) 5
D) [3, 2) E) (–3, –2)

11. What is the solution of the inequality 15. What is the solution of the inequality system
( x  3)( x + 2)
<1?  x < 2
x2  1  ?
 ( x  3)( x  2)  0
A) (–2, –1)  (1, 3) B) (–2, –1) C) (1, 3)
D) (–5, –1)  (1, ) E) (–1, 1)  (5, ) A) (–2, 2) B) [2, 3] C) (–2, 3)
D) [3, 2) E) (–3, –2)

12. What is the sum of the integer values of x which 16. What is the solution of the inequality
x2  8x +7 3 x  x + 3 >1?
satisfy the inequality < 0?
( x + 2)2 A) (–2, 2) B) (1, ) C) (–2, –3)
A) 32 B) 28 C) 24 D) 20 E) 16 D) (–, 2) E) (2, )

232 Algebra 8
CHAPTER REVIEW TEST 6B
1. What is the solution of the inequality 5. What is the solution of the inequality
x +1 x  1
 > x? x2 > x +1?
2 3

A) x > 1 B) x < –3 C) x < 3  1


A) (–2, 4) B)  1,   C) (–, 2)
 2
D) x < 1 E) x > 6
 1
D) [2, ) E)  ,  
 2

2. What is the solution of the inequality


(x – 6)(x + 3)  2 – 2x?
6. What is the smallest possible integer value of x
A) [–5, 4] B) [–4, 5] C) (–4, 2) which satisfies the inequality (x + 7)2(x – 4)  0?
D) [–5, 4) E) (–4, 5]
A) 3 B) 4 C) –8 D) 8 E) –13

3. What is the solution of the inequality


7. What is the solution of the inequality
x2 + x (x2 + 8x – 9)(x2 – 4)  0?
<1?
5x  3
A) (–, –2)  [1, ) B) (–, –2]  (2, )
 3 3  C) (–, –1]  [1, ) D) R – {–1, 1}
A)   ,   (1, 3) B)  , 1  (3, )
 5 5 
E) (–, –9]  [–2, 1]  [2, )
C) (–, 1) D)  – {–1, 1}

E) (–1, 1)  (3, )

8. Which inequality is represented in this sign


chart?
4. What is the solution of the inequality -2 5

1 5 + – +
+ <1?
2  x 2+ x
A) [–, –1)  (1, 3) B) [3, ] A) x2 – 3x + 10 < 0 B) x2 > 3x – 10
C) (–, –2)  (2, ) D) (1, 3) C) x2 < 3x + 10 D) x2 – 2x + 5  0
E) (1, 3) E) x2 < 2x + 5

Chapter Review Test 6B 233


9. What is the solution of the inequality 13. What is the domain of the function
( x  1)3( x  2) 7
 0? y = 4 x  x2 + ?
( x  3)2 x2
A) (–, 1) B) [2, ] C) [1, 2]
A) (–2, 4] B) [–1, 2] C) (–4, 2)
D) [–1, 2) E) (2, )
D) [2, 4) E) (2, 4]

14. What is the solution of the inequality system


10. The equation 3x2 + bx + 12 = 0 has real roots.
What are the possible values of b?  x5
 >0
 x+ 3
A) [–12, 12] B) (–, –12] C) [12, )  ?
 x+ 7
D) (–, –12]  [12, ) E) (–1, 1)  (5, )  <0
x3

A) (2, 3) B) [–7, 3] C) (–7, –3)


D) [–3, 3) E) (7, –2)

x
11. What is the solution of the inequality  1?
x +1 15. How many integer values of x are there which
satisfy the inequality (x + 8)(x - 1)2(x – 5) < 0?
 1 
A)   ,   B) (–2, –1) C) (1, 3)
 2  A) 8 B) 9 C) 10 D) 11 E) 16
1 
D)  ,   E) (–1, 1)
2 

16. x1 and x2 are the roots of x2 – 3mx + m – 3 = 0.


1 1
+ > 4 is given.
x1 x2
12. What is the sum of the integer values of x which What are the possible values of m?
( x  2)( x  4)x2
satisfy the inequality  0?
( x +1)2 A) (–, ) B) [2, ] C) (–, –3)
A) 8 B) 9 C) 10 D) 11 E) 16 D) (3, 12) E) (0, 12)

234 Algebra 8
EXERCISES 1 .1
1. a. 6 b. 10 c. –11 d. 4x e. 5y f. 11a2 2. a. 3 b. 5 c. 6 d. 9 e. 4x f. 12xy g. aò15 h. 72xy 3. a. 5 b. 3
2 3xy y
c. 2 d. 3x e. f. 4. a. 2ñ2 b. 6ñ2 c. 9ñ3 d. 10ò10 e. 5ñ5 f. xyñx 5. a. 5ñ3 b. 7ñ5 c. 0 d. –2ñ6
3 2x
4
e. 21ñ2 f. 2ñ5 g. 10ñ3 h. –13xñx i. 1 j. 6. a. ñ2 + 1 b. ñ3 – ñ2 c. ñ2 + ñ6 d. 2ñ2 + ñ3 e. ñ7 – 1
3
f. ñ3 + ñ2 g. 2 – ñ3 h. 2ñ2 i. ñ6 j. ñ2 k. ñ3 + 1 7. a. 6 b. 2 c. 4 d. 4 8. a. 2 b. 9 c. 24 9. a. ò15 + ò10
3 2 2 3
b. 7 + ñ7 c. 4 – ñ6 – ñ2 d. 12 e. 6(ñ3 + ñ2) f. 4 g. –1 h. 8 i. 2 j. –2 k. 1 l. ñ3 m. ò13 10. a. ñ3 b.
6
– 11 6 3 12 3 – 8 2 70  5 10
c. d. e. –1 – ñ2 f. 2ñ3 – 4 g. 2 + ñ3 h. i. j. 3 – ñ6 k. ñ5 + 1 l. 1
11 12 23 3

2 33 2 5 3 –1 22
11. a. b. c. 6ñ3 + 7ò11 d. 12. a. ñ2 b. 1 c. 9
6 2 5

EXERCISES 1 .2
1 1 2 1 6 1
– 2
1. a. 212 b. 7 c. 5 7 d. xa 3 e. 215 f. 3 5 g. 3 3 2. a. ña b. 3 b2 c. ( )3 d. c xab e. 4 a 3. a. 6 3 b. 10 a
3
1 27
c. 100 d. ñ5 e. 3 f. 5 g. –3 h. 2x i. a2  6 a j. 2 k. 3 l. 9 m. n. 12 3 o.p. 24 313 q. 24 231
3 4. a. 1
5 5
5 17 39
b. 6 24  33 c. 2 d. e. ñ2 + 1 f. –2 5. a. 4 b. 26 c. 0 d. 54 e. 44 f. 17 g. h. i. 2 j. {–4, 4} k. 10
4 5 2
18
l. 7 m. 9 n. 9 o. ñ6 + 1 p. 16 q. –1 r. s. 2 6. a. 27 b. 55 c. 46 d. 24 e. 32x+1 f. 30x g. y – x h. 2100
7
1 a
7. a. 4 b. c. a d. ( )x 1 e. –2 f. 5 8. a. 256 b. –729 c. –4 d. 64 e. a9 f. a 9. a. 2x+1 b. 0 c. 110  3x – 2
5 b
89 4
d. 2x e. 81 f. g. 1 h. 18 10. a. 6 b. 8 c. 5 d. 9 e. 4 f. { , 4} g. 3 h. 4 i. 3 j. –3 11. 2 12. 268
30 3
1 16
13. ax+2y 14. – 15. –2 16. 128 17. 2 18. x – 2 19. –4 20. 21. a. 7 b. 8ñ3 c. 2.8 d. ñ5 e. 3
4 5
22 2
f. 20 g. 2ñ7 h. 2ñ3 i. 2ñ2 j. 0 k. l. 0 m. ò14 22. 15 23. 0 24. 3 25. 3ñ2 26. 8 –
3 2
19
27. 9 28. 57122 – 57121 29. 30. 2651 31. –121
8

Answers to Exercises 235


EXERCISES 2 .1
1. scores (x) frequency (f) 2. Percent 3.
1 judo
1
35 karate 10%
2 2 5% football
3 1 30 20%
wrestling
4 3 25 7%
5 3 20 basketball
6 5 12%
15
7 3
swiming
8 3 10 29% volleyball
9 1 17%
5
10 2
0 Expenses

Entertaiment

Other
Clothing
Rent

Books
Food

4. 5. a. 2705000 tons b. 2001 c. 1080000 tons


Anchocy
Horse mackerel d. Hazelnut production in Turkey (tons)
700 000
Scad
600 000
Gray mullet
500 000
Blue fish
400 000
Pichard
300 000
Whiting
200 000
Hake
100 000
1999

2000

2001

2002

2003
Other

EXERCISES 3 .1
5 3 8 1 5
1. a. 0 b. 0 c. 0 d. 0 2. a. 0,  b. 0, c. 0, d. 0, 3. a. 0, –2 4. 0,  5. –1, 2 6. a. 4 b. no real solution
2 7 3 9 2
5 4
c.  d. 5 e.  2 5 f.  7. a. –1,5 b. no real solution c. (–1 – ñ5), (–1 + ñ5) d. –2, 4 8. ñ3 9. a. 0, 1
3 5 5
1 5 3 2 3 1 2 5b
b. 0, c. 7 d. 5 e. –1, 2 f. –1, –2 g. 1 h.  , i. , j.  , 10. a. –a, –1 b. no real solution c.  ,1
3 3 2 5 2 4 3 2a
11. a. (2 + ñ5), (2 – ñ5), b. (ñ7 – 2), (–ñ7 – 2) c. (3  ò22) d. no real solution e. no real solution f. no real solution
1 3
g. 3  2ñ2 h. –1  7 12. a. 2  ñ2 b. 1, c. x   d. 5  13 e. no real root f. 1  2 7 g. no real root
6 2 2 6 3
4 7 2 1 8
h. x   i. , 4 j.  , 1 k. , 1 l. –1, 23 13. a. 1  33 b. 3  73 c. 1, 5 d. , 5 14. a. two real roots
5 4 3 26 8 4 5
9
b. two real roots c. one double root d. one double root e. no real solution f. no real solution 15. a. a 
40
9 9 7 7 7 1
b. a  c. a  16. a. m   b. m   c. m   17. – , 1 18. 8 cm, 10 cm
40 40 2 2 2 4
19. 12 cm, 20 cm 20. 4, 6, 8, 10 21. 256 cm2, 441 cm2 22. 7, 49

236 Answers to Exercises


EXERCISES 3 .2
11 6 72 40 10 5 4 5 18 8 24 2 21 17
1. a. ,  b.  ,  c. ,  d. ,  e.  , f. (3ñ2 – 2), 0 2. 3.  4. a. b. 53 c. 
10 5 49 21 3 3 3 6 7 7 7 5 4 9
153 5 2
d. 5. m1 = –5, m2 = 4, x2   6. n1 = –4, n2 = 2, x2   7. 0 8. m = 2n 9. k = 3
256 4 3
14 7 5
10. ,4 11. 5 12.  , 13. –6, 2 14. 1 15. k > 6 16. 6, 7 17. a  b 18. 0
3 3 3

1 5
19. 7 20. c = 2, 1, 21. –13 22.  1 , 1  13 23. 4 + 2ñ3 cm
2 2 2

EXERCISES 3 .3
1. a. x2 – 1 = 0 b. 4x2 – 8x + 3 = 0 c. x2 – 4x = 0 d. x2 – 4x + 2 = 0 e. x2 – 2ñ3x + 1 = 0 f. p2q2x2 – (p2+q2)x+1=0
2. x2 + 2x + 5 = 0 3. x2 – 8x – 12 = 0 4. x2 + x + 27 = 0 5. a. x2 – 4 = 0 b. x2 + 8x + 7 = 0
c. 4x2 – 25 = 0 d. x2 – 2x + 1 = 0 e. x2 + 2mx – 8m2 + 27m – 18 = 0 f. mx2 – 2mx – 8m – 9 = 0 6. 5
4
7. –7, –6, –5 or 5, 6, 7 8. 9 9. 15 hours, 10 hours 10. 3 hours, 2 hours 11. 6 hours 12.
13. 18(ñ2 + 1)
7
14. from A to B 60 km/h, from B to A 80 km/h 15. 160 km/h 16. 16 km/h 17. 7 women 18. 80 km/h
19. 24 hours, 48 hours 20. 20 km/h 21. height = 6 m, base = 12 m 22. 40 kg 25%, 20 kg 40%
23. 60 hours, 84 hours 24. 4 hours, 8 hours 25. 6 m, 8 m

EXERCISES 3 .4
1 3 5
1. a. 2, 3 b. ñ3 c. ò10, 2ñ3 d. –5 ñ7 e. –5 f. 79 g. 4 h. 2 ñ3 i. 2 ñ2 2. a. b. (1 ò10), –1, 3
2 2
1 6  42 7
c. –2, –1, 0, 1 d. 1, 2 e. –6, 1 3. a. 0,  , 1, 5 b. –10, (–4  ñ2), 2 c. 2 d. 5 e. f.  5, , 1 g. no solution
4 3 3

11 – 21
h. 1, 2, 4 i. 0 j. –3 ò15 4. a. 2 b. c. –7 d. 1, 2 e. 12 f. 3 g. –1, 2 h. 0 i. –1, 7 j. no solution
2
8 7
5. a. 7, 8 b. 2 c. 0 d. 0, 2 e. 0 f. no solution g. – , 1 h. 2 6. a. –1 b. , 3 c. 2 d. –1, 5 e. –1, 3 f. no solution
3 3
1 2 1 3 3
g. 3 – ñ7, 2 h. 2 i. no solution 7. a. 0 b.  c. –2, 0 d. 2 e. – , , 2 f. x  (– , ] 8. a. 1 b. 0, 3 c.
2 3 2 2 4
d. –1, 4

Answers to Exercises 237


EXERCISES 3 .5
5 7
1. a. (–6, –18), (18, 6) b. ( , – ) c. (–3, 6), (10, –7) d. (10, 15), (15, 10) e. (4, –1), (–4, 1) f. (3, 3), (–3, –3)
2 2
g. (–3, –4) h. (0, –5), (1, –4)
3
2. a. (–4, –4), (–6, –2) b. (0, 0), ( , 3) c. (0, 0) d. (1, 0) e. (2, 4), (4, 2), (ò22 – 4, –ò22 – 4), (–ò22 – 4, ò22 – 4)
2
7 7 7 7
f. (0, –2), (2, 0) g. (0, ñ2), (0, –ñ2) h. (2, 1), (–2, –1), ( ,– )(– , )
2 2 2 2

EXERCISES 4 .1
1. a. x = –1, y = 4 b. x = 1, y = 0 2. a. {3} b. 1 c. {1, 3, 5, 7, 8, 9, 10} d. {1, 2, 3, 4, 5, 7, 8, 9, 10}
e. {3} f. 2 3. a. (–1, 1], (–3, 2) b. (2, 5), (–3, ) 4. a. A = {1, 2}, B = {a, b, c} b. A = {1, 2}, B = [2, 4]
5. a. {(1, 1), (1, 2), (1, 3), (2, 1), (2, 2), (2, 3), (3, 1), (3, 2), (3, 3)}, {(–1, –1), (–1, 1), (1, –1), (1, 1)}, {(1, –1),
(1, 1), (2, –1), (2, 1), (3, –1), (3, 1)}, {(–1, 1), (–1, 2), (–1, 3), (1, 1), (1, 2), (1, 3)} b. {(December, December),
(December, January), (December, February), (January, December), (January, January), (January, February),
(February, December), (February, January), (February, February)}, {(2, 2), (2, 5), (2, 6), (5, 2), (5, 5), (5, 6), (6, 2),
(6, 5), (6, 6)}, {(December, 2), (December, 5), (December, 6), (January, 2), (January, 5), (January, 6), (February, 2),
(February, 5), (February, 6)}, {(2, December), (2, January), (2, February), (5, December), (5, January), (5,
February), (6, December), (6, January), (6, February)}

6. a. B b. B c. B 7. a. {(0, 0), (1, 1), (2, 4), (3, 9)}


6 4 4
3 b. {(–2, –5/3), (–1, –1), (0, –1/3), (1, 1/3), (2, 1),
4 2
2 (3, 5/3)}
A A
1 4 -2 3
A
1 2 3

8. a. · 1 · b b. · 1 · 0 9. a. y b. y c. y d. y
· 2 · c · 2 · 1
· 3 · d · 3 · 2 3 7 1
· 5 · e · 4 · 3 2
· 5 · 4 1 1
1 x -1
1 2 x x
10. a. {(December, winter), (September, fall),
1
(June, summer), (March, spring)}, {December,
3 x
September, June, March}, {winter, fall, summer, spring}
b. x = 2y – 3, [–43, 65], [–20, 34] c. x = y2 + 2, [2, 11], [–3, 3]

238 Answers to Exercises


11. a. y b. y c. y 12. y
(0, 4) (0, 3)
(4, 2) 1
(2, 0) x (-2, 2) (2, 2) x
-1
(0, -3)
(0, 0) x
(0, 0) x (4, -1)

y
13. {(a, a), (a, b), (b, b), (b, c), (c, a), (c, b), (c, c), (c, d), (d, a), (d, d)} 14. 84 17. 5
4
15. use the fact that n(A) + n(B) includes n(A  B) twice. 16. {(1, a, blue), (1, a, red), (1, b, blue),
(1, b, red), (2, a, blue), (2, a, red), (2, b, blue), (2, b, red), (3, a, blue), (3, a, red), (3, b, blue), 1
-1 3
(3, b, red)}, use three coordinate axes that are perpendicular to each other in the space x

-5

EXERCISES 4 .2
x
1. a. function b. not c. not 2. a. f(x) = 5x + 2 b. f(x) = x2 – 2x c. f ( x) = + 3x3
2
3. a. multiply by 2, then subtract 4 b. add 1, then take the square root of all c. divide by three times itself increased by 1

4. a. –3, 33, 2x2 + 5x, –2x2 + 5x, 2x – 5ñx, 2x2 + 4hx – 5x + 2h2 – 5h b.  1 , 11 , 9 x2  1 ,  6 x + 2 3 x2  1
, ,
2 18 9x + 2 x2 + 2 x4 + 2
6a + 3b  1 2 2 2 4 4 2
8
c. – , undefined, 4u3 +1 , 3 x +22 x + 2 , 6u +1 2 , 6u + 2 4u +1 2 5. 
4a + 4ab + b2 + 2
2
3 8u + 4u x + 3x +5x + 3 u + 2 u u + 4u + 4u 3

6. a. y b. y c. y d. y 7 4
7. a. ; – 7 b. 6, –1; –6 c. 2;  d. 21; –3
3 3 3 5
-2 -1 1 2 2
2
1 1
x
-1 3
1 4 9 –1/2 8. a. function b. not c. function d. function
-1 1 2 x x
-1 x e. not f. not
-4

9. a.  c. (–, 4)  (4, ) d. (–, –3)  (–3, 3)  (3, ) e. [2, ) f. (–, –3)  (–3, 0)  (0, )
b. 
1  10   10 5
g. [5, 6)  (6, ) h. [5, 10] i.  ,1 j. [1, 1)  1,  k. 3, 
3  3 5
   ,   10. a.   ,  b. [4, 1)  ( 1, 3 )
5   2   3   3   2 4 
c. (–1, 1)  (1, ) d. (–, –2)  (4, ) e. [–1, 2] f. [1, 2]  {0} g. (0, 1)  (1, 3) h. [–5, –1)  (1, 5] i. (–, –1]  [11, )

11. a. f ( x) = 3  x b. f ( x) =  x  4x c. f ( x) = x  3  ( x + 2)(1  x)
2 2
j. [–5, –2]  [2, 3)  (3, 5]
x5 2
x + 2x
12. a. – 4 b. –4, 4 c.  13. a. odd b. neither c. even d. neither e. even f. even g. odd h. odd i. neither j. odd
3
14. a. even b. odd c. even d. odd e. neither f. even

Answers to Exercises 239


15. a. (-4,6) y (4,6)
b. y c. y 16. a. y
(-2,4) (2,4)
(-4,4) (0,4) (4,4)
(-7,2) (7,2)
(-7,4) (7,4)
(0,0) (-2,0) (2,0)

x x (-5,0) (5,0) x
(-2,-2) (2,-2) (-7,-4) (7,-4) (0,-2) 2 5 x

y (-5,-6) (5,-6) y b.
(-4,6) (2,4)
y

(-5,6)
y (0,2)
(7,2)
(-7,4) (2,2)
(4,4) (-5,0) (-2,0)
(0,0) (-7,4)
(0,4)
(2,0) (5,0) x
x
(0,-2)
(-2,-2) (-7,-2) -5 2 6 9 11 x
x
(7,-4) (-2,-4)
(7,-4)
(4,-6) (-4,-4) (0,-4)
c. y
(5,-6)

17. consider f(x2) – f(x1) where x1 < x2 on the given interval 18. 3 19. –2
20. a. ,  b. increasing on (–, –2] and [5, ), decreasing on [–2, 5] c. –6, 3, 6; 3
345 7 x
d. –4, 5, 8 e. (–, –6)  (3, 6)

21. a. (-, –10)  (–10, 10)  (10, ), [–7, ) b. –5, 5 c. 2 d. undefined e. [–5, 0)  (0, 5] f. (-, –10) (10, )
g. (–10, –6]  {0}  [6, 10) h. even i. increasing on [3, 6] and (10, ), decreasing on (–, –10) and [–6, –3],
constant on (–10, –6], [-3, 0), (0, 3] and [6, 10) j. –7,  k. [–3, 0)  (0, 3] l. (–6, –3)  (3, 6) 22. 0 23. d(t) = 15t
24. a. F(x) = 1.8x + 32 b. 1.8
 10t
225h2 if 0  h  2  if 0  t  90
25. a. V(t) = 10t b. V( h) =  c. h(t ) =  15
1200 h  1500 if 2  h  3 2+ t  90 if 90  t  210
 120
26. until his twenties the person gained weight and kept his weight constant until his forties, then he lost weight
because of a diet or an illness and after his mid-forties he started to gain weight again. 27. he left home at 8 a.m. and
1
during the day visited four places, probably for a sale, and at 6 p.m. left for home. 28. a. (–, 4] b. [3, ) c.  0, 
 6 
 1
d. (–, 1) (1, ) 29. 100 30. 0, 2, –1 31. –8 32. decreasing on  ,   , increasing on   1 ,  
 5  5
 
3 13 2x 2
33. –x – 3x 34. 35. 12 36.
9 3x

EXERCISES 4 .3
1
1. a. x2 + x,  x2 + x + 2, x3 + x2  x 1, with domain  \ {–1, 1} b. x3 + 4 x2 +5 x, x3 + 2 x2  5 x,
x 1
x2 + 3x x+ 3
x5 +8 x4 +15 x3 , with domain  \ {–5, 0} c. x + 3+ x + 2, x + 3  x + 2, ( x + 3) x+ 2,
x +5 x+ 2

240 Answers to Exercises


x 1 2
d. x  1+ x +1, x  1  x +1, x 1  x +1, 2. a. 25 b. undefined c. –11 3. a. x, x 2 + x +1 ,
x +1 x +x 1
2 x2 + 2 x 7 x +6 11x 5 x + 3 10 x  3
, x4 + 2 x3 + 2 x2 + x b. x4 – 4x3 + 4x2, x2, x2 – 2x + 2, x + 2 c. , , ,
x2  2 x +1 2 x + 3 x + 3 4x +9 13  3 x
x4 41
4. a. f(x) = ñx, g(x) = 2x – 4 c. f(x) = x3 – x + 4, g(x) = 2x2 – x 5. a.
b. f ( x) = x7 , g( x) = b. –1
5 11
c. 4 d. 9 6. 2 7. 2 8. x 9. a. not b. not c. one-to-one 10. a. one-to-one b. one-to-one c. not d. one-to-one
5
6x x+ 4 x +5  x  9  +1
e. one-to-one 11. a. one-to-one b. one-to-one c. not 12. a. b. c. 3 d.  
4 x3 4  3 
13. a. y b. inverse function c. y d. y e. y

(5, 4)
does not exist (1, 4) (7, 5)
(-5, 6) (-5, 6)

(-2, 4)
(-2, 2) (-3, 2)
(3, 0)
x
x (-1, -2) (0, 0) x x

(-3, -6)
(4, -5) (3, -5)
(7, -6)

(-3, -8)
f. inverse function does not exist

2x  1 22 x  1 6x  5 3x  7 2x  1 2x
14. a. 5 b. 0 c. 3 d. –14 e. 7 15. a. b. c. 16. a. b. c. x d. e. 2x – 11
x2 3x 2 x +1 2 2x 3x  2
3x  7 x 1
f. g. 17. a. 1 b. –1 c. 1 d.  17 18. a. 1 b. –2 19. a. find two different x-values that give the same
x5 2x  4 16
y-value b. assume that for x1  x2, f(x1)  f(x2) 20. –2 21. Compare the functions when the argument is x and –x

22. Compare the functions when the argument is x and –x 23. a.  x +9  3 b. x +11+ 4 24. Try to compose

each of the sides with g  f

EXERCISES 5 .1
y y y
1. a. b. -1 0 1 c.
4 3 y=3x2
x
y – 4x2=0
3 2y=–3x2
-
2
2 y=2x2 y=–3x2
-3 1
3 3y=x2
2
3 2x2=3y -4 y+4x2=0 -1 1 x
1
- 2y+x2=0
-1 0 1 x 2

1 1 7 25 -2 y+2x2=0
2. a. yes b. no c. yes d. no 3. a. b. c. – d. –
3 5 9 8

Answers to Exercises 241


y
4. a. 5. a. V(0, 0), x = 0, ymin = 0 b. V(0, 0), x = 0, ymax = 0
-4 -3 -2 -1 0 1 2 3 4
-
1 x c. V(0, 2), x = 0, ymin = 2 d. V(0, –1), x = 0, ymax = –1
2
-2 5 25 5 25 6 6 6 6
e. V( , ), x = , ymax = f. V( , ), x = , ymin =
4 8 4 8 7 5 7 5
9 1 5 1 5
- 2 5 2 5
2 g. V( , ), x = , ymin = h. V( , ), x = , ymax =
3 3 3 3 2 4 2 4
-8 3 75 3 75
i. V( , – ), x = , ymin = – j. V(–4, 0), x = –4, ymax = 0
2 4 2 4
41 5
k. V(–3, –5), x = –3, ymax = –5 l. V(1, 3), x = 1, ymin = 3 6. – 7. 1 8. – 9. a. (0, 0), (4, 0)
8 2
3 3
b. (3, 0), (0, 9) c. (0, 396) d. (–2ñ2, 0), (2ñ2, 0), (0, –8) e. (– , 0), (–1, 0), (0, 3) f. (0, –5) g. (– , 0), (0, 9)
2 4
h. (0, –4) 10. a. m  (–2ñ2, 2ñ2) b. m = 2ñ2 c. m  (–,
–2ñ2)  (2ñ2, ) 11. 3
y y y y y y
12. a. b. 7 c. -4 d. e. 1 f.
4
-ñ3 ñ3 x x 98 -1 3 x
5 -4
-2 x
-3
-1 1 x -48 -4
-9 -7 x

y
y y y y
g. h. i. j. 13. 7 f(A)= [–9, 7)
13 12
1

2 – ñ2 2+ñ2 x -ñ5 ñ5 x -1 1
-3 4 -2 4 5 x
-5
3 x -3 4 x -5

-8
-9
y
9 1
14. 3 15. a. ymax = 12, ymin = 3 16. ymax = 64 17. ymin = – 18. a = – , b =1
4 2
16
-1 3 19. y = 3(x – 5)2 20. –3(x – 1)2 + 2  2; –3(x – 1)2  0; 3(x – 1)2  0 21. m  [0, ]
1 x 9

-5

242 Answers to Exercises


y y y y
22. a. y=x2+4 b. y=x2 c. d.
-5
y=x2 y=x2 – 2 x
-2 x
4
-ñ2 ñ2 x
y=–x2
x -2 -25
y=–x2 – 2
y=–(x+5)2 y=–x2

y y y y
e. y=x2 f. g. h. y=(x – 2)2+3
-1
16 3
y=(x – 4)2 x
x y=(x – 2)2
-1 7
y=x2
-2 y=–x2
-9
4 x 4
y=–x2 y=–(x+1)2
y=–(x – 3)2
y=–(x+1)2 – 2 x
2

y y y y
23. a. b. 3
c. d.
1 7
3
1
1 1 2 x
-1 1 x
1 2 3 x -1

2 3 4
x
-1

1 1
24. a. y = –(x – 1)2 + 3 b. y = x(x – 5) c. y = (x + 3)2 d. y = (x + 1)2 + 3 e. y = –x2 + 4x – 3
6 3
1
f. y = – (x – 3)(x + 3) 25. a. y = –3x2 + x + 4 b. y = –x2 + 4x – 4 26. a. y = x2 – 4 b. y = (x – 2)2
3
18 1 25 9
c. y = (x + 1)2 – 4 d. y = –x2 – x – 4 27. –12 28. 29. 0 30. 31. 32. square units
5 2 4 4
49
33. 10 square units 34. m  (–, –1)  (0, ) 35. 64 square units 36. 37. 2 38. (–3, 0) 39. 9 40. 1
20
41. 35 square units

Answers to Exercises 243


EXERCISES 6 .1
1 3
1. a. (–2, ) b. (–, 6] c. (–, –7) d. [4, ) e. [2, 4) f. (–1, 0] g. (–5, ) h. [0.5, ]
2 2

2. a. x –¥ -1/4 b. x –¥ -5/3 c. x –¥ 14/3


x 7
4x + 1 – + –3x – 5 + – - +
2 3
+ –

1 –1/2 0
d. x –¥ 6/ñ3 3. a. x > – ; b. x  0;
2 x x
ñ3x – 6 – +

c. x > 2; 2
d. x  3; 3
e. x > 2 ; 2/3
x x 3 x

–3
6 6/(a+3) –3 a4+4
f. x  ; g. x > 4 ;
a+ 3 x a +4 x

EXERCISES 6 .2
1. a. x –¥ 1 4 b. x –¥ -2 3/2 c. x

x2 – 5x+4 + – + 2x2+x –6 + – + 2x2 – 3x+4 + + + + + +

d. x –¥ 1/4 e. x f. x –¥ –ñ3/6

–16x2+8x – 1 – – –4x2+10x – 25 – – – – 12x2+4ñ3x+1 + +

4 7 4 4
2. a. (4, 5) b. (0, ) c. (– , – ]  [0, ) d.  e. x = – f. ( , 1) g. (–, –4)  (2, ) 3. a. (0, 2) b. 0, 2
7 6 3 5
c. (–, 0)  (2, ) 4. (–, –2ñ3 – 1)  (2ñ3 – 1, ) 5. (3 – 2ñ2, 3 + 2ñ2) 6. {–6, –5, –4, –3, –2, –1, 0 1, 2}

1 3 2 2 5
7. a. (0, ), x  1 b. (– , – )  ( , 2) c. x  –2, (– , –1)  ( , 3) d. (– , )  (1, )
3 2 3 3 2
1 3 7
e. (–4, –3)  (–2, – 1)  ( , 3) f. (–1, 5) g. (– , )  ( , ) h. (–8, –1) i.  – {2} j. (–, –7]  (–1, 0)  (0, 1]  (3, )
2 2 3

– 6 6 4 1 11
k. (– , ]  [–1, 0)  [1, ] l. (– , 1)  ( , 2) m. (–, 2) – {–4, –2} n. [0, ] o. (– , 1)  ( , )
2 2 3 3 5
1 3
p. (–, –2)  (–2, –1)  (–1, 0) q. [– , 0]  [1, ) 8. a. (0, 1) b. (0, 1) c.  d. [ , 2) 9. a.  – {7, –7}
2 2
b. (0, 16)  (16, ) c. (–, –2)  [2, ) d. [2, 3]  [–1, 1] 10. a. (0, ñ3 – 1)  (–4, –3)  (–ñ3 – 1, –2)  (1, 2)
11. x  0, (–2, 1)

244 Answers to Exercises


EXERCISES 6 .3
3
1. a. (–2, ) b. (1, 3) c. (–6, –5) d. (–1, 1) e. (1, 2) f. [1, 2) g. (2, 4) h. (0, 1) 2. a. (0, 1) b. (–1, 1)  (3, 5)
2
13 3 5
c. (–8, – )  (0, 5) 3. a. (– , 1]  [ ,  ) b. (– , – )  (3, ) c. (–, –5)  (–1, ) d. (–, –4)  (0, )
2 2 3
3 2 2 13 5 3
e. [ , 2) f. [ , ] g. (–, 3) 4. a. (3, ) b. [ , 4) c. [4, ) d. (–3, 1) e. [3, ) f. (9, ) 5. a. [3, ) b. [–2, – ]
2 7 3 5 10 2

13 – 5 417 – 5 –1 13 + 3 – 11 – 1 11+1
c. [2, 3] d. [–15, ) e. [3, ) 6. a. [ , ] b. [–1, 3] c. [– 6, ) ( , 6]
2 64 2 2 2 2
5
7. [1, 5)  (10, ) 8. a. (2, ) b. (1, ) c. (– , – ) d. (–, –2)  (2, ) 9. (–5, –1)
2

TEST 1A TEST 1B TEST 3A TEST 3B


1. B 11. B 1. C 11. D 1. C 9. B 1. B 9. A
2. D 12. B 2. D 12. C 2. E 10. B 2. D 10. E
3. A 13. C 3. B 13. C 3. C 11. D 3. E 11. B
4. D 14. B 4. C 14. B 4. B 12. A 4. A 12. D
5. B 15. C 5. B 15. A 5. E 13. E 5. D 13. B
6. B 16. D 6. D 16. C 6. D 14. C 6. B 14. B
7. C 17. C 7. C 17. C 7. C 15. B 7. A 15. S
8. C 18. D 8. B 18. D 8. C 16. D 8. C
9. C 19. A 9. D 19. B
10. C 20. B 10. D 20. C

TEST 6A TEST 6B
1. E 9. E 1. D 9. C
2. B 10. B 2. B 10. D
3. E 11. D 3. A 11. A
4. A 12. D 4. C 12. B
5. C 13. C 5. E 13. E
6. D 14. B 6. B 14. C
7. A 15. A 7. E 15. D
8. D 16. B 8. C 16. D

Answers to Exercises 245


246 Answers to Exercises
Answers to Exercises 247
248 Answers to Exercises

Вам также может понравиться